OB/GYN exammaster

¡Supera tus tareas y exámenes ahora con Quizwiz!

A women presents to the labor department complaining of contraction every 3 to 4 minutes for the last 3 hours. She is a G1P0 at 40 weeks gestation. Her pregnancy is uncomplicated; her group B strep culture is negative. Physical exam vitals are normal, the baby is cephalic in a +2 station, and the bag of water is intact. The fetal heart monitor reveals fetal heart tones in the 140s with contractions every 3 minutes lasting 45 seconds. Her cervix is 4-cm dilated and 50% effaced. What is the expected rate of cervical dilation? A .5 cm per hour B 1 cm per hour C 1.2 cm per hour D 1.5 cm per hour E 1.7 cm per hour

1.2 cm per hour C This patient is in stage one active labor and is a primigravida. She should expect 1.2 cm of dilatation per hour; multigravida women can expect a faster rate of dilatation.

A 25-year-old woman brings in her menstrual calendar as part of a preconceptional counseling visit. Her cycles are regular, occurring every 30 days and lasting 3-4 days. She has mild cramping on days 1 and 2 that is easily relieved by ibuprofen or acetaminophen. On what day of her cycle is she most likely ovulating? A 12 B 14 C 16 D 18 E 20

16 C Day 1 of menses is the start of a new menstrual cycle. In normally menstruating women, the luteal phase is stable at 14 days, i.e., ovulation ordinarily occurs 14 days before the onset of the next menses. Therefore, in a woman with a very regular 30-day cycle it is most likely to occur on day 16. She may, however, become pregnant if she times intercourse for two days before (B) or after (D) the day of ovulation, due to other factors such as the duration of activity of sperm. Days 12 (A) and 20 (E) are at the margins of the period of fertility, but neither is the day of likely ovulation.

A 25-year-old woman brings in her menstrual calendar as part of a preconceptional counseling visit. Her cycles are regular, occurring every 30 days and lasting 3-4 days. She has mild cramping on days 1 and 2 that is easily relieved by ibuprofen or acetaminophen. On what day of her cycle is she most likely ovulating? A 12 B 14 C 16 D 18 E 20

16 Day 1 of menses is the start of a new menstrual cycle. In normally menstruating women, the luteal phase is stable at 14 days, i.e., ovulation ordinarily occurs 14 days before the onset of the next menses. Therefore, in a woman with a very regular 30-day cycle it is most likely to occur on day 16. She may, however, become pregnant if she times intercourse for two days before (B) or after (D) the day of ovulation, due to other factors such as the duration of activity of sperm. Days 12 (A) and 20 (E) are at the margins of the period of fertility, but neither is the day of likely ovulation.

A 19-year-old female patient presents to her family practice office for her annual Pap test and her first dose of the quadravalent human papillomavirus (HPV) vaccine. As her physician assistant, when would you schedule her to come in for her second dose of the HPV vaccine? A two weeks B two months C three months D four months E six months

2 months People are frequently confused by differences between the quadravalent and bivalent versions of this vaccine. But dosing schedules are not one of the confusing issues. The Centers for Disease Control and Prevention (CDC), Advisory Committee on Immunization Practices (ACIP), and the manufacturers of both the HPV4 and the HPV2 vaccines all agree: The dosing and administration schedules are the same for HPV4 and HPV2. Each dose is 0.5 mL, administered intramuscularly, preferably in a deltoid muscle. The vaccines are administered in a three-dose schedule. The second dose is administered one to two months after the first dose, and the third dose is administered six months after the first dose.

A women is being evaluated at her 36-week obstetrical appointment. She is not obese, her bladder is empty, and she does not have any complications. The fetus is in a cephalic position by Leopold maneuver. You measure her fundal height. What should it measure? A 32 to 34 cm B 32 to 36 cm C 34 to 38 cm D 35 to 39 cm E 32 to 39 cm

34 to 38 cm Fundal height in an uncomplicated, normal weight pregnancy should be within 1 to 2 cm per week of gestation in pregnancies above 20 weeks.

You are caring for a 29-year-old G1P0 at who is pregnant with twins. She has received routine obstetrical care and her pregnancy has been uneventful to date. What is the average gestation age for twins at delivery? A 33 to 34 B 39 to 40 C 35 to 36 D 36 to 37 E 37 to 38

36 to 37 D The average length of gestation for a single fetus is 40 weeks; the average age of gestation decreases with increasing number of fetuses.

A 14-year-old girl presents because she has not yet had her 1st menstrual period; she has no other health problems. Her mother states that her daughter suffers from what seems to be mild mental retardation and attends special education classes. On examination, the girl is 4 feet 5 inches tall, her neck skin shows web-like folds on the side, her breasts are not fully developed, and the nipples are widely separated. No heart murmurs are detected; the lower limb pulse is weaker than that of the upper extremities. What is the most likely karyotype of this girl? Answer Choices 1 45,X 2 47,XYY 3 47,XXY 4 46,XY 5 47,XXX

45, X Explanation Turner syndrome is a leading cause of gonadal (ovarian) dysgenesis. It is caused by sex chromosomal abnormalities resulting in a girl/woman having only 1 X chromosome. The typical karyotype of Turner syndrome is 45,X, but it may also present with mosaic pattern 46,XX, 45,X, and sometimes 46,XX where the 2nd X chromosome is nonfunctional due to an abnormality. Turner syndrome presents in 1 of 2500 live female births. It affects many systems in the body and can be diagnosed prenatally by amniocentesis. It causes short stature, webbed neck, widely separated nipples, and intellectual disabilities. Gonadal dysgenesis leads to amenorrhea; if a mosaic pattern is the case, premature ovarian failure occurs. The gonads are always streak gonads with a small number of ovarian follicles. The pattern 47,XYY is also called super male; it is characterized by intellectual disabilities and large testicles in boys/men. The pattern 47,XXY is Klinefelter syndrome, resulting in a tall boy/man with small gonads and mild intellectual disabilities. The pattern 46,XY is the normal male karyotype. The pattern 47,XXX, is also called super female; it results in tall, relatively normal girls/women who have a lower IQ than their siblings.

In taking the family history of your 25-year-old male patient, you discover that he has numerous relatives with breast and ovarian cancers. In the past, his mother received genetic counseling and testing for the BRCA1 and BRCA2 gene mutations and was found to be positive for a mutant allele. What is his risk for developing this genetic cancer? A His chance is 0%, because this is not transmitted to men. B 25% C 50% D 75% E 100%

50%

You are caring for a pregnant woman who is Rh-negative. The father of the child is Rh-positive and heterozygous. What percent chance will the fetus have of being Rh-positive? A 0% B 25% C 50% D 75% E 100%

50% C The mother is negative and has no genetically positive material to pass to the child; the father is heterozygous, so only half of his genetics contain the positive antigens.

A G4P4 woman delivers a viable infant at 38 weeks gestation by normal spontaneous vaginal delivery. The infant has apgars of 7 and 8 (at 1 and 5 minutes respectively). What is the most crucial time for maternal and fetal physiologic changes to occur? A 30 minutes prior to birth B 60 minutes prior to birth C Birth D 30 minutes after birth E 60 minutes after birth

60 minutes after birth E The hour after birth is the most critical time for physiologic changes to occur, including maternal fluid shifts, hemorrhage, retained placenta, and fetal lung cardiovascular transitions.

A 19-year-old presents to clinic requesting emergency contraception. She is a G1P0Ab1 and a non smoker who has had intercourse and the condom broke. Her LMP was 3 weeks ago. Her PMH is negative. What would be the time frame for maximum efficacy for her to use emergency contraception? A 12 hours B 24 hours C 48 hours D 72 hours E 120 hours

72 hours Emergency contraception, both hormonal and IUD, reduces pregnancy rates for 120 hours, but there is a significant decrease in efficacy after 72 hours.

A 29-year-old G2 P1 who is term requests induction. Her pregnancy has been uncomplicated. She has been bothered by significant, poor quality contractions, which have caused her pain and interfered with her sleep. An elective induction is considered safe when the Bishops score is greater than what number? A 6 B 7 C 8 D 9 E 10

8 C A Bishop score greater than 9 is considered a positive predictor for safe delivery in a term pregnancy.

A 51-year-old male patient presents to your family practice office complaining of genital discomfort with dysuria. His digital rectal exam reveals an enlarged, tender prostate. His prostate-specific antigen (PSA) returns elevated with a value of 11.1 mg/mL, which you fractionate, and this reveals approximately 75% free PSA. His urinalysis reveals moderate white cells and trace blood. What would be your next step in treating this patient? A Begin him on 6 weeks of doxycycline to treat his prostatitis and when resolved, repeat his PSA level. B Immediately refer him to a urologist for prostate biopsy to rule out prostate cancer. C Immediately refer him to a urologist for cystoscopy to rule out bladder cancer and perform a computed tomography (CT) scan of the abdomen and pelvis in the interim. D Order a stat testicular sonogram to rule out torsion. E Order a CT scan of the abdomen and pelvis.

A Begin him on 6 weeks of doxycycline to treat his prostatitis and when resolved, repeat his PSA level. A This patient has signs and symptoms consistent with prostatitis. Additionally, while his PSA is elevated, this is common in prostatits as well as prostate cancer, and his free PSA is of a percentage that prostate cancer is unlikely. However, it would be prudent to recheck his PSA after treatment and resolution of his symptoms to confirm that an underlying cancer is not smoldering.

A 36-year-old female presents to your family practice office concerned about a breast lump she discovered in the shower last night. You obtain a detailed history and these are some of your findings: • G2P2 • LMP: 2 weeks ago, normal • Sister diagnosed with breast cancer at age 42. • Mother is currently being treated for ovarian cancer. • Father is of Ashkenazi Jewish ancestry. Which of the following is an appropriate recommendation for this patient? A Enhanced surveillance, including mammography alternating with MRI, every six months. B Genetic counseling and testing for BRCA1 and BRCA2 status. C Referral to a breast surgeon for mastectomy D Chemoprophylaxis with an aromatase inhibitor weekly.

A Enhanced surveillance, including mammography alternating with MRI, every six months. A Because of her ethnicity (Ashkenazi Jewish descent) and family history (two first degree relatives with breast and ovarian cancers), this patient is at a higher than normal risk for breast and ovarian cancers. With a physical finding of a palpable breast mass, this is even more ominous. She should be referred for an immediate mammogram but not necessarily MRI - and have continued enhanced surveillance annually and not every 6 months regardless of the result - as well as a referral to a breast surgeon for immediate consideration of a biopsy. A mastectomy is a possible consideration but not mandatory. She is certainly an appropriate candidate for consideration of referral to a genetic counselor and genetic testing to ascertain her BRCA1 and BRCA2 status. She is also an appropriate candidate for chemoprophylaxis.

A 37-year-old woman, G3P2 at 30 weeks gestation, complains of lower extremity swelling and her weight is up 5 pounds this week. Her PMH is insignificant, and her other pregnancy was uncomplicated. Her BP baseline is now 142/92. On exam her BP is unchanged, her UA shows 2+ protein, and FHTs are 152. What is the cause of the protein in her urine? A Glomeruloendotheliosis B Glomerulonephritis C Renal vasospasm D Glomerular hemorrhage E Glomerular infarct

A Glomeruloendotheliosis A The classic histological change that occurs in the renal system (in preeclampsia) is swelling and inflammation of the endothelium and of the glomeruli, which leads to endothelial leaking.

A 37-year-old woman, G3P2 at 30 weeks gestation, complains of lower extremity swelling and her weight is up 5 pounds this week. Her PMH is insignificant, and her other pregnancy was uncomplicated. Her BP baseline is now 142/92. On exam her BP is unchanged, her UA shows 2+ protein, and FHTs are 152. What is the cause of the protein in her urine? A Glomeruloendotheliosis B Glomerulonephritis C Renal vasospasm D Glomerular hemorrhage E Glomerular infarct

A Glomeruloendotheliosis The classic histological change that occurs in the renal system (in preeclampsia) is swelling and inflammation of the endothelium and of the glomeruli, which leads to endothelial leaking.

A 19-year-old G1 P0 presents to the emergency department complaining of abdominal pain. Her LMP was 2 months ago, and she has been spotting for the last two days. Her HCG is positive, and transvaginal ultrasound reveals no intrauterine gestational sac. The patient has an ectopic pregnancy, which has likely implanted in the fallopian tube. Why is the fallopian tube the most common non-uterine implantation site? A Lack of sub-mucosal layer in the fallopian tube B Inflammatory response in the endometrium C Proliferative endometrium D Extra myometrial tissue (fibroids) E Excessive trophoblastic proliferation

A Lack of sub-mucosal layer in the fallopian tube A The fallopian tube is the most common site of ectopic pregnancy, accounting for over 95% of ectopic pregnancies. The lack of a submucosal layer allows for easy wall access and implantation of the fertilized ovum. The increasing rate of Chlamydia infections in the U.S. also impacts the physical anatomy of the fallopian tube, and impacts where the ovum implants. Excessive trophoblastic activity does not impact where the ovum implants, and proliferative myometrium enhances uterine implantation if the fertilized ovum makes it to the uterine cavity.

A 33-year-old returns to clinic for reevaluation of her dysmenorrhea. She is a G3P3 who menstruates every 27 days. She has had a tubal ligation. She is a non-smoker. She has tried Tylenol (acetaminophen) and just completed 3 months of ibuprofen 800 mg TID x 7 days, starting 2 days before her menstrual cycle. She had only mild improvement of her symptoms. What should be the next step in management of her symptoms? A Norgestimate/ethinyl estradiol B Acetaminophen/ tramodol C Acetaminophen/ codeine D Cyclobenzaprine E Paroxetine

A Norgestimate/ethinyl estradiol A Primary dysmenorrhea is associated with ovulation, so suppression of ovulation in a woman who no longer desires fertility usually improves symptoms. Narcotics are not indicated as primary treatment in the absence of pathologic disease. Combination oral contraception will suppress ovulation.

A 34-year-old woman presents for her obstetrical checkup. She is a G2P1 and her pregnancy is uncomplicated to date. During her visit, what is the most important predictor of fetal well-being? A Normal maternal vitals and fetal activity B Maternal weight gain and fetal activity C Absence of contractions and fetal activity D Normal maternal vitals and absence of contractions E Maternal weight gain and normal maternal vitals

A Normal maternal vitals and fetal activity A Maternal vitals are a sign of maternal well-being; combined with normal fetal activity, this gives the best predictive value for fetal well-being without direct fetal surveillance.

A 17-year-old female presents to your office with intermittent menstrual pain. She is sexually active with a single male partner, consistently utilizing condoms. She reports that she has had this pain before, most commonly two weeks before her period, and that it has been increasing in severity over the past few months. You perform a pelvic examination and she has no lesions, discharge, or discomfort on bimanual and speculum exam. She is urinary chorionic gonadotropin (UCG) negative. Of the following, what is the most appropriate treatment for this adolescent? A Tell her she has mittelschmerz and prescribe an anti-inflammatory p.r.n. B Tell her she has primary dysmenorrhea and send her home on pain medications. C Recommend that she see a gynecologist to have an exploratory laparoscopic surgery to rule out endometriosis. D Refer her for a vaginal ultrasound to rule out uterine fibroids. E Draw a serum β-hCG.

A Tell her she has mittelschmerz and prescribe an anti-inflammatory p.r.n. Midcycle pain (mittelschmerz) is common in women with regular menstrual periods who are not taking birth control pills. These patients may commonly have midcycle spotting caused by an estrogen surge. There is no fever and no other abnormal bleeding such as that resulting from trauma to the cervix (e.g., coitus, douching). Pain usually occurs over several cycles. There is no history of intermittent lower abdominal pain. Examination at the time of mittelschmerz may reveal some lower quadrant tenderness with or without rebound. Bimanual examination may show localized tenderness. A palpable ovary may be present, but a history of regular menses, lack of fever, and negative pregnancy tests confirm the diagnosis. Mild analgesics, especially nonsteroidal anti-inflammatory drugs (NSAIDs), and reassurance are usually adequate for these patients.

A 53-year-old woman presents to clinic complaining of amenorrhea, irritability, and hot flashes for 6 months. She would like to try hormone replacement therapy. You are counseling her about possible risks versus benefits. Which of the following is a benefit of combined estrogen progestin therapy? A A decreased risk of breast cancer B A decreased risk of myocardial infarction C A decreased risk of stroke D A decrease in somatic symptoms E A decreased rate of cognitive decline

A decrease in somatic symptoms D The WHI study showed increase chance of cardiovascular risks and breast cancer, and showed no improvement in prevention of cognitive decline.

During her active phase of labor, a provider has placed an external fetal monitor on a patient. The fetal heart rate (FHR) is noted to have a base line rate of 109, there are no late or variable decelerations, and the baseline variability is 0 beats per minute. What is this monitor tracing indicative of? A A normal FHR pattern B An indeterminate FHR pattern C An unreadable FHR pattern D Poor contact with monitor E A distress FHR pattern

A distress FHR pattern According to the Three-Tier Fetal Heart Interpretation System, recommended by the 2008 NICHD workshop on electronic fetal monitoring, the definition of fetal distress includes absence of baseline variability, and either bradycardia (FHR<110) or recurrent variable or late decelerations.

A 23-year-old woman presents with complaints of pelvic discomfort and a vaginal discharge for the past 3 days. She finished her period last week. She is taking oral contraceptives as directed. Her medical history is significant for a therapeutic abortion with no other hospitalizations or pregnancies. She has had three sexual partners in the past 6 weeks and does not use condoms. Her most recent partner reported that he was treated recently for gonorrhea. On examination, she has a mucopurulent discharge with "strawberry" cervix on speculum examination. After collecting the appropriate specimens, the best therapeutic option for this patient is A ofloxacin 400 mg i po × 1 dose plus azithromycin 250 mg iiii po × 1 dose B fluconazole 150 mg i po × 1 dose C metronidazole 500 mg iiii po × 1 dose D ceftriaxone 250 mg IM × 1 dose

A ofloxacin 400 mg i po × 1 dose plus azithromycin 250 mg iiii po × 1 dose A Clinical presentation is consistent with cervicitis in a young woman with risk factors for sexually transmitted infection. She has likely been exposed specifically to Neisseria gonorrhea. Coinfection with Chlamydia trachomatis is common. While test results are pending, the Centers for Disease Control and Prevention in its 2006 Guidelines for STD treatment recommend treating for both with single doses (improved compliance) of ofloxacin and azithromycin first-line. In areas with quinolone resistance, intramuscular ceftriaxone is an option but coverage for Chlamydia is still necessary. Metronidazole is the appropriate therapy for trichomoniasis and fluconazole for vaginal candidiasis.

A 22-year-old sexually active woman presents for her annual gynecologic evaluation. She reports one partner for the past 6 months and takes oral contraceptive pills as directed. Her periods have been regular. Her examination is unremarkable and her Pap smear returns with atypical squamous cells of undetermined significance and positive for human papillomavirus-16. The next most appropriate step for this patient is to A proceed with colposcopy B repeat Pap smear in 12 months C repeat Pap smear in 24 months D schedule her for a loop electrosurgical excision procedure (LEEP)

A proceed with colposcopy Human papillomavirus subtypes 6, 11, 16, and 18 increase risk for the development of cervical cancer. In a young woman over 21 years old with atypical squamous cells of undetermined significance and positive HPV 16 subtype, the next step in evaluation is the colposcopic evaluation. Alternatively, she could be followed with Pap smears at 6 and 12 months. The LEEP procedure is indicated for those with recurrent histologic finding of cervical intraepithelial neoplasm grade 2 or 3.

A 23-year-old primiparous woman visits her primary care provider for follow up with her pregnancy. Today her gestational age is 25 weeks. Her pregnancy has been uneventful so far. She has no personal or family history of diabetes mellitus. Her height is 5 feet 9 inches and her weight is 140 lbs. She has no chief complaints. The physician would like to screen for gestational diabetes and opts to perform a 50-g oral glucose challenge test. Regarding performance and interpretation of this test, what is true? Answer Choices 1 A standard fasting 50-g glucose challenge test with a glucose level of ≥100 mg/dL after 1 hour is abnormal and warrants further testing 2 A standard nonfasting 50-g glucose challenge test with a glucose level of ≥135 mg/dL after 1 hour is abnormal and warrants further testing 3 A standard fasting 50-g glucose challenge test with a glucose level of ≥110 mg/dL after 1 hour is abnormal and warrants further testing 4 A standard nonfasting 50-g glucose challenge test with a glucose level of ≥110 mg/dL after 1 hour is abnormal and warrants further testing 5 A standard fasting 50-g glucose challenge test with a glucose level of ≥95 mg/dL after 1 hour is abnormal and warrants further testing

A standard nonfasting 50-g glucose challenge test with a glucose level of ≥135 mg/dL after 1 hour is abnormal and warrants further testing Explanation Gestational diabetes affects approximately 5 - 6% of pregnancies in the United States. Most obstetricians adopt a policy of universal screening of pregnant women between 24 and 28 weeks gestational age using the oral 50 g glucose challenge test. The test is performed by administering a 50-g oral glucose load without fasting before the test. The test has a low specificity with handheld monitors; therefore venous blood should be used to obtain blood samples for glucose measurement. Most clinicians use a value above 135-140 mg/dL to prompt the need for follow-up testing (usually with 3-hour 100-g oral glucose testing, given after an overnight fast).

Your patient is a 16-year-old female who weighs 300lbs and is 5 ft. 2 inches tall. She has severe facial acne vulgaris, hirsutism, and amenorrhea. Her pediatrician has sent her to your 24-hour endocrinology clinical research station, because she thinks that the patient may have polycystic ovary syndrome. Since ovulation occurs after a sequence of steps involving changing blood hormonal levels, you could follow these levels in the patient during an entire ovarian cycle by an indwelling monitoring catheter. What would you look for to rule out the disease? Answer Choices 1 A very large rise in estrogen level during secretory phase 2 A very large drop in progesterone just before ovulation 3 A very large rise in luteinizing hormone (LH) just before ovulation 4 A very large rise in LH just before ovulation with the FSH level dropping during proliferative phase 5 A very large drop in estrogen level just before ovulation

A very large rise in luteinizing hormone (LH) just before ovulation Explanation There is a very large rise in LH level just before ovulation. FSH produces a maturing effect on the follicle and ova. In addition, the follicle produces a protein, inhibin that inhibits FSH production from the gonadotrope. This protein will become important in the events leading up to ovulation. When the follicle and ova are fully matured, LH binds to LH receptors and LH sponsors ovulation. But just prior to ovulation, the LH spike occurs by the following mechanism: at normal levels of the estradiol, the gonadotrope is inhibited, but if the levels of 17b-estradiol are very high, they produce a stimulatory effect on the gonadotrope. Just prior to ovulation, the levels of estradiol rise to extremely high levels. This stimulates the gonadotrope to produce both FSH and LH, but since FSH is inhibited by inhibin, it has a small peak, whereas LH levels rise dramatically. This is called the "LH spike." This dramatic rise in LH triggers ovulation. After ovulation, the follicle changes to become the corpus luteum. Under the primary control of LH, the corpus luteum synthesizes progesterone. Stimulation of the endometrial cells through these receptors leads to vascularization and thickening of the endometrium. These are the preliminary stages of readying the endometrial lining for trophoblast implantation. If fertilization of the ovum does not occur, then a series of events leads to the involution of the corpus luteum. Since the corpus luteum is dependent on LH, it eventually involutes under decreasing levels of LH. Involution of the corpus luteum leads to the decline of estradiol and progesterone levels. This, in turn, leads to the programmed cell death of the endometrial cells lining the uterine cavity and to the process of menses. The inhibition on the gonadotrope cell is released, and the cycle repeats itself.

A 14-year-old boy presents to the emergency department with acute scrotal pain and vomiting for the past 2 hours. His left testicle is in extreme pain and he states the pain started while playing basketball in gym class. On physical exam of the affected testicle, which of the following findings would suggest testicular torsion? A Transillumination B Positive Prehn's sign C Positive cremasteric reflex D Abnormal transverse lie E Mass of enlarged veins palpated

Abnormal Transverse Lie Testicular torsion is most common between ages 12-18 with the classic presentation of abrupt and severe onset of pain with nausea/vomiting. The testicle on physical examination is painful, swollen, high-riding, tender, and has an abnormal transverse lie (D). Transillumination (A) is when light is placed behind the scrotum and fluid is illuminated in cases of hydroceles. Prehn's sign (B) is pain relief with elevating the scrotum and is positive in cases of epididymitis. The cremasteric reflex (C) is a normal finding that causes elevation of the testis on the ipsilateral side when the inner aspect of the inner thigh is stroked. The absence of cremasteric reflex on the affected side is often found in acute torsion. A mass of enlarged veins (E), or "bag of worms," is a finding associated with a varicocele.

A 27-year-old female is 8 weeks postpartum with her first child and has been exclusively nursing since discharge at the hospital. She has a 5-day history of engorgement in her right breast, which is red, tender, and feels warm to the touch. She states she is feverish but has not taken her temperature. On physical examination you see the breast as shown below. Which of the following is the most likely diagnosis? Source: McPhee SJ, Papadakis, MA: Current Medical Diagnosis and Treatment 2011, 50 th Edition: http://www.accessmedicine.com A Galactorrhea B Fibroadenoma C Acute mastitis D Lobular breast cancer

Acute Mastitis C Postpartum mastitis occurs sporadically in mothers that are nursing and usually develops after hospital discharge. The inflammation is usually unilateral and begins with an engorged breast or sore nipple. Frequently, mastitis begins within 3 months postpartum and affects first time nursing mothers more often. Cellulitis is usually seen in the affected breast and redness, tenderness, local warmth, and fevers and chills are common complaints. Fibroadenoma (B) and galactorrhea (A) do not look like this clinical picture. Gynecomastia (E) affects males. Inflammatory carcinoma can in rare cases be mistaken for mastitis. Lobular carcinoma (D) would not present this way.

A 17-year-old G1, P0 girl presents after being found in a crack house by local police. She was initially cooperative, but she is now experiencing severe abdominal pain, and she has developed vaginal bleeding. She says that she is pregnant, but she has not received any prenatal care. Examination reveals a blood pressure of 90/50 mm Hg, pulse of 120/min, and respiratory rate of 25/min. She is diaphoretic and clammy. Her uterus measures 25 cm from the pubic symphysis, but no fetal heart tones can be appreciated. There is a large amount of dark blood around the vagina. Ultrasound reveals an intrauterine fetal demise and a hyperechoic retroplacental hematoma. Labs are pending. Question What is the best course of action at this time? Answer Choices 1 Emergent labor by artificial rupture of membranes 2 Emergent labor induction with Pitocin 3 Administer fibrinogen 4 Emergency cesarean section 5 Administer crystalloids and obtain type and crossmatch for packed RBCs

Administer crystalloids and obtain type and crossmatch for packed RBCs Explanation This patient probably has a grade 3 placental abruption, characterized by external uterine bleeding and fetal demise, as well as maternal hemodynamic instability. Patients with placental abruption should have adequate intravenous access established. Crystalloid fluids are initially used to resuscitate the patient. Blood for transfusion should be typed and crossed (usually at least 4 units). Coagulation studies should be drawn to assess for signs of coagulopathy. Fibrinogen is usually given if the fibrinogen level is <100 mg/dL. Platelets may be administered if the platelet count is <50,000 /uL. Placental abruption (abruptio placenta) is defined as the premature separation from the uterus of a normally implanted placenta. As the placenta separates, there is a large amount of bleeding which irritates the uterus and causes uterine contractions, as well as fetal distress because fetal perfusion is compromised. The incidence of placental abruption ranges from 1 in 75 to 1 in 225 births. Placental abruption is classified by degree of separation. Grade 1 abruption presents with slight vaginal bleeding, mild uterine irritability, and a normal fetal heart rate tracing. Maternal blood pressure is normal, as is the maternal serum fibrinogen level. A grade 2 abruption is characterized by mild-to-moderate vaginal bleeding and pronounced uterine irritability, which may include tetanic uterine contractions. The patient may exhibit orthostatic blood pressure changes and often has an elevated pulse. The fetal heart rate usually shows some evidence of distress. Maternal fibrinogen levels are usually reduced to 150 to 250 mg/dL. A patient with a grade 3 abruption will have moderate-to-severe vaginal bleeding (unless concealed in the uterus), painful, tetanic uterine contractions, and hypotension. The fetus will be dead and the patient is likely to exhibit a coagulopathy with fibrinogen levels less than 150 mg/dL, thrombocytopenia, and reduced levels of clotting factors. Abruptio placenta is associated with several risk factors. Maternal hypertension is one of the most commonly associated risk factors identified in patients who develop placental abruption, especially those with grade 3 abruption (where 40 - 50% of patients will have hypertensive disease of pregnancy). A history of prior abruption, tobacco abuse, cocaine abuse, poor nutrition, and chorioamnionitis are also associated with an increased risk for abruption. Uterine trauma is also associated with a risk for placental abruption, causing 1 - 2% of grade 3 abruptions. Patients with trauma in pregnancy may present with minimal physical evidence of trauma, but they will still have a significant abruption that can progress from grade 1 to grade 3 within 24 hours. Multiple gestations and polyhydramnios can also cause placental abruption if the uterus decompresses rapidly during labor. Placental abruption will classically present with painful vaginal bleeding in the 3rd trimester of pregnancy (80% of patients present with bleeding), although the amount of bleeding may be concealed within the uterus (20%). Ultrasound is used to rule out placenta previa (the other common and dangerous cause of third trimester bleeding) and may identify placental abruption. Unfortunately, the ultrasound appearance of an abruption may lag behind the clinical degree of bleeding, so this imaging modality cannot rule out abruption. The prognosis and management of abruptio placenta depends greatly on the gestational age of the fetus, as well as the grade of the abruption. A grade 1 abruption with a term fetus can often be managed with a controlled induced delivery, and since blood is a strong uterine irritant, many of these patients will deliver relatively quickly. However, the management of preterm pregnancies is less clear and depends on the degree of abruption and fetal distress, with the risks of premature delivery measured against the risks of progression of the abruption.

A 24-year-old woman at 32 weeks of gestation presents with a fever, chills, generalized malaise, and vomiting. T=100.9oF, BP=110/70 mm Hg, P=100/bpm, RR 20/min On pelvic examination, she has acute fundal tenderness, the cervix is 2 to 3 cm dilated, she is 40% effaced, and vertex is at -1 station. Contractions are palpated and recorded every 5 to 10 minutes. Urinalysis shows no evidence of bacteria. On vaginal examination, membranes are ruptured. Question In addition to the administration of steroids, what is the most appropriate next step? Answer Choices 1 Administration of intravenous antibiotics and induction of labor 2 Assess fetal well-being witha non-stress test and biophysical profile 3 Fetal fibronectin testing 4 Performance of a cervical length ultrasound 5 Treatment with magnesium sulfate for tocolysis

Administration of intravenous antibiotics and induction of labor Explanation This patient has preterm premature rupture of membranes (PPROM) and she is in active labor. PPROM is defined as rupture of fetal membranes prior to 37 weeks' gestation. Given her high temperature and fundal tenderness, her clinical presentation is most consistent with chorioamnionitis. In the setting of ruptured membranes, the diagnosis is more likely. Management: Immediate antibiotics for chorioamnionitis therapy should be started, and the commencement of labor induction or augmentation should be considered once the patient is stabilized. Chorioamnionitis, labor, or non-reassuring fetal heart rate testing mandates delivery at any gestational age. In the absence of labor, chorioamnionitis, or non-reassuring fetal heart rate testing, patients with PPROM can be expectantly managed until 34 to 35 weeks' gestation with corticosteroids and broad-spectrum antibiotics: Corticosteroids: A complete course is indicated from 24 to 34 weeks' gestation. Because the degree of lung development is critical for premature neonates, betamethasone or dexamethasone are given to stimulate fetal lung development before delivery. This reduces the risk of respiratory distress syndrome (RDS) and intracranial hemorrhage (ICH). A recent review of several studies found significant benefits and no increase in risk of infection for mothers or infants given steroids. Current recommendations are that all patients with PPROM before 32 weeks receive steroids. Broad-spectrum antibiotics: A 7-day course is given. Antibiotics prolong the latency period and improve perinatal outcomes in patients with PPROM. Fetal well-being is assessed daily with a non-stress test, and a follow-up biophysical profile as needed is used for ongoing monitoring following pharmacologic intervention. Fetal fibronectin is a glycoprotein that plays a role in fetal membrane adhesion. It can be detected in the cervicovaginal fluid in the late 2nd and early 3rd trimester, and it has been associated with preterm birth. Fetal fibronectin testing may be useful in women with symptoms and negative tests because the negative predictive value is greater than 95%. This may avoid unnecessary treatment. However, the poor positive predictive value creates clinical ambiguity in patients who test positive. It is a diagnostic tool and would not be useful in this case. Tocolysis with magnesium sulfate is only useful for the immediate 48 hours following membrane rupture while achieving steroid therapy. Treatment beyond this period has not found to be effective; in fact, it may be harmful.

When is RhoGAM (D immunoglobulin) given to Rh-negative patients? Answer Choices 1 At the time pregnancy is diagnosed 2 At the beginning of the 2nd trimester 3 If the pregnancy continues beyond the estimated due date 4 After delivery if the infant is found to be Rh negative 5 After delivery if the infant is found to be Rh positive

After delivery if the infant is found to be Rh positive Explanation D immunoglobulin is administered to Rh negative mothers to prevent isoimmunization to the D antigen from an Rh positive fetus. It is recommended that all pregnant patients have a determination of blood type and antibody screening as part of their initial prenatal workup. A negative antibody screen indicates a non-isoimmunized mother who should have a repeat screen at 28 weeks and then have D immunoglobulin administered if her repeat screen remains negative. Once the D antibody is present, the patient is D-sensitized and there is no longer any benefit to the use of D immunoglobulin. Additional times in a pregnancy when the administration of D immunoglobulin to an Rh negative mother would be appropriate include: after spontaneous or induced abortion, after ectopic pregnancy, after any procedure which invades the uterine cavity (amniocentesis, chorionic villus sampling, fetal surgery), and after external cephalic version of a breech infant. Finally, D immunoglobulin should be administered post-partum to a patient whose infant is confirmed to be Rh positive. There is no indication to administer RhoGAM post-partum if the fetus is Rh-negative.

A 29-year-old G3P2 presents to the office for her obstetrical visit. She is currently 16 weeks gestation by ultrasound. Her pregnancy has had no complications to date. She is a non-smoker and takes her prenatal vitamin. What routine obstetric lab should be offered at this visit? A Amniocentesis B Alpha fetal protein screen C 3D ultrasound D D. Glucose challenge E E. HIV testing

Alpha fetal protein screen AFP testing is only available between 15 and 20 weeks gestation. HIV is done on initial visit and amniocentesis is offered for risk factors, advanced age, or abnormal AFP. 3D ultrasound is not routine standard of care, and diabetic screening is done between 24 and 32 weeks gestation.

A 37-year-old female presents to the labor and delivery department complaining of intermittent pain and contractions. Upon arrival, she also complains of vaginal bleeding. She is a G3P2 at 39 weeks gestation; no other prenatal complications are noted. She is a non-smoker. A physical exam reveals the following: P 90, BP 130/80, T 98.7°F, abdomen gravid, positive bowel sounds, and left lower quadrant tenderness noted. A sterile speculum exam reveals the cervix to be dilated 8, fetus is cephalic, and membranes are intact. The fetal monitor reveals heart tones in the 140s with mild, decreased variability and good quality contractions noted. Delivery is felt to be imminent, and vaginal delivery has been determined to be the best course of action. What will likely decrease bleeding and shorten time to delivery? A Increased activity level B Amniotomy C Oxytocin therapy D Epidural placement E IV sedation

Amniotomy If the fetus is mature and vaginal delivery (versus c-section) has been determined to be the best course of action, then amniotomy may diminished amnionic fluid volume. This might also allow for better spiral artery compression, and serve to both decrease bleeding from the implantation site and reduce entry of thromboplastin into the maternal circulation.

You are monitoring a 30-year-old G2P1 at 40 weeks gestation, who is in an active stage of labor and is 6-cm dilated. The fetal heart tracing has a baseline heart rate of 140, with 7 to 10 beats of variability. With the last five contractions you have noted late decelerations. What would be the next most appropriate course of action? A Close observation of FHR tracing B Assessment of dilatation C Augment contractions with oxytocin D Intravenous analgesic E Surgical intervention

Assessment of dilatation B The presence of recurrent late decelerations should raise the suspicion for fetal distress. Vaginal evaluation for change in dilatation or cord prolapse, and to assess the fetal response to stimulation, are the first steps in evaluating the need for intervention.

A 28-year-old woman presents for a well woman examination. She complains about a foul smelling vaginal discharge. She thinks it may be greenish in color. She does have a history of sexually transmitted disease, including chlamydia. She has been treated for it twice. A Pap smear a year ago showed evidence of HPV infection, for which she had colposcopy and cryotherapy. Her last menstrual period was 10 days ago, and it was normal. Past medical history is otherwise significant only for obesity and intermittent low back pain. She takes ibuprofen on a regular basis. She is not allergic to any medications. Family history is significant for breast cancer (grandmother and aunt), diabetes, hypertension, and stroke. Social history reveals that she smokes 1 pack per day and has done so for the past 10 years. She admits to alcohol use and experimentation with marijuana. She is unemployed, but she is attending night school. She lives with her mother and 3 children. On review of systems, she does not have any abdominal pain, dysuria, hematuria, diarrhea, constipation, or fever. She has not noticed any concerning lumps during breast self-examination. She has used oral contraceptives in the past, but wishes to try a different method of birth control because she frequently forgets to take the pill. On physical examination, blood pressure is 140/90 mm Hg, and weight is 245lbs. Pulse is 70 and regular, and respirations are 12. HEENT is normal. Heart and lung examinations are normal. Abdominal examination is difficult due to her body habitus, no abdominal tenderness, and normoactive bowel sounds. Breast examination is normal. External genitalia are normal. On speculum examination, the cervix is parous, and there are no suspicious lesions. A Pap smear and cultures are obtained. Bimanual examination shows there to be no cervical motion tenderness; there is no adnexal mass or tenderness. Question What Pap smear result is potentially the most serious and requires more aggressive investigation and treatment? Answer Choices 1 Normal squamous cells but lack of endocervical cells 2 Colonization with Gardnerella vaginalis and Candida 3 Atypical glandular cells, not otherwise specified 4 Atypical squamous cells of undetermined significance 5 Substantial colonization with Trichomonas vaginalis

Atypical glandular cells, not otherwise specified Explanation Atypical glandular cells, not otherwise specified ( AGC-NOS) indicate a much higher likelihood of more serious underlying disease than atypical squamous cells of undetermined significance (ASCUS). AGC-NOS is found on less than 1% of all Pap smears, but on further study a significant percentage (20 - 50%) of these patients will be found to have high-grade pre-invasive disease, carcinoma in situ, or invasive adenocarcinoma. Colposcopy is recommended for these patients, and depending on the findings, conization biopsy should be strongly considered. ASCUS Paps can simply be repeated once any underlying infection is treated, and does not immediately require colposcopy or more invasive investigation. A lack of endocervical cells is indicative of an inadequate specimen. The Pap can simply be repeated at an interval best determined by underlying risk factors, but there is no need for an immediate colposcopy. Gardnerella and Candida are frequently part of the normal vaginal flora, and they do not need to be treated unless symptomatic. Trichomonas is a sexually transmitted disease and is readily treated with metronidazole. While it is important to treat this infection, the more serious finding would be abnormal cytology. Having this infection does not increase risk of future cervical dysplasia; however, certain strains (16 and 18) of human papillomavirus do increase risk.

You are taking care of a 32-year-old G2P1 at 39 weeks gestation in active labor. Her pregnancy is complicated by gestation diabetes. The fetal head delivered, but the anterior shoulder did not deliver with gentle downward traction. What would be the next most appropriate action? A More forceful traction and fundal pressure B Call for assistance and McRoberts maneuver C Call for assistance and more forceful traction D Call for help and fundal pressure

B Call for assistance and McRoberts maneuver Shoulder dystocia is an obstetrical emergency, and help should always be summoned. The McRoberts maneuver increased the AP diameter, thus accommodating a large head; subrapubic pressure can help dislodge the anterior shoulder, but simple fundal pressure continues to impact it against the pelvic bone.

A 24-year-old female, with a history of type 2 diabetes, presents with the inability to conceive after 14 months of unprotected sexual intercourse with her husband. Her vital signs are unremarkable and you calculate a BMI of 31. Physical examination reveals acne vulgaris and hirsutism. Which of the following treatment options for her infertility would be the most effective considering your suspected diagnosis? A Medroxyprogesterone acetate B Clomiphene citrate C Metformin D Spironolactone E Mini-pill (progestin only)

B Clomiphene citrate Clomiphene citrate is highly effective as the first line treatment for infertility in PCOS. It can be accompanied with metformin, weight loss, exercise, and exogenous gonadotropins when clomiphene fails. PCOS in over half of patients is accompanied with obesity, abnormalities in insulin control, metabolic syndrome, and infertility. Medroxyprogesterone acetate (A) and the mini-pill (E) are used for endometrial protection and with oral contraceptive pills. Metformin (C) will help with her diabetes. Spironolactone (D) is a diuretic, which acts as a weak androgen receptor antagonist.

A 30-year-old woman delivers a viable 7 pound 6 ounce female infant by normal spontaneous vaginal delivery. The infant was delivered and held below the introitus. The infant was dried, stimulated, and apgars were assigned. The cord was then clamped and the infant was placed on the maternal abdomen. Which of the following is a complication that can arise from this? A Hypovolemia B Hyperbilirubinemia C Hypoglycemia D Hyperglycemia E Hypoxia

B Hyperbilirubinemia B In a normal delivery, after the infant is delivered through the introitus it should not be held below it; excessive fluids can be passed to the infant, resulting in increased hematocrit and hemoglobin, which will hemolyze and cause hyperbilirubinemia.

A 24-year-old delivers twins by cesarean section. The twins are monozygotic. The placenta was fused. What does this implies? A Single ovum and single chorion B Single ovum and double chorion C Double ovum and single chorion D Double ovum and double chorion E Triple chorion

B Identical twins indicate single ovum. If the placenta is fused or double it means there are two chorions, and that the trophoblast differentiation occurred before day 3.

A 24-year-old delivers twins by cesarean section. The twins are monozygotic. The placenta was fused. What does this implies? A Single ovum and single chorion B Single ovum and double chorion C Double ovum and single chorion D Double ovum and double chorion E Triple chorion

B Single ovum and double chorion B Identical twins indicate single ovum. If the placenta is fused or double it means there are two chorions, and that the trophoblast differentiation occurred before day 3.

Which bone is the most susceptible and most often fractured at birth? A Calcaneus B Clavicle C Femur D Humerus E Patella

B The clavicle is the most common bone broken during childbirth. It often is associated with shoulder dystocia, but clavicular fractures can occur in uncomplicated pregnancies. They are usually of the greenstick variety and heal without complications. Calcaneal and patellar fractures are highly unlikely to occur since they are not long bones which are much more vulnerable to fracture. Fractures of the humerus and femur are possible during childbirth, but generally only in traumatic births. Humerus and femur fractures are much less common than clavicular fractures

A 27-year-old female presents to the office complaining of a lump on one side of her vagina. It has been present for several weeks, and causes only slight discomfort with intercourse. She has no new sexual partners and no other vaginal or systemic symptoms. On physical exam you note a 1.5-cm area of swelling in the left posterior labia majora. It is firm, well circumscribed, and minimally tender, and no changes are noted in the skin. No vaginal discharge or other lesions are noted. What is the most likely diagnosis? A Vulvar abscess B Bartholin gland duct cyst C Vulvar lipoma D Acrochordon E Lichen planus

Bartholin gland duct cyst B Bartholin obstruction leads to an asymmetrical posterior labia majora or vestibule, which is not typically painful unless it abscesses. Lipomas are less likely, and lichen planus causes pain and itching without a discrete mass.

A 20-year-old woman presents to the medical office to start birth control pills. She is engaged to be married soon. She has never been sexually active. She states she feels healthy, and she denies any current complaints. A summary of her past medical history includes: Medications: None Allergies: Sulfa medications Surgical history: None Medical history: No known conditions. OB/GYN history: Menarche age 13. Regular monthly menses, with no menstrual complaints. Family history: Paternal grandfather has diabetes and hypertension. Maternal grandmother had a stroke. Maternal grandfather had prostate cancer. Father has hypertension and history of a heart attack. Social history: Patient works as a retail clerk part-time and is attending college. She currently lives with her parents. She denies the use of tobacco, alcohol, and recreational drugs. Question What component of the physical exam is the most important before prescribing birth control to this patient? Answer Choices 1 Bimanual pelvic exam 2 Blood pressure measurement 3 Breast exam 4 Cardiovascular exam 5 Pap smear with vaginal speculum exam

Blood pressure measurement Explanation It is most useful and important to obtain a blood pressure measurement prior to initiation of combined hormonal contraceptives. Routine screening, including physical exam and tests, is generally not recommended prior to initiating contraceptives. If the patient is found to have severe hypertension, combination hormonal contraceptives should be avoided. Likewise, if blood pressure measurements rise dramatically after using these contraceptives, they should be discontinued and alternate methods initiated. A bimanual pelvic exam has been somewhat controversial, with no compelling evidence to continue the practice in asymptomatic women. In fact, exams viewed as invasive or embarrassing (such as pelvic and breast exams) can be seen as a barrier for preventing unintended pregnancies; some women may avoid encounters to obtain contraceptives in order to avoid the exams. Breast exam is another routinely performed exam; it has limitations in the asymptomatic patient population for screening. It becomes much more useful in screening for breast cancer in older women, who are higher risk for breast cancer. A breast exam is not routinely recommended prior to initiating birth control. A cardiovascular exam is not recommended prior to initiating birth control. Even with some cardiovascular disease in this patient's family history, the cardiovascular exam is likely be normal and would not be useful in initiating contraceptives. The Pap smear (short for Papanicolaou) with vaginal speculum exam is primarily a test for cervical cancer and pre-cancerous dysplasia. Many years ago, it was thought that birth control increased risk of cervical cancer. That myth has been disproved. There is no compelling reason to obtain a Pap smear in order to initiate birth control. Routine cervical cancer screening guidelines should be followed, which would indicate that this patient should wait until the age of 21 years before beginning Pap testing. If the patient noted vaginal complaints or desired testing for sexually transmitted diseases, a speculum exam would be useful.

The patient is a 26-year-old woman who presents to her gynecologist's office with a 4-month history of the chief complaint of amenorrhea. She has had some breast tenderness, but denies nausea, vomiting, fatigue, and abdominal pain. She was sexually active until about 6 weeks ago when she broke up with her boyfriend, but states that they used condoms. Question Which of the following choices would best describe the appearance of her cervix during an internal examination if the patient is 10 weeks pregnant? Answer Choices 1 purulent discharge from cervical os 2 strawberry-like appearance to cervix 3 bluish appearance to cervix 4 cervical os slightly open 5 blood from cervical os

Bluish appearance to the cervix Explanation Bluish appearance to the cervix is the correct answer. While some pregnant women can have a normal appearance to their cervix, most will have a bluish appearance. In addition, bimanual palpation will reveal the cervix to be soft and flexible. The uterus also increases in size with gestational age and will fill the pelvis around 12 weeks. Strawberry-like appearance to the cervix is not the correct answer, as this appearance is more likely to be seen in patients who have some form of cervicitis. In particular, patients with cervicitis caused by trichomonas usually have purulent vaginal discharge and a friable cervix with punctate hemorrhages that give it a "strawberry-like" appearance. Cervical os slightly open is not the correct answer, as it is not the most likely appearance of the cervix, but could actually be seen in the cervix of a pregnant female. However, the os is usually only slightly open if the patient is experiencing a spontaneous abortion, has an incompetent cervix, or is in the early stages of labor. Purulent discharge from the cervical os is not the correct answer. If purulent discharge is seen coming from the cervical os, the likely diagnosis has something to do with an infection. The patient may have cervicitis or pelvic inflammatory disease. Blood from cervical os is an incorrect answer, as it is not typically indicative of a pregnant patient. Blood coming from the cervical os is more likely to be seen in a patient currently having their menses, a patient experiencing a spontaneous abortion, cervical malignancy, and during some stages of labor. If a female is known to be pregnant and has blood coming from the os, there are likely to be other symptoms associated with various disorders such as placental abnormalities.

A 22-year-old woman presents with increased vaginal discharge. She is sexually active with 2 male partners, and she uses birth control pill for contraception. Her last menstrual period was 12 days ago, and she has noticed an increased whitish vaginal discharge for the past week. Physical exam reveals a soft, non-tender abdomen. On pelvic exam, she has a light-yellow cervical discharge with erythema of the cervical os. There is no cervical motion tenderness and no adnexal masses or tenderness. Wet mount of the vaginal discharge reveals epithelial cells and WBCs, and no yeast or protozoa. Gram stain of the vaginal discharge reveals many leukocytes. A urine pregnancy test is negative. What is the best treatment for this patient? Answer Choices 1 Azithromycin 1 g once PO, and doxycycline 100 mg twice daily, for 14 days 2 Ciprofloxin 500 mg PO, and ceftriaxone 125 mg IM once 3 By mouth, azithromycin 1 g once, and Ciprofloxin 500 mg once 4 By mouth, azithromycin 1 g PO, and ceftriaxone 250 mg IM once 5 By mouth, metronidazole 2 g once

By mouth, azithromycin 1 g PO, and ceftriaxone 250 mg IM once Explanation This patient has a mucopurulent cervicitis on physical exam, which is usually caused by Neisseria gonorrhoeae or Chlamydia trachomatis. In all practicality, a patient should be treated with antimicrobial agents that will cover both pathogens because there is a high risk of co-infection with both pathogens (at least 50%). Patients should be tested for infection with both chlamydia and gonorrhea before treatment, usually with an immunoassay. In addition, a Gram stain of the cervical discharge can help elucidate the responsible pathogen. In 60% of women with gonorrhea, Gram-negative intracellular diplococci may be identified. In chlamydial infections, the Gram stain will reveal many leukocytes (>10/high powered field) but no gonococci. These patients should also be screened for co-infection with syphilis and possibly HIV. This patient does not have signs of salpingitis or tubo-ovarian abscess (pelvic inflammatory disease). Specifically, she has no cervical motion tenderness or peritoneal signs, and no adnexal tenderness or masses. She will qualify for outpatient treatment of cervicitis and can avoid the more aggressive treatment used for salpingitis. In either condition, both of the patient's sexual partners will require medical evaluation and treatment. Additionally, if either Neisseria gonorrhoeae or Chlamydia trachomatis is identified with culture or immunoassays, infection with these pathogens must be reported to the local health department. Acceptable regimens for the treatment of gonorrheal cervicitis include one-time treatment with ceftriaxone 250 mg IM or cefixime 400 mg PO. Alternative regimens for gonorrheal cervicitis include ceftizoxime 500 mg IM; cefoxitin 2 g IM with oral probenecid 1 g; or cefotaxime 500 mg IM. Spectinomycin 2 mg IM is the recommended treatment for patients with penicillin or cephalosporin allergy; however, spectinomycin is not currently available in the United States. Previously, quinolones were an option for gonorrheal cervicitis, but these regimens are no longer recommended by the CDC due to increasing resistance to the quinolones in Asia and many metropolitan areas of the U.S. Additionally, all patients with cervicitis should receive a regimen to treat chlamydia. These regimens include azithromycin 1 g PO once or doxycycline 100 mg PO twice a day for 7 days. Metronidazole 2 g PO is the regimen used to treat vaginitis due to Trichomonas, not mucopurulent cervicitis.

A 25-year-old woman and her husband have been using condoms and spermicidal foam for the 8 months since the birth of their baby. She plans to wean the baby from the breast sometime between a year and 18 months of age, but would like to begin a "less messy" method of contraception. Prior to her pregnancy she took combination oral contraceptives for several years without any difficulties. Which of the following is an appropriate recommendation for this woman? A "Breast feeding alone will prevent pregnancy until the baby is weaned." B "Condoms and spermicide are your only option until you wean the baby." C "Progestin-only pills are recommended for women who are breast feeding." D "A vaginal ring will not affect your milk production." E "You can start back on the same oral contraceptive you took before."

C "Progestin-only pills are recommended for women who are breast feeding." C The progestin-only pill is ideal for breast feeding mothers because this pill does not interfere with lactation the way combination pills do. Breast feeding (A) is reasonably effective in preventing pregnancy only as long as breast milk is the infant's only source of nutrition. Condoms and spermicide (B) are options, but not the only ones for breastfeeding couples. A vaginal ring (D) does decrease the amount of milk production somewhat, but may be an effective option if lactation is well established. Combination oral contraceptives (E) are not recommended for breastfeeding women.

You are providing care to a woman who is at 33 weeks gestation. Her pregnancy is complicated by gestational diabetes. She is being provided education by the dietician, and has weekly obstetrical appointments. What fasting blood sugar (FBS) readings should necessitate switching from diet control to insulin therapy? A FBS > 70 mg/dL B FBS > 90 mg/dL C FBS > 95 mg/dL D FBS > 110 mg/dL E FBS > 126 mg/dL

C FBS > 95 mg/dL Gestational diabetes has different risks associated with it, in particular for the fetus. Stricter glycemic controls are recommended for pregnant versus non pregnant women by the ACOG and the ADA.

A 26-year-old female presents to clinic complaining of increasing headaches for one year, irritability, bloating and fluid retention, and abdominal discomfort with loose stools during her menstrual cycle. The symptoms begin a day or two before her menses, and last until the middle of her cycle. She has tried acetaminophen and ibuprofen without improvement. On physical exam she is a well-developed, well-nourished female in no acute distress. Vitals are normal, CV and lungs are normal, pelvis exam is normal, pap smear is normal, and GC and Chlamydia testing are negative. What would be the most appropriate next step? A Pelvic ultrasound B FSH, LH levels C Fluoxetine on cycle day 21-7 D Paroxetine daily E Serum HCG

C Fluoxetine on cycle day 21-7 (Prozac) C Hx and Px are key to diagnosing premenstrual syndrome. Laboratory and radiologic procedures are not useful, particularly in light of a normal exam. With failure of NSAIDS, treatment is aimed at reducing symptoms. For mild to moderate symptoms, SSRI therapy prior to and through the menstrual cycle has become a primary therapy.

A 27-year-old woman and her male partner come to the emergency department for assistance with emergency contraception. They experienced condom failure during intercourse an hour ago and neither desires pregnancy. Her last menstrual period was approximately two weeks ago and her cycles occur every 28-30 days. Her medical history includes a deep venous thrombosis during labor and delivery 5 years ago. What is the most appropriate course of action at this time? A Administration of an ethinyl estradiol and levonorgestrel combination now and in 12 hours B Dilation and curettage C Insertion of a copper-containing intrauterine device D Serial beta hCG determinations E Testing for factor V Leiden

C Insertion of a copper-containing intrauterine device C Insertion of a copper-containing IUD is an effective means of preventing an unintended pregnancy in this case. The woman's history of clotting is a contraindication to use of combination oral contraceptives (A). Dilation and curettage (B) is neither appropriate nor effective for emergency contraception. Performing beta hCG determinations (D) would merely detect pregnancy if it were to occur. Given her history of clotting, testing for factor V Leiden (E) may be appropriate but will not affect management at this time.

A 30-year-old woman and her husband have been trying unsuccessfully to become pregnant for the past year. Over-the-counter ovulation tests have indicated that she is ovulating. As part of her evaluation she undergoes a hysterosalpingogram that reveals tubal scarring. Which of the following is the most likely explanation for this finding? A Congenital anomaly of the tubes B Diethylstilbestrol exposure C Past asymptomatic chlamydial infection D Previous abdominal surgery E Scarring from prior uterine instrumentation

C Past asymptomatic chlamydial infection C Chlamydial infection can cause "silent" pelvic inflammatory disease, leading to scarring with subsequent tubal obstruction that can cause infertility or ectopic pregnancy. Congenital anomalies of the tubes (A) may also contribute to infertility, but is not associated with scarring. DES exposure (B) in utero may lead to reproductive-system anomalies in offspring. Past abdominal surgery (D) may lead to adhesions that inhibit fertility. Uterine instrumentation (E) may lead to intrauterine synechiae.

A 30-year-old female G2 P2, who delivered via normal spontaneous vaginal delivery, presents complaining of increasing vaginal pressure, low back pain, and stress incontinence. What is the mostly likely cause of her condition? A Damage to the levator muscles B Increased intra abdominal pressure C Widening of the levator gap D Widening of the AP pelvis diameter E Endopelvic fascia remodeling and cervical elongation

C Widening of the levator gap Damage to the levator and increased abdominal pressure are known risk factors, but the widening of the gap with the associated risk factors is what allows the defect to occur. Fascia remodeling can occur instead of a prolapse defect.

A 60-year-old postmenopausal woman who has had negative annual Papanicolaou smears of the cervix for many decades reports that her husband has been in a nursing home for almost a year and that she is no longer sexually active. He was her only sexual partner. She has no personal or family history of cancer and has never taken or been exposed to any kind of hormone. She asks if she really needs to continue having testing every year. What is the correct advice for her? A At your age, you should really have a Pap smear every six months. B You should continue to have annual Pap smears. C You can safely extend the time between Pap smears to three years. D We can discontinue Pap smears altogether if you first have a negative colposcopy. E People who are not sexually active do not need Pap smears at all.

C You can safely extend the time between Pap smears to three years. C Women between the ages of 30 and 64 who have had 3 consecutive negative Pap smears and no additional risk factors may reduce the frequency of their screenings to every 2-3 years. Women who are HIV positive should have cervical screening every six months (A). Women with risk factors such as multiple partners, history of sexually transmitted infections or in utero exposure to diethylstilbestrol, immunosuppressive therapy should be screened annually (B). A colposcopy is not needed given her history (D). People who are not sexually active (E) may need to continual annual screenings if other risk factors exist.

A 32-year-old nulligravida woman presents with gradual onset of excessive hair growth on the face, chest, abdomen, back, and upper parts of her limbs, together with the irregular menstruation. She denies changes in weight, body shape, and voice. Her menarche started when she was 8. Her family history is non-contributing. Physical examination reveals short stature, excessive male-pattern hair growth, and slight baldness. Her BMI is 18.5. The rest of the examination, including pelvic ultrasound, is normal. Question What is the most likely diagnosis? Answer Choices 1 Adrenal tumor 2 Ovarian tumor 3 Congenital adrenal hyperplasia 4 Polycystic ovarian syndrome 5 Idiopathic

CAH Explanation This patient has male-pattern growth of body hair. It usually occurs in androgen-stimulated parts of the body (the face, chest, areolae). In women, androgens are produced in ovaries, adrenal glands, and the hair follicle. Precocious puberty, gradual onset of the hirsutism without virilization in second or third decade, and irregular menstruation in a woman with short stature will lead you to consider late onset congenital adrenal hyperplasia. Congenital adrenal hyperplasia (CAH) is one of the most common autosomal recessive disorders. It is proven by the presence of elevated first morning 17-hydrohyprogesterone because of the deficiency of the enzyme 21- hydroxylase. Absolute or partial deficiency of the enzyme can manifest early or late in life. There are early forms in female newborn with masculinization and ambiguous genitalia (the shunting of 17-hydroxyprogesterone to the production of androgens) or in some salt-wasting cases (the deficiency in production of cortisol and aldosterone). The combination is also possible. Late onset cases are most commonly mild. They usually manifest as some type of androgen excess later in life and without the aldosterone deficiency. An adrenal tumor will present with rapid onset of virilization, which is not the case in this patient. Elevated dehydroepiandrosten sulfate (DHEAS) produced by adrenal glands will confirm the diagnosis. Ovarian tumor will also present with rapid onset of virilization, probably with palpable adnexal mass. Elevated testosterone will confirm your diagnosis. Polycystic ovarian syndrome (PCOS) is the most common endocrine disorder of reproductive-age women. It typically presents with irregular vaginal bleeding, obesity, infertility, and hirsutism, and is characterized by ovarian dysfunction, disordered gonadotropin secretion, and hyperandrogenemia. Anovulatory cycles, infertility, amenorrhea, hirsutism, acne, and acanthosis nigrans are sometimes the result. Ultrasound may reveal hyperplastic endometrium because of the absence of ovulation and progesterone cyclic shedding (this may lead to the endometrial cancer) and characteristic bilateraly enlarged ovaries with thick capsules, multiple subcapsullar follicles, and hyperplasia of ovarial stroma. The levels of testosterone might be mildly elevated, but DHEAS and 17-hydrixyprogesterone are normal. Confirmation test consists of LH:FSH ratio and decreased level of sex-hormone binding protein. BMI that puts a patient in the underweight group and short stature are not parts of the clinical picture. In most cases, hirsutism is a benign, primarily cosmetic condition. When accompanied by masculinizing signs or symptoms, particularly when these arise after puberty, hirsutism may be a manifestation of a more serious underlying disorder such as an ovarian or adrenal neoplasm. Therefore, patients with clinical evidence of hyperandrogenemia, including hirsutism and acne, should have an evaluation of adrenal androgens. Before you declare the diagnosis of idiopathic hirsutism, you should exclude those disorders.

Which of the following tests should an HIV-positive pregnant woman undergo in each trimester of pregnancy? A CD4+ lymphocyte count B cytomegalovirus serology C postpartum depression with controls D shielded chest radiography E venereal disease research laboratory (VDRL)

CD4+ count lymphocytic count A HIV-positive pregnant women should undergo CD4+ serology each trimester. Early in the pregnancy, they should undergo shielded chest radiography, CMV baseline testing, and tuberculosis testing with controls. Syphilis testing should be completed initially and as usually recommended later in pregnancy.

A 25-year-old woman presents with pelvic pain for the last 3 - 4 menstrual cycles, but the most recent episode with increasing symptoms for approximately 2 weeks. She has tried over-the-counter pain relievers without relief. She admits deep dyspareunia and has been with her current sexual partner for about 6 months. She has no chronic medical conditions and no allergies to any medications. She denies urinary issues. She admits to nausea, but no vomiting or diarrhea. She reports a negative home pregnancy test this morning. Her last pelvic exam was over 1 year ago. LMP was 1 week ago. Question What physical exam findings would best narrow your diagnosis? Answer Choices 1 Abdominal guarding 2 Adnexa approximately 2.5 cm in size 3 Anteverted uterus 4 Cervical motion tenderness 5 Discomfort with speculum insertion

CMT Explanation Cervical motion tenderness, along with a history suspicious for pelvic inflammatory disease (PID), is a red flag for the diagnosis. While testing for sexually transmitted infections (especially Chlamydia trachomatis and Neisseria gonorrhea) are indicated, the diagnosis is typically based upon history and physical exam. The Centers for Disease Control Sexually Transmitted Treatment Guidelines recommend empiric treatment for PID if one or more of the three following minimum criteria are present on exam: cervical motion tenderness, uterine tenderness, and/or adnexal tenderness. A high index of suspicion is needed, and treatment should be initiated while awaiting laboratory results. The examiner should further elicit this woman's sexual partner history and potential vaginal/cervical discharge. Other conditions, such as endometritis and endometriosis, could cause cervical motion tenderness. The CDC guidelines are helpful in selecting antibiotic therapy and whether to treat as inpatient or outpatient. Most women will find that pelvic exam is uncomfortable, but tenderness with movement of the cervix is not a normal finding and indicates inflammation in the uterus.The remainder of the physical exam findings listed are less helpful than a finding of cervical motion tenderness. Abdominal guarding is a common physical exam finding with many gastrointestinal, urinary, and pelvic disorders. Guarding refers to voluntary or involuntary contraction of the abdominal musculature in response to pain felt during palpation. It is not a specific finding for pelvic inflammatory disease and would not help narrow a diagnosis as much as cervical motion tenderness. A physical exam finding of an adnexa approximately 2.5 cm in size is a normal physical exam finding. When the patient is thin enough for a good estimate of ovarian size, an examiner may notice enlargement in the case of ovarian cysts, ectopic pregnancy, tumors, and other ovarian disorders. A finding of normal ovarian size does not narrow the differential for this patient. An anteverted uterus on physical exam simply indicates the angle of the fundus points toward the ventral side of the patient. This is also a normal finding and does not assist in arrival at a definitive diagnosis. Discomfort with speculum insertion can be related to many factors, including examiner skill, patient anxiety, vaginal inflammation, and many gynecologic disorders. This finding is nonspecific and also does not help with diagnosis.

A young pregnant woman comes to her physician for a routine check-up. She tells her physician that she would like to test for any fetal abnormalities. After you discuss with her the tests available, she agrees on the procedure that allows for the earliest prenatal diagnosis in pregnancy. Which of the following test is the most likely? Answer Choices 1 Amniocentesis 2 Fetal Biopsy 3 Chorionic villus samplings 4 Umbilical blood sampling 5 Ultrasound

CVS Explanation Chorionic villus sampling can be performed as early as the eighth week of pregnancy under the guidance of an ultrasound. Chromosomal and biochemical abnormalities can be detected. This procedure has potential complications such as bleeding from the biopsy site, compromise of fetal membranes, and infection. Amniocentesis may be performed from 12-16 weeks of gestation. Penetration of the placenta is an unwanted complication. Amniocentesis should be considered if the woman is over 35-years-old, or if there is a previous child with a chromosomal anomaly.

A 26-year-old woman at 32 weeks gestation is brought via ambulance to the ER with a 30-minute history of convulsions; they occurred while she was at work. Her vital signs include a blood pressure of 160/95 mm Hg, heart rate of 84 beats/min, and respiratory rate of 22 breaths/min. On physical examination, she is noted to have lower extremity edema and is hyperreflexic. She was treated with intravenous magnesium sulfate at a rate of 2 g/hour for 12 hours, and she is now is hyporeflexic; her respirations are decreased. What would you prescribe this patient? Answer Choices 1 Diazepam 2 Oxygen 3 Continue magnesium 4 Calcium 5 Potassium

Calcium Explanation The clinical picture is suggestive of eclampsia. In this instance, she became hyporeflexic and her respirations decreased, indicating possible magnesium toxicity. This can be reversed by giving calcium gluconate. Diazepam does not reverse the effects of magnesium toxicity. Oxygen will also not reverse the effects of magnesium toxicity. Continuing the magnesium would worsen the condition. Potassium does not reverse the effects of magnesium toxicity.

A 52-year-old woman presents with vaginal discharge that is white curd-like in appearance but is not malodorous. She has a 19-year history of obesity and poorly controlled type 2 diabetes mellitus. Microscopic examination of the discharge with 10% potassium hydroxide demonstrates filaments and spores. Which of the following is the most likely etiologic agent? A Candida B Gardnerella C Lactobacillus D Staphylococcus epidermidis E Trichomonas vaginalis

Candida A This case has several clues pointing to a Candida infection, including the fact that diabetes mellitus can predispose patients to Candida infections and the presence of the white curd-like discharge that is not malodorous. In Trichomonas vaginalis, the discharge is malodorous and yellow-green in color. With Gardnerella, there is also a malodorous discharge. Lactobacillus is the predominant, normal microorganism of the vagina and keeps it slightly acidic to help reduce the growth of potentially harmful organisms. Staphylococcus epidermidis is also part of the natural flora of the vagina

While doing routine newborn exams in the nursery on full term infants, you note a 4 - 5 cm soft, raised, round, nontender, bruised swelling overriding the left occipitoparietal suture on a 30-hour-old male infant. He was born by spontaneous vaginal delivery with rupture of membranes at home. He has been feeding well. On the rest of his exam, his red reflex is present bilaterally, lungs, heart and abdominal exams seem normal. Rest of his skin is pink, warm and dry, without cyanosis. What is the most likely diagnosis? Answer Choices 1 Cephalohematoma 2 Hydrocephalus 3 Caput succedaneum 4 Plagiocephaly 5 Craniosynostosis

Caput succedaneum Explanation Caput succedaneum represents an accumulation of serosanguineous subcutaneous fluid. This fluid is thus not bound by suture lines, but may have poorly defined margins and extend across midline and suture lines. It is caused by mechanical trauma of the presenting portion of scalp pushing through a narrowed cervix. It is more likely to be seen after a prolonged or difficult delivery especially after prolonged rupture of membranes. This fluid collection will resorb within a few days, thus no treatment is necessary. Jaundice might result with very large caputs. Cephalohematoma is due to subperiosteal hemorrhage involving the outer table of a cranial bone. Unlike the caput, the swelling does not extend across a suture line. This may result from an instrumented delivery causing rupture of blood vessels between the skull and periosteum. Healing by resorption can take several weeks to months with some peripheral calcification. Large cephalohematomas can contribute to hyperbilirubinemia. Hydrocephalus refers to the presence of excess cerebrospinal fluid (CSF) within the ventricles and is caused by either excessive production of CSF or blockage of CSF circulation. Clinically, this presents with an increasing head circumference, full fontanelles, irritability, poor feeding and vomiting. There may be sunset eye sign with the eyes deviated downward revealing the upper sclera. Plagiocephaly is an asymmetry of the cranial vault caused by constant pressure from positioning on the dependent side. It disappears as the baby becomes more active and spends less time in one position. Craniosynostosis is the premature closure of one or more sutures altering the shape of the head. This can be differentiated into primary craniosynostosis, which has no underlying brain abnormality or metabolic defect, but may be related to intrauterine constraints and secondary craniosynostosis that may be due to a structural cerebral abnormality or related to other conditions such as hyperthyroidism, severe anemia, or a metabolic condition. Some genetic abnormalities can also cause craniosynostosis.

A 52-year-old woman present to her gynecologist's office with a 6-month history of hot flashes, night sweats, mood swings, and vaginal dryness that interferes with intercourse. The symptoms seem to be worsening and are now interfering with her productivity at work and with her relationships with family and friends. Her LNMP was 8 months ago; the patient notes: "after that period, they just stopped." She denies any history of tobacco use, and she drinks one glass of red wine daily. She has a history of coronary heart disease (CHD), with stent placement 2 years ago. She recently read an article about hormone replacement and would like you to prescribe this for her. Question What is true regarding hormone replacement for this patient? Answer Choices 1 She is not a candidate for hormone replacement because of her coronary artery disease. 2 Estrogen replacement alone would be recommended due to the increased risk of endometrial cancer with progesterone administration. 3 Hormone replacement will only control her hot flashes and night sweats, but will not affect her vaginal dryness. 4 Hormone replacement is contraindicated for her because of her chronic alcohol use. 5 Certain antidepressant could be an alternate therapy for treatment of her menopause related mood swings.

Certain antidepressant could be an alternate therapy for treatment of her menopause related mood swings. Explanation This patient is presenting with classic symptoms of natural menopause and could be a candidate for hormone replacement therapy based on the history information that is provided. CHD is not a contraindication for HRT. Some studies have shown a decrease in CHD in postmenopausal women on HRT. Estrogen therapy alone is contraindicated in this patient because her uterus is intact and estrogen that is unopposed by progesterone increases the risk of endometrial cancer. Hormone replacement will in fact diminish vasomotor symptoms, as well as vaginal dryness. Modest alcohol use is not a contraindication for hormone replacement. SSRIs would be an alternative therapy option for the treatment of this patient's mood swings. However, SSRIs would not result in relief of vasomotor symptoms or vaginal dryness.

You are caring for a 29-year-old G3P2 at 39 weeks gestation, who has been laboring for 6 hours. She is a diet-controlled diabetic. Her last child was 9 pounds 8 ounces. She has been completely dilated for 2 hours, and the fetal head is at a plus 2 station, which is unchanged. What is the next most appropriate course of action? A Begin oxytocin B Vacuum extraction C High forcep extraction D High forcep rotation E Cesarean section

Cesarian Section The patient is a diabetic with a history of a macrosomic infant; the likelihood of macrosomia in this infant is significant. Instrument delivery is not recommended if macrosomia is suspected. By definition, she has had an arrest of descent of the fetal head and one should be highly suspicious for macrosomia, in which case a c-section is the preferred method of delivery.

A 25-year-old primigravida presents at 8 weeks gestational age for her first prenatal visit. She denies any abnormal vaginal discharge or pelvic pain. On pelvic exam, you note cyanosis of the proximal vagina and cervix. Question What is this finding consistent with? Answer Choices 1 Acute cervicitis 2 Chronic cervicitis 3 Hegar's sign 4 Chadwick's sign 5 Cervical ectopic pregnancy

Chadwick's sign

A 40-year-old female is status post a dilatation and curettage for hydatidiform mole. On week 3 post surgery, her follow-up quantitative hCG level has elevated slightly. What is the most likely diagnosis? A Adenocarcinoma of the ovary B Adenocarcinoma of the uterus C Retained hydatidiform mole D Choriocarcinoma E Corpus luteum cys

Choriocarcinoma 5% of hydatidiform mole progress to choriocarcinoma; the longer the mole in intrauterine the higher the risk. Pre-surgical evaluation for mole removal includes a chest x-ray to rule out distant metastasis. hCg that either plateuas or elevates is choriocarcinoma until proven otherwise, and requires prompt evaluation.

our patient, a 48-year-old woman, presents with vaginal bleeding and states that she is "alarmed" because she is quite sure she is 2 months pregnant. History includes unremarkable live birth of a male child 7 years ago and a molar pregnancy a year ago. Examination reveals a uterus that is inappropriately large for gestational length and hCG levels are higher than expected. Fetal parts and heart sounds are not present. Your diagnosis is that of carcinoma, but you are able to reassure your patient that this neoplasm is of the type that is most sensitive to chemotherapy. What is your diagnosis? Answer Choices 1 Endometrioid carcinoma 2 Ovarian dysgerminoma 3 Choriocarcinoma 4 Serous cystadenocarcinoma 5 Ovarian teratoma

Choriocarcinoma Explanation Gestational Trophoblastic Diseases are a group of related diseases forming a spectrum from Benign Hydatidiform Mole, to Invasive Mole, to Placental-Site Trophoblastoma, and finally to Choriocarcinoma. Treatment of women who have nonmetastatic gestational trophoblastic disease is almost 100% successful, and allows reproductive function to be preserved. The cure rate for metastatic disease is approximately 90 percent. Serous cystadenocarcinoma occurs in the ovary and is a cystic or semi-cystic neoplasm. It usually occurs bilaterally An ovarian dysgerminoma is a malignant ovarian neoplasm, hypothesized to be derived from primordial germ cells of the sexually undifferentiated embryonic gonad. Endometrioid carcinoma resembles the typical carcinoma of the endometrium, as its name suggests, but is an ovarian carcinoma. This neoplasm may be seen with endometrial carcinoma. Ovarian teratoma is composed of tissues that are derived from three germinal layers, the endoderm, mesoderm, and ectoderm. Many teratomas will contain hair or teeth. These true neoplasms will usually present with lower quadrant pain.

A 40-year-old female is status post a dilatation and curettage for hydatidiform mole. On week 3 post surgery, her follow-up quantitative hCG level has elevated slightly. What is the most likely diagnosis? A Adenocarcinoma of the ovary B Adenocarcinoma of the uterus C Retained hydatidiform mole D Choriocarcinoma E Corpus luteum cyst

Choriocarcinoma 5% of hydatidiform mole progress to choriocarcinoma; the longer the mole in intrauterine the higher the risk. Pre-surgical evaluation for mole removal includes a chest x-ray to rule out distant metastasis. hCg that either plateuas or elevates is choriocarcinoma until proven otherwise, and requires prompt evaluation.

A 24-year-old female, with a history of type 2 diabetes, presents with the inability to conceive after 14 months of unprotected sexual intercourse with her husband. Her vital signs are unremarkable and you calculate a BMI of 31. Physical examination reveals acne vulgaris and hirsutism. Which of the following treatment options for her infertility would be the most effective considering your suspected diagnosis? A Medroxyprogesterone acetate B Clomiphene citrate C Metformin D Spironolactone E Mini-pill (progestin only)

Clomiphene citrate B Clomiphene citrate is highly effective as the first line treatment for infertility in PCOS. It can be accompanied with metformin, weight loss, exercise, and exogenous gonadotropins when clomiphene fails. PCOS in over half of patients is accompanied with obesity, abnormalities in insulin control, metabolic syndrome, and infertility. Medroxyprogesterone acetate (A) and the mini-pill (E) are used for endometrial protection and with oral contraceptive pills. Metformin (C) will help with her diabetes. Spironolactone (D) is a diuretic, which acts as a weak androgen receptor antagonist.

A 29-year-old woman comes in for evaluation of an increased vaginal discharge for the past week. She describes it as "sort of whitish gray with a disgusting odor." She has no other symptoms. She has had no new sexual partners, has taken no antibiotics, and has not used any new hygiene products. Examination reveals no vulvar erythema. She does have an adherent whitish discharge in the vaginal vault, but no vaginal erythema. Whiff test of the secretions is positive. Microscopic examination of saline and potassium hydroxide preparations is most likely to reveal which of the following? A Clue cells B Increased polymorphonucleocytes C Motile flagellates D Small, rounded parabasal epithelial cells E Spores and filaments

Clue cells A Examination of vaginal secretions in a woman with bacterial vaginosis demonstrates the presence of clue cells, which are epithelial cells that appear granulated due to G vaginalis cells adhering to them. Increased PMNs (B) may be present in a number of vaginal conditions, but not typically in bacterial vaginosis. Motile flagellates (C) are present in vaginal trichomoniasis, while small parabasal epithelial cells (D) are present in atrophic vaginitis. Spores and filaments (E) (pseudohyphae) are typical of candidiasis.

A 36-year-old gravida 5, para 3013 at 34 weeks gestational age presents to the Triage Unit of Labor and Delivery with a 2-hour history of painless bright red bleeding per vagina. She states that after urinating, she found the toilet "filled with blood" when she stood up. There is normal fetal movement. She had 1 Caesarean section for uncertain fetal status 2 years ago. Her other 2 term pregnancies resulted in normal vaginal deliveries. She denies recent intercourse, prior history of vaginal bleeding during this pregnancy, syncope, dizziness, and headache. Vital signs: BP is 100/72 mm Hg, and maternal pulse is 110 bpm; respirations are 18/min. The patient is afebrile. Fetal heart rate is160 bpm, absent variability, absent accelerations, and occasional variable decelerations; therefore it is a category II fetal heart rate tracing. Occasional irregular contractions are seen on tocodynamometer. Question What management step should be taken immediately? Answer Choices 1 Caesarean section 2 Complete blood count, type and hold, administration of IV fluids 3 Biophysical profile 4 Amniocentesis for lecithin and sphingomyelin 5 Induction of labor; anticipate normal vaginal delivery

Complete blood count, type and hold, administration of IV fluids Explanation The most likely diagnosis for this patient with 3rd trimester bleeding is placenta praevia. Obstetrical hemorrhage is still one of the most common etiologies of maternal death in the United States. In a patient with significant blood loss, attention must first be paid to determination of the degree of blood loss, volume expansion with isotonic IV fluids, and access to blood replacement if necessary. Caesarean section is not indicated at this time, as maternal complete blood count is not yet available, and as at this point the fetal heart rate tracing is Category 2. A biophysical profile may be of benefit in determining fetal status, but is not the most critical next step. Sonography to confirm placenta praevia is more critical than the biophysical profile at this time. Amniocentesis for lecithin and sphingomyelin may be of benefit in determining whether the fetus has achieved lung maturity, but it is not the next best step in managing this emergent situation. Induction of labor and normal vaginal delivery are contraindicated in placenta praevia.

A 20-year-old woman who was 3 months pregnant aborted a mass which resembled a bunch of grapes. The specimen that she passed is shown in Figure G3.5. Karyotype analysis of the specimen revealed that it was 46, XX. The MOST likely diagnosis is which of the following? Answer Choices 1 Incomplete abortion, normal placenta 2 Choriocarcinoma 3 Ectopic pregnancy 4 Partial (incomplete) hydatidiform mole 5 Complete hydatidiform mole

Complete hydatidiform mole Explanation A hydatidiform mole occurs once in about 2000 pregnancies. Its incidence is higher among women at the two extremes of maternal age than in other women. The serum and urinary chorionic gonadotropin levels are usually abnormally high. In complete mole, the cause is fertilization of an ovum which has lost its chromosomes. Almost all have a 46XX diploid pattern from the sperm by androgenesis. There are no fetal parts. Almost every chorionic villus is cystic and avascular. There is diffuse trophoblastic proliferation. About 2 percent develop choriocarcinoma. Partial mole is due to fertilization of an ovum by 2 sperms. It has a triploid 69XXY chromosomal pattern. Only some villi are cystic, and there is slight trophoblastic proliferation. Fetal parts may be present. Choriocarcinoma rarely develops from a partial mole. The first manifestation of the presence of a mole is often the excessively rapid enlargement of the uterus due to growth of the placental mass. This is usually followed, at about the third to fifth month of pregnancy, by uterine bleeding or abortion. Placental tissue passed at this time show that the villi are cystic and grape-like. Microscopically, the mole shows cystic avascular villi together with irregular clumps of large syncytial and cytotrophoblastic cells. Hydatidiform mole may be complete or partial. It is characterized by cystic chorionic villi. Choriocarcinoma may develop in a mole, especially the complete type. About 50 percent of gestational choriocarcinomas arise in moles. Many of these have been treated successfully with chemotherapy. Choriocarcinoma is a rare neoplasm. In about half the cases, it arises in women with a hydatid mole. The rest of the cases are about equally divided between abortions and full-term pregnancy. A few develop as a teratoma unrelated to a pregnancy. Grossly, the uterus containing a choriocarcinoma shows shaggy, fleshy hemorrhagic masses filling the uterine cavity and extending into the myometrium. Microscopically, the neoplasm is composed of clusters of large, bizarre, often multinucleated giant trophoblastic cells, extending into the myometrium and often filling the vascular sinuses. Necrosis and hemorrhage are seen, both in the tumor masses and in the myometrium. Very rarely both the primary neoplasm and the metastases have disappeared spontaneously. Numerous cures have been achieved by methotrexate and actinomycin D in choriocarcinomas related to a mole or pregnancy. Typically the tumor spreads locally to the vagina and parametrium, invades the veins, and metastasizes to the lungs, liver, and brain. Usually the neoplastic cells produce chorionic gonadotropin in both the primary growth and the metastases, sometimes with extremely high titers. Most ectopic pregnancies occur in a uterine tube. Any pathologic change which causes narrowing or blockage of the tubal lumen predisposes to tubal pregnancy. The narrowing may be intrinsic due to inflammation or extrinsic due to formation of adhesions around the tube. Tubal ligation is designed to prevent fertilization of ovum by sperm and thus any pregnancy. Tubal pregnancy is the most common form of ectopic pregnancy. It occurs about once in 200 pregnancies. The burrowing trophoblast usually erodes and ruptures the tube early in pregnancy. Sometimes tubal abortion occurs, the fetus being extruded from the lateral end of the tube into the abdominal cavity. Interstitial pregnancy is implantation in a cornu of the uterus in the interstitial portion of the uterine tube. 'Abdominal' pregnancy (peritoneal pregnancy) may result either from direct implantation of the fertilized ovum upon the peritoneum or from extrusion of the fetus of a tubal pregnancy into the abdominal cavity. Ovarian pregnancy is extremely rare and presumably results from fertilization of the ovum at the time of ovulation.

A 27-year-old G3P2 at 39 weeks gestation has been in labor for 6 hours. Her membranes ruptured 3 hours ago and revealed a large amount of clear fluid. She has previously delivered a 7 pound 2 ounce infant. She is having good quality contractions, and has been completely dilated and pushing for 55 minutes. The fetal head is +4, FHT 128. What would be the most appropriate management at this time? A Continue pushing B Instrument delivery C Emergency caesarean section D Non emergent caesarean section E Epidural pain control

Continue pushing A This is a multigravida patient with an adequate pelvis. She is making steady progress and there are no signs of maternal or fetal distress, so continued labor management is expected.

A 35-year-old female presents to discuss non-permanent methods of birth control. She is a G3P2, PMH negative. She is a smoker. Which method of contraception would have the lowest risk profile for her? A Combination low dose pill daily B Injectable progestin monthly C Transdermal combination patch D Levonorgestrel intrauterine device E Copper intrauterine device

Copper IUD E A women who is over 35 and smokes is at high risk for cardiovascular complications. All choices except the copper IUD contain hormones, which may increase the risk of complications. The copper IUD is long term but non-permanent.

A 47-year-old female presents to the office complaining of excessive menstrual flow for the past two months. She complains of mid-cycle bleeding, as well as periods that last for nine days. What is the mostly likely cause of her bleeding? A Chlamydia trachomatis infection B Adenocarcinoma of the uterus C Uterine leiomyoma D Anovulatory uterine bleeding E Pregnancy

D Anovulatory uterine bleeding D While all are causes for abnormal bleeding, anovulotory bleeding occurs in perimenopausal women due to the unopposed estrogen. STD and pregnancy would be more likely during reproductive years. Neoplasms accounts for approximately 20% of abnormal uterine bleeding in the post reproductive years.

A 24-year-old female, with a history of type 2 diabetes, presents with the inability to conceive after 14 months of unprotected sexual intercourse with her husband. Vital signs are as follows: Temperature 37˚ C, BP 115/85, P 65, RR 20. Her BMI is 31. Which physical examination finding would you expect to observe with your suspected diagnosis? A Undeveloped breasts B Palpable uterus on abdominal exam C Acromegaly D Coarse dark hair on the face, chest, and back E Egophony on lung exam

D Coarse dark hair on the face, chest, and back Polycystic ovarian syndrome (PCOS) affects 5-10% of reproductive age women. It is associated with hirsutism (50% of cases) (D), obesity (80% of cases), and virilization (20%), and those affected have an increased risk of diabetes mellitus, cardiovascular disease, and metabolic syndrome. PCOS patients are often infertile. A palpable ovary may be noted on thin individuals with enlarged ovaries, not the uterus (B). Undeveloped breasts are not a diagnostic indicator for PCOS (A) and acromegaly (C) is seen with Marfan's Syndrome. Egophony (E) is when there is consolidation of the lung on exam.

An 18-year-old G1P0 presents for evaluation of her amenorrhea. Her LMP was 2 months ago. Her HCG is positive, and she has been having vaginal spotting x 2 weeks. Her ultrasound reveals an intrauterine heterogeneous echogenic mass, without fetus or placenta. What is the most likely diagnosis? A Threatened abortion B Missed abortion C Ectopic pregnancy D Hydatidiform mole E Early intrauterine pregnancy

D Hydatidiform mole D The classic signs for a mole are a heterogenous mass without the placenta or fetus, and vaginal spotting is present all of the time. The presence of the mass intrauterine without fetus or placenta rules out ectopic and IUP, and therefore threatened abortion. Missed abortion is not echogenic.

A 27-year-old G 0 P 0 woman has been trying to become pregnant for the past 10 months. Menarche occurred at age 13 and her cycles have "always been irregular" except during the 10-year period when she took oral contraceptives. Since stopping contraception two years ago, she and her partner of six years have used condoms consistently until they desired pregnancy. Neither partner has a history of sexually transmitted infection, and both have been mutually monogamous. On physical examination she is 64" tall and weighs 189 pounds. She has increased dark hair on her upper lip and chin and on her lower abdomen and says she has had this hair distribution as long as she can remember. Of the following, what is the most likely cause of her apparent infertility? A Androgen-secreting ovarian neoplasm B Hypothyroidism C Pituitary adenoma D Polycystic ovarian syndrome E Premature ovarian failure

D Polycystic ovarian syndrome D Patients with polycystic ovarian syndrome typically are obese, infertile, hirsute, and have menstrual disorders, making this the most likely of the diagnoses. An androgen-secreting ovarian neoplasm (A) would cause infertility, but the associated hirsutism is rapidly progressing. Hypothyroidism (B) may cause amenorrhea and weight gain and contribute to infertility, but the overall picture is more suggestive of PCOS. The hyperprolactinemia associated with pituitary adenoma (C) might also produce mild hirsutism, but other symptoms such as galactorrhea would also likely be present. Premature ovarian failure (E) is the spontaneous cessation of menses prior to age 40, and does not apply to this woman.

A woman presents for a routine post-partum checkup four weeks after delivery of her child. She is currently breast feeding without difficulty. She tells you that she has been feeling anxious and very warm, despite the change of season into winter. She is happy as a mother and has not had problems caring for her baby. What is the next best step in the investigation of her symptoms? A Radioactive iodine uptake test B Serum total T4 level C Thyroid fine needle aspiration D Serum TSH level E MRI of the anterior pituitary

D Serum TSH level The correct choice is D, serum TSH level. This patient is presenting with post-partum thyroiditis. Thyroid dysfunction occurs in 2 to 5% of women after giving birth. It can recur with subsequent pregnancies and develop into long-term thyroid disease as well. Typically, women with this disorder first develop signs and symptoms of hyperthyroidism, which then later changes to hypothyroidism. Most symptoms resolve spontaneously within a few months. In the hyperthyroid stage of this disorder, blood tests will reveal a suppressed TSH level with elevated serum thyroid hormone levels, as is common in all forms of primary hyperthyroidism. Choice A, a radioactive iodine uptake test, would reveal little or no uptake, but this test should not be ordered in a woman who is breastfeeding. For choice B, a serum total T 4 level can be elevated, but is not as sensitive or specific as TSH as a screening test. Any protein status changes in the woman can cause an elevated total T 4 without any thyroid dysfunction. Choice C, a thyroid fine needle aspiration, can be performed, but is not likely necessary and would not be the best next step for this patient. Choice E, an MRI of the anterior pituitary, would only be suggested if there is a suspicion of a pituitary tumor. This is a rare cause of hyperthyroidism and would not be the best next step.

A progestin-only contraceptive, or "minipill," would be most appropriate for which of the following patients? A a 25-year-old woman in excellent overall health B a 28-year-old woman with a history of epilepsy C a 32-year-old woman with a history of pelvic inflammatory disease D a 37-year-old woman who smokes 2 packs per day and has a history of hypertension E a 38-year-old woman with a history of asthma and bronchitis

D a 37-year-old woman who smokes 2 packs per day and has a history of hypertension D In the majority of cases, a combined hormonal contraceptive (ie, one that contains both an estrogen and progestin) is the preferred method of oral contraception because of its efficacy when used perfectly (>99%). However, for women older than 35 years of age who are smokers or are obese, or who have a history of hypertension or vascular disease, progesterone-only contraceptives are recommended. Ethinyl estradiol (EE), the most common estrogen found in combined hormonal contraceptives, has been associated with an increased risk of myocardial infarction in women older than 35 years of age who are smokers. Additionally, EE has also been shown to cause increases in blood pressure in both normotensive and mildly hypertensive women. Progestin-only contraceptives, however, tend to be less effective than the combined hormonal contraceptives

A patient with no history of treatment for primary syphilis presents with symptoms and signs consistent with secondary syphilis. The most common sign of secondary syphilis is A generalized lymphadenopathy B aseptic meningitis C alopecia D generalized maculopapular rash E superficial painless gummas

D generalized maculopapular rash Secondary syphilis generally manifests itself a month or two after appearance of the primary chancre. Patients will complain of headache, fever, sore throat, and malaise and will exhibit generalized lymphadenopathy along with a maculopapular rash that begins at the sides of the trunk and later spreads over the rest of the body. The skin lesions may coalesce in warm moist areas, such as the perineum, and form large, flat-topped, pale papules termed condyloma lata. Skin and mucosal lesions are the most common signs of secondary syphilis. Aseptic meningitis and alopecia may also occur in secondary syphilis. Formation of granulomatous nodules (gummas) is not a feature of secondary disease, but rather is the hallmark of tertiary syphilis.

Which of the following U.S. Preventive Services Task Force (USPSTF) recommendations for prostate cancer is correct? A All African-American men over 40 years old should have a digital rectal examination (DRE) and a prostate-specific antigen (PSA) screening annually. B All patients should receive a DRE and PSA screening at 50 years of age. C All high-risk patients should receive a PSA screening annually starting at 50 years of age. D DREs are not recommended and PSA screening is recommended based upon patient risk, but only after discussing the benefits and risks with the patient and the patient expressing the desire to have the examination performed.

DREs are not recommended and PSA screening is recommended based upon patient risk, but only after discussing the benefits and risks with the patient and the patient expressing the desire to have the examination performed.

Your patient is a 45-year-old woman concerned about the accelerated aging of her skin. 5 years ago, she underwent a total hysterectomy with oophorectomy. Since then, she gained about 20 pounds. She also has diabetes mellitus type II. On examination, you find a slightly overweight female (body mass index 26) with skin that has lost its elasticity and has reduced water-holding capacity, increased pigmentation, and decreased vascularity. Question What are her symptoms signs of? Answer Choices 1 Elevated testosterone 2 Decreased testosterone 3 Elevated estrogen 4 Decreased estrogen 5 Insulin resistance

Decreased estrogen Explanation Lower levels of estrogen are associated with skin aging, most probably because of telomeres shortening. The effects of reduced estrogen levels cause loss of elasticity, reduced water holding capacity, thickness, increased pigmentation, decreased vascularity in the skin, and facial hair. Some of those signs are found in your patient. Your patient has no ovaria, and testosterone is primarily secreted in the ovaries (small amounts are also secreted by the adrenal glands). Too much testosterone will cause excessive oiliness and acne rather than accelerated aging of the skin. Testosterone levels in this woman are decreased after the removal of the uterus and ovaries. Symptoms related to her skin are not characteristics of decreased testosterone levels. More often in decreased testosterone there will be a loss of muscle strength and mass, accumulation of fat, especially around the abdomen, osteoporosis, vaginal dryness, etc. High levels of estrogen can be found in women who are pregnant, extremely overweight, have diabetes, or have liver disease. They are sometimes associated with acne, red flushed appearance, spider nevi, etc. Skin changes in this patient cannot be attributed to high estrogen. Insulin resistance is a condition when insulin becomes less effective at lowering blood sugars. It can lead to weight gain and estrogen hyperproduction, resulting in skin changes attributable to high estrogen levels (acne, red flushed appearance, spider nevi, etc). Skin changes in this patient cannot be attributed to the high estrogen.

Which of the following is an acceptable method for diagnosing endometriosis in a patient who describes a history of increasing pelvic pain which is cyclic in nature and increases in intensity just before menses? Answer Choices 1 Ultrasound 2 Endometrial biopsy 3 Magnetic resonance image (MRI) 4 Serum carcinoembryonic antigen (CA 125) 5 Diagnostic laparoscopy

Diagnostic laparoscopy Explanation Endometriosis refers to the presence of endometrial glands and stroma implanted in a location outside of the intrauterine cavity. There are multiple theories regarding the mechanism by which this occurs, they include: retrograde menstruation, metaplasia of coelomic epithelium, and embryonic rests or remnants. It is a progressive disease and often results in significant management problems. Interestingly, the extent of the disease often does not correlate well with the perceived degree of discomfort. Although various imaging techniques have been described as being helpful in following the progression of known pelvic disease, such as endometriosis, none are considered acceptable for the actual diagnosis of the condition. CA 125 is a cell surface antigen found in tissues derived from coelomic epithelium, including gastrointestinal and gynecologic tissues. Although it can be use to monitor the progress of a diagnosed inflammatory or malignant process, it is not diagnostic and should not be used as a screening test. The current standard of care regarding the definitive diagnosis of endometriosis is laparoscopic visualization, preferably with a confirmatory biopsy of a lesion.

A 27-year-old female is 8 weeks postpartum with her first child and has been exclusively nursing since discharge at the hospital. She has a 5-day history of engorgement in her right breast, which is red, tender, and feels warm to the touch. She states she is feverish but has not taken her temperature. She reports no known drug allergies. On physical examination you see the breast as shown below. Which of the following is the most appropriate therapy in the management of this patient? A Cephalexin 500 mg every 6 hours for 3 days; discontinue breastfeeding B Cephalexin 500 mg every 6 hours for 3 days; continue breastfeeding C Discontinue breastfeeding and see if symptoms resolve D Dicloxacillin 500 mg orally every 6 hours for 10 days; continue breastfeeding E Dicloxacillin 500 mg orally every 6 hours for 10 days; discontinue breastfeeding

Dicloxacillin 500 mg orally every 6 hours for 10 days; continue breastfeeding Staphylococcus aureus is usually the causing agent in puerperal mastitis. Treatment includes antibiotics that are effective against penicillin-resistant staphylococci and nursing of the infant with the affected breast is safe. Alternatively, a cephalosporin can be given orally for 10-14 days. Cephalexin 500 mg every 6 hours (A and B) can be used but should be continued for longer than 3 days. Failure of the patient to respond to antibiotics after three days becomes concerning for an abscess or resistant organism. Further evaluation would be needed including surgical consultation, ultrasound, and hospital admission for IV antibiotics. Dicloxacillin is commonly used in uncomplicated cases without any known allergies. To preserve milk supply, it is encouraged to continue breastfeeding with mastitis; do not recommend discontinuing breastfeeding (A, C, and E).

A 26-year-old woman is in the 24th week of her first pregnancy. She is in fairly good shape, and the pregnancy is progressing well. However, a fasting blood glucose done in the office shows it to be 146 mg/dl. It is repeated the next day, and the value is 142 mg/dl. An oral glucose tolerance test is ordered, which comes back as abnormal. She is diagnosed with gestational diabetes and advised to meet with her obstetrician. What should be the recommended regimen for this patient? Answer Choices 1 Diet and exercise until delivery 2 Diet and metformin if blood glucose remains high despite diet control 3 Diet and sulfonylurea if blood glucose remains high despite diet control 4 Diet, exercise, and insulin therapy if blood glucose remains high despite diet control 5 Diet and acarbose if blood glucose remains high despite diet control

Diet, exercise, and insulin therapy if blood glucose remains high despite diet control Explanation Gestational diabetes mellitus (GDM) should be treated early and optimally to reduce risks to the fetus. In pregnancy there is an increased resistance to insulin which causes high levels of glucose, lipids, and insulin. Placental lactogen and high levels of estrogen and progesterone cause this scenario. Risk factors for GDM include family history of diabetes, prior delivery of large baby, age >25 years, obesity, steroid use, glycosuria in first antenatal visit, ethnicity like Hispanic, African, Native American, Asian or Pacific Islander, polycystic ovarian disease (PCOS), and prior fetal loss of unknown cause. Hyperglycemia is harmful to the mother and the fetus and can cause increased perinatal mortality. Preeclampsia and polyhydramnios have a higher incidence in GDM. Trauma during birth and operative delivery are some other adverse outcomes. Macrosomia, or larger than normal fetus, is common. Metabolic abnormalities in the baby post delivery like hypoglycemia and hyperbilirubinemia, childhood obesity, attention deficit disorder (ADHD), and early diabetes may occur in the child. Universal screening of all pregnant women is recommended (though very low risk females may be excluded), usually at 24-28 weeks of pregnancy except if risk factors are evident like obesity, prior history of GDM etc, in which case screening should be done as early as possible. A random blood sugar of >200 mg/dL or fasting glucose of >126 mg/dL on 2 occasions is diagnostic of diabetes and does not require screening. Screening is done with a 50 gm oral glucose challenge test. A value of >140 mg/dL is positive, and a 3-hour glucose tolerance test is then done for definitive diagnosis. GDM is confirmed if 2 or more of the following are present: fasting glucose> 95 mg/dL, 1-hour glucose >180 mg/dL, 2-hour glucose >155 mg/dL, and 3-hour glucose >140 mg/dL. Referral to a nutritionist for strict diet control is a must to maintain euglycemia, prevent ketosis, and monitor adequate weight gain. Blood glucose should be measured before breakfast and 1 hour after each meal by the patient and moderate exercise is encouraged. Fasting blood glucose should be <90 mg/dL and post-prandial should be <120 mg/dL. If blood glucose remains high in spite of these measures, then insulin is the treatment of choice. Oral hypoglycemic agents are not approved for use in pregnancy in USA due to transplacental passage and fetal morbidity. If fasting blood glucose is high, then NPH insulin is used at bedtime in the dose of 0.2 units/kg body weight. If post-prandial glucose is high, then regular insulin is used before meals. If both are high, then a regimen of 4 injections a day needs to be started, including NPH before breakfast and at bedtime and regular insulin before each meal.

Which of the following, is a classical hydatidiform mole? Answer Choices 1 Diploid karyotype, absence of an embryo, swelling of all villi 2 Triploid karyotype, trophoblastic hypoplasia, viable embryo 3 Trophoblastic hyperplasia, normal karyotype, twin embryos 4 Vascular malformations, intraplacental mass lesions, fetal hydrops 5 Decreased uteroplacental blood flow, multiple placental lobes, vesicular pattern

Diploid karyotype, absence of an embryo, swelling of all villi

You are a general practitioner in private practice and have been treating a 12-year-old boy with headache, fever and meningismus. You suspect meningitis and send him off to the nearest hospital; an analysis of cerebral spinal fluid indicates meningococcusmeningitis. All the members of your team who have come in close contact with the boy need to take rifampicin p.o. for 2 to 4 days prophylactically. One nurse is worried because she might be 1 week pregnant. What do you do? Answer Choices 1 Give her a urine pregnancy test 2 Draw blood for quantitative ßHCG 3 Give her a one-shot rifampicin 4 Give her the same rifampicin dosage as the others 5 Substitute rifampicin with tetracycline

Draw blood for quantitative ßHCG Explanation The correct response is to draw blood for quantitative βHCG. If the nurse is only 1 week pregnant, you could prescribe the same rifampicin p.o. dosage as for the others. At that time, there is no direct blood flow between the conceptus and maternal circulation; you have to draw blood for quantitative ßHCG level to make sure that, if she is pregnant, she is not further along. A urine pregnancy test usually shows a positive result about 2 weeks after the time the next menstruation is expected. Since rifampicin is not available in injectable form, there is no possibility for a one-shot. Tetracycline is not approved for prevention of meningococcus meningitis; furthermore, it is contraindicated during pregnancy.

A 37-year-old female presents to the office complaining of increasing heavy vaginal bleeding with her menses. CBC reveals an iron deficient anemia. Her pelvic exam is notable for a moderately enlarged uterus. A transvaginal ultrasound reveals a 3-cm leiomyoma and a 15-cm leiomyoma. The patient wishes to preserve her fertility. Appropriate management would include which of the following? A Expectant management aimed at pain reduction B Bilateral uterine artery embolization C Reduction of blood loss with combined oral contraceptives D Immediate hysterectomy E Gonadotropin-releasing hormone analog followed by myomectomy

E Gonadotropin-releasing hormone analog followed by myomectomy E GRH causes a reversible hypogonadism, which reduces tumor size, makes surgical intervention safer, and reduces bleeding. The patient wishes to preserve her fertility, making embolization and hysterectomy non-viable options. COC do not significantly reduce bleeding, and do not regress the tumor for optimal surgical removal.

A 38-year-old woman presents for a 3-month follow-up for her HIV infection. She states that she is doing well and adheres to her HIV medication regimen with minimal side effects. She is currently on zidovudine, lamivudine, and efavirenz; she has been on them for the past 5 years, and she has a consistent viral load of <50 copies/mcL and a current CD4 cell count of 306 cells/mcL. She is also on trimethoprim/sulfamethoxazole (Bactrim) for pneumocystis prophylaxis, and she takes insulin for diabetes. She states that she has not been taking her birth control pills, and she took a urine pregnancy test at home yesterday that was positive. A serum pregnancy test confirms her pregnancy. Question What medication mentioned has the most reproductive risk potential? Answer Choices 1 Insulin 2 Zidovudine 3 Lamivudine 4 Efavirenz 5 Sulfamethoxazole / trimethoprim (Bactrim)

Efavirenz, a pregnancy Category D medication, may cause fetal harm when taken by a woman during the 1st trimester of her pregnancy; therefore, it should be avoided. If a woman becomes pregnant while taking Efavirenz, she should be notified of the potential harm to the fetus, and Efavirenz should only be continued if the potential benefits outweigh any risks to the fetus (e.g. no other treatment options are available for her).

A 51-year-old female presents to the office complaining of intermittent vaginal spotting for three months. She has a history of well-controlled hypertension. She is a nonsmoker. Her LMP was two years ago. Her family history is significant for colon cancer. Physical exam and pelvic exam were performed and unremarkable. What is the best diagnostic step in evaluating her vaginal bleeding? A Transvaginal ultrasound B Endometrial sampling C Pap smear D Pregnancy test E STD testing

Endometrial sampling B In postmenopausal women with a family history of colon cancer, there is a 30% risk of endometrial cancer. Pregnancy and STD are less likely, and do not exclude endometrial cancer. Pap smear and ultrasound are useful but can be negative. Endometrial sampling is required to confirm or rule out cancer.

A 46 year-old female has just been diagnosed with ovarian cancer. Where would you expect the ovarian neoplasm to most likely arise from? A Germ cell B Stromal cell C Epithelial cell D A Metastatic tumor E Endometrial cell

Epithelial cell C The most common and most lethal of the ovarian neoplasms arise from the ovarian epithelium found both on the surface of the ovary and in subsurface locations. The ovarian epithelium generally is in good health and appears as a simple epithelium. With neoplastic transformation there undergoes metaplastic changes into what is termed Müllerian epithelium. The Müllerian epithelium has a variety of subtypes, each providing a specific phenotype of the tumor and may have a different clinical presentation. Epithelial tumors are the most common ovarian neoplasm and may be: benign (50%); malignant (33%); or borderline malignancy (16%). Tumors may also metastasize (D) to the ovary from the breast, gastric, pancreatic, and colon primary cancers, but is not as common. Germ cell tumors (A) are more similar to testicular tumors in males. Stromal tumors (B) arise from steroid hormone producing cells. Endometrial (E) is usually specific to the uterus.

A 46 year-old female has just been diagnosed with ovarian cancer. Where would you expect the ovarian neoplasm to most likely arise from? A Germ cell B Stromal cell C Epithelial cell D A Metastatic tumor E Endometrial cell

Epithelial cells The most common and most lethal of the ovarian neoplasms arise from the ovarian epithelium found both on the surface of the ovary and in subsurface locations. The ovarian epithelium generally is in good health and appears as a simple epithelium. With neoplastic transformation there undergoes metaplastic changes into what is termed Müllerian epithelium. The Müllerian epithelium has a variety of subtypes, each providing a specific phenotype of the tumor and may have a different clinical presentation. Epithelial tumors are the most common ovarian neoplasm and may be: benign (50%); malignant (33%); or borderline malignancy (16%). Tumors may also metastasize (D) to the ovary from the breast, gastric, pancreatic, and colon primary cancers, but is not as common. Germ cell tumors (A) are more similar to testicular tumors in males. Stromal tumors (B) arise from steroid hormone producing cells. Endometrial (E) is usually specific to the uterus.

A 45-year-old woman presents with a 3-day history of persistent nipple discharge. She is essentially healthy with an insignificant past medical history. She quit her job to stay home with her young children in the past year; she has noted a weight gain of 15 lbs, which she attributes to being at home more, leading to increased snacking as well as less activity. She also says her periods have spaced out, occurring anytime from 28 - 53 days. They still last about 2 - 3 days, as they previously had. She denies hot flashes and the possibility of pregnancy; her husband had a vasectomy. Her review of systems is otherwise negative. She takes Vitamin E every day, but she takes no other medications or herbs; she denies the use of illicit drugs. On physical exam, you note a healthy-looking woman. Her breast exam is negative for lumps, dimpling, and nipple retraction. You are able to express some clear fluid from the left nipple. It seems serous. Her axillary exam is negative for any enlarged lymph nodes; the rest of her exam is normal. Question What is the next step in the management of this patient? Answer Choices 1 Check an FSH 2 Do microscopy on the fluid 3 Check a prolactin level 4 Order a mammogram 5 Check a TSH

Explanation A mammogram should be ordered. The patient is a 45-year-old woman with spontaneous serous discharge. Nipple discharge is a common breast complaint and is usually associated with benign etiology. Because most nipple discharge is a result of benign processes, less-invasive, non-surgical diagnostic modalities have been explored to alleviate the need for surgical intervention. Most of the patients with nipple discharge have benign disease, such as intraductal papilloma, papillomatosis, duct ectasia, or fibrocystic condition. Studies have shown that approximately 11% of patients will have an underlying carcinoma. To be significant, nipple discharge should be spontaneous, persistent, and non-lactational. Persistent clear or bloody nipple discharge merits, at the very least, diagnostic breast imaging. All nipple discharges associated with a breast lump must be surgically evaluated. The incidence of associated cancer is much higher when the discharge is serosanguineous or sanguineous. Mammography has been utilized for evaluation of obvious abnormality to guide surgical intervention. The sensitivity of mammography for detecting malignant ductal pathology is 57% and the positive predictive value is 16.7%. It should be the initial test in patients with clinically significant nipple discharge and in women above the age of 40 years. Cytological analysis of the discharge fluid has traditionally shown high false-negative rates, and it is not revealing in localizing the lesion. Hemoccult testing has been shown to be useful for determining the presence of blood, but its use in predicting histology is questionable. For the patient in this case, all the tests noted (A through E) could be indicated. An FSH can be used to check for menopausal status, and a prolactin level could be done to check for abnormal levels. A TSH to check for hypothyroidism could also be done; decreased thyroid hormones are associated with galactorrhea. Hypothyroidism would also explain the menstrual irregularity. Mammography should be conducted initially; later, hormonal assay and a cytological examination can be carried out. In summary, given that this patient presented with spontaneous persistent discharge, the most appropriate initial approach would be to perform a mammography.

What component of a patient's health history would be considered protective against breast cancer? Answer Choices 1 Onset of menses at age 11 years 2 Breast fed all 3 of her children 3 Recovering alcoholic for the last 6 months 4 Mother had menopause at the 4th decade of life

Explanation Breast-feeding is protective for breast cancer. Early menarche and excessive alcohol use are factors that increase susceptibility to breast cancer. Mother's age of menopause does not affect breast cancer risk one way or another.

A 33-year-old woman presents to discuss management of her premenstrual dysphoric disorder. The patient experiences significant depression and irritability for the 2nd half of her menstrual cycle nearly every month, but she has refused a trial of a selective serotonin reuptake inhibitor (SSRI) because she doesn't want to be on an antidepressant. She has previously been on oral contraceptives, but she stopped taking them due to worsening of her migraine headaches. She asks you about herbal remedies. What is true regarding herbal agents that contribute towards improvement in premenstrual dysphoric disorder? Answer Choices 1 Chaste tree berry decreases secretion of luteinizing hormone and has been proven to reduce PMS symptoms in many women 2 Gingko biloba has been shown to worsen breast pain associated with PMS 3 Chaste tree berry increases secretion of luteinizing hormone and has been proven to improve PMS symptoms 4 Natural progesterone has been proven to improve PMS symptoms in numerous well-controlled trials 5 St. John's Wort does not significantly improve the dysthymic symptoms of PMS

Explanation Chaste tree berry increases secretion of luteinizing hormone and has been proven to improve PMS symptoms. Premenstrual syndrome (PMS) is defined as symptoms of mood changes, such as depression, nervousness, increased irritability and lability, and anxiety, as well as physical symptoms, such as breast pain, headaches, and edema (weight gain). These symptoms usually begin 7 - 10 days before the onset of menses and abate quickly as soon as menses begin; however, some individuals with more severe premenstrual dysphoric disorder symptoms can experience symptoms beginning shortly after ovulation, and thus approximately half of each month is affected by these complaints. Nearly 80% of women experience some emotional or physical symptoms prior to their menses, although only about 1/3 of these women have significant problems due to these symptoms. Only about 2.5 - 5.0% describe their premenstrual symptoms as having a negative effect on their lives to the point where occupational or interpersonal relationships are affected. The type and intensity of symptoms are different between different patients, and they can vary from cycle to cycle in a given woman. PMS is most likely due to fluctuations in levels of progesterone and estrogen. Both estrogen and progesterone can cause fluid retention, which seems to explain some of the physical complaints associated with PMS. Some studies have suggested that patients with PMS symptoms metabolize progesterone differently, causing less production of allopregnanolone, a GABAergic substance that reduces anxiety. Selective serotonin reuptake inhibitors (SSRIs) have been proven to improve the symptoms of premenstrual dysphoric disorder and are the most effective agents in the treatment of the symptoms of PMS. Other useful agents include oral contraceptives to reduce the cyclic variations in estrogen and progesterone. Supplementing the diet with pyridoxine has also been shown to improve some of the symptoms of PMS. Diuretics had been used in the past to help with symptoms of water retention, but their use is losing favor in recent years. For patients who would prefer a naturopathic approach to the management of PMS, some "natural" agents have been shown to be effective in the treatment of PMS. Chaste tree berry is probably one of the most effective botanical agents for the treatment of PMS. It is thought that chaste tree berry increases secretion of luteinizing hormones and may affect progesterone. Several studies have found chaste tree berry to be more effective than placebo in treating both the emotional (mood changes, irritability, and anger) and some of the physical symptoms (headache and breast tenderness, although not bloating) of PMS. However, these improvements have not been replicated in all studies of this agent, with other studies showing no improvement in symptoms when compared to placebo. St. John's Wort has been shown to be useful in treating the emotional, dysphoric symptoms of PMS. Patients in one small trial showed improvement in depression, confusion, anxiety, nervousness, and insomnia associated with PMS when treated with St. John's Wort. Ginkgo biloba has been shown to improve the breast pain and tenderness associated with PMS, but has not been useful in treating other premenstrual complaints. Natural progesterone has been touted as an effective treatment for PMS, although there is not much data to support this claim; in fact, several studies have shown no improvement in the symptoms of PMS when compared to placebo, especially for the vaginal and rectal preparations of progesterone. An exception to this is the oral micronized formulation of progesterone, which has been shown to improve anxiety, depression, fluid retention, and hot flushes in PMS. Additionally, many naturopathic practitioners advocate the use of natural progesterone cream rubbed into the skin, although this is not supported by scientific study.

A 53-year-old woman is seen by her gynecologist. She had 3 children, and she is tubectomized. She attained menopause at 48. Over the last few weeks, she has noted some vaginal bleeding; it occurs unpredictably. She has had regular pap smears that have always been normal. The last one was 11 months prior to presentation. Speculum examination and bi-manual palpation of the genitalia reveal no abnormalities. Question What is the next best step in management of the patient? Answer Choices 1 Repeat pap smear 2 Chlamydia serology 3 Laparoscopy 4 Endometrial biopsy 5 Hormonal profile

Explanation Endometrial biopsy is the correct answer. Vaginal bleeding in a post-menopausal woman mandates endometrial sampling, with a view to ruling out endometrial cancer. Histopathological evaluation of material is essential. The most common causes of post-menopausal bleeding are vaginal atrophy (60 - 80%), exogenous estrogens (15 - 25%), endometrial cancer (10%), endometrial hypertrophy (5 - 10%), as well as cervical and endometrial polyps (2 - 12%). Repeat pap smear is incorrect. The patient had one done less than a year ago, which was normal. In light of her previously normal pap smears, the next one is not due for another 3 - 5 years. Pap smears may show the occasional dysplastic cell of endometrial origin, but it is by no means an accurate means of diagnosing endometrial pathology. Chlamydia serology is incorrect. It is done for the diagnosis of pelvic discharge in sexually active women; it is not used in cases of post-menopausal bleeding Laparoscopy is incorrect. It is not useful in the workup of post menopausal bleeding. Hormonal profile is incorrect. While exogenous estrogen is a possible cause of post menopausal bleeding, it is a diagnosis made from taking a careful history.

A 28-year-old pregnant woman at 18-weeks gestation presents because she has been exposed to fifth disease. The patient is currently asymptomatic. What can you tell her about human parvovirus B19 and pregnancy? Answer Choices 1 Less than 25% of adults are immune to this virus, and she should be treated with immunoglobulin 2 Parvovirus B19 has not been associated with any fetal effects 3 If she develops an infection, she will need to be followed with serial fetal ultrasounds 4 The virus is spread by fecal-oral transmission, and she should wash her hands very carefully 5 The risk of fetal loss in an infected mother is between 45-55%

Explanation Fifth disease is caused by human parvovirus B19, which is a DNA virus. Fifth disease, also called erythema infectiosum, is usually a mild exanthem of childhood, but infection of a pregnant woman can have severe fetal complications Fetal (transplacental) infection with parvovirus B19 can result in a variety of fetal complications, including fetal loss, especially if the infection is between gestational weeks 10 and 20. A common complication includes fetal hydrops; it is caused by damage to fetal hematopoietic tissue, and it causes severe anemia and a resultant congestive heart failure. The virus can also cause a fetal viral myocarditis, which further worsens cardiac function and fetal hydrops. More rarely, first trimester infections with parvovirus B19 can cause teratogenic effects, including multiorgan abnormalities. Between 30-60% of adults are immune to parvovirus B19, as evidenced by the presence of IgG to B19 in their serum; therefore, most pregnant patients are probably immune to this virus. However, if a pregnant patient develops a rash or aplastic crisis that may be consistent with fifth disease, then igG and IgM serologies for parvovirus should be drawn to evaluate for acute infection. If parvovirus B19 infection is diagnosed in a pregnant patient, then her physician may choose to follow her with serial fetal ultrasounds to evaluate for the development of fetal hydrops. In some cases, fetal umbilical cordocentesis has been used to detect fetal infection. In children, fifth disease is characterized by a classic "slapped-cheek" facial erythema; it is associated with fever and often GI or other systemic symptoms. Adult patients will demonstrate a rash. The rash may be reticular, morbilliform, or even purpuric. Adult patients often have fever, lymphadenopathy, and/or arthritis. Parvovirus B19 is also associated with an acute transient aplastic crisis. Unfortunately, fifth disease is infectious for days before the onset of the rash; thus, many obstetrical patients may be exposed, particularly if they work closely with children. The virus is spread by aerosolized respiratory droplets, and it has an incubation period of 4 to 14 days.

A 26-year-old woman presents with an abnormal lump in her right breast. Physical examination demonstrates a well-circumscribed, palpable mass in the upper outer quadrant of the right breast. The patient denies a family history of breast carcinoma or any additional medical problems. Question What would be the best test to order for further evaluation of this palpable abnormality? Answer Choices 1 Bilateral mammography 2 Right breast MRI 3 Right galactogram 4 Stereotactic biopsy 5 Focused ultrasound

Explanation In women under 30 years of age with a palpable lump in their breast, the initial imaging modality should be an ultrasound. Ultrasound is an excellent modality in this scenario because it can define the borders of the mass and determine whether the lesion is solid or cystic. If the lesion is solid, then the characteristics of the mass can be better defined before either following the lesion or proceeding to biopsy. Bilateral mammography is not indicated in this patient (who is under the age of 30) at this point. If the patient were > 30 years of age with a palpable breast lump, then a unilateral mammogram and focused ultrasound would be the best tests to order for further evaluation. However, in patients < 30 years of age, ultrasound is a better choice for the initial test because most of these 'lesions' are fibroadenomas. Additionally, it is desirable to limit radiation exposure in patients younger than 30 years of age, thus supporting ultrasound as the best choice in this case. Breast MRI exams can be used to evaluate for lobular cancer; occult breast carcinoma; to define the extent of disease in a patient with a positive surgical margin; to evaluate for multifocal, multicentric, or bilateral cancers; and to evaluate for postoperative scar versus tumor recurrence. Breast MRI is not the initial test that should be ordered to evaluate a 26-year-old patient with a palpable breast mass. Galactograms are types of mammograms whereby a breast duct is accessed using a blunt needle to evaluate for unilateral or bilateral bloody nipple discharge, which is most often caused by an intra-ductal papilloma, but could be caused by intra-ductal papillary carcinoma or invasive ductal carcinoma. Galactograms would not be the initial test in the workup of a palpable mass in this young patient. A stereotactic biopsy is performed to obtain a tissue sample after a suspicious mass orcluster of microcalcifications is found on either a diagnostic or screening mammogram. A stereotactic biopsy would not be performed before an ultrasound is ordered in this patient. If a suspicious mass is seen in ultrasound, then an ultrasound-guided biopsy would be the next best course of action in order to exclude a breast carcinoma.

A 23-year-old woman presents with a history of chronic pelvic pain. She attained menarche at 13, and she has had regular periods. She has been experiencing severe pain during menses; the pain has been increasing in frequency. She has never had inter-menstrual bleeding, and she regularly uses barrier contraception. However, she has refrained from intercourse for over 6 months due to the pain it causes her. Her vital signs and general physical examination appear to be normal. A vaginal exam reveals cervical motion tenderness, with reduced mobility and nodularity of the utero-sacral ligaments. Question For a definitive diagnosis, what is the diagnostic test of choice? Answer Choices 1 HSG 2 FSH and LH levels 3 Ultrasonography 4 Laparoscopy 5 Pap smear with endocervical curettage

Explanation Laparoscopy is the correct answer. Laparoscopy is considered the gold standard in diagnosis of endometriosis. Laparoscopy allows visualization of the islands of abnormally implanted endometrial tissue. However, visual inspection alone has a high false positive rate. Confirmation requires biopsy of the abnormal areas, with histopathology demonstrating endometrial epithelium, glands, stroma, or hemosiderin-laden macrophages. Hysterosalpingography is incorrect. A visualization of the uterine cavity and tubes by this technique is useful in delineating anatomic defects, such as a bicornuate uterus, as well as mechanical obstruction by adhesion or stenosis as seen post pelvic inflammatory disease. It is not of any use in endometriosis. FSH and LH levels is an incorrect response. They may be used in the workup of infertility, but they provide no information on endometriosis. Ultrasonography is incorrect. In general, imaging techniques are not particularly helpful in diagnosis. It is useful in ruling out other pelvic pathology. Pap smear with endocervical curettage is incorrect. It is mostly a screening procedure for cervical and endocervical pathology. It can help in the identification of endometrial dysplasia and malignancy, but not endometriosis.

A 32-year-old healthy woman is asking for advice about contraception. She wants contraception that is both effective and convenient in terms of application. She thinks that oral contraceptive pills will work the best, and she is interested in additional benefits that the pill can provide (particularly cancer protection). Question You inform her that, in her case, oral contraceptives may reduce the risk of death from what condition? Answer Choices 1 Breast cancer 2 Ovarian cancer 3 Colon cancer 4 Cervical cancer 5 Hepatic cancer

Explanation Ovarian cancers are often diagnosed in the advanced stages; this is the reason why ovarian cancer accounts for more deaths than any other gynecological cancer. The risk of ovarian cancer is reduced by 30% with pill use for <4 years, by 60% with use for 5 - 11 years, and by 80% after 12 or more years of use. Therefore, protection against ovarian cancer reduces the risk of gynecological cancer death. The relationship between breast cancer and OCP is controversial. Some research finds no increased risk of breast cancer in current or former users of OCPs aged 35 - 64, in women with a family history of breast cancer, and in women who started oral contraceptive use at a young age. Other research has found a very low risk of breast cancer in women of childbearing age (relative risk of 1.1 - 1.2, depending on other variables). This small increase is not substantially affected by duration of use, dose, type of component, age at 1st use, or parity. Even 10 years after the discontinuation of oral contraceptive use, no difference in breast cancer incidence between past users and those who have never used oral contraceptives has been found. Breast cancers diagnosed in women who have used oral contraceptives are more likely to be localized to the breast; therefore, overall, there is no significant difference in the cumulative risk of breast cancer between those who have used oral contraceptives and those who have never used them. There also is no evidence that OCP use offers protection. There is no evidence that oral contraceptive pills have any effect on colon cancer. Combined OCP use does not decrease the risk of cervical cancer; it increases it by about 2-fold, but only in long-term users (>5 years) with persistent human papilloma virus infection. Combined OCP use does not decrease the risk of hepatic cancer. Rarely, it has been associated with the development of benign or malignant hepatic tumors (adenoma and hepatocellular carcinoma). The risk increases with higher dosage, longer duration of use, and older age. Current estimates indicate there is about 2 times the risk of liver cancer after 4 - 8 years of use; therefore, protection by OCPs against ovarian cancer reduces the risk of gynecological cancer death.

A 32-year-old woman presents for a routine gynecological exam. She has been married for 5 years and plans to start a family with her husband in the near future. Her first menstrual period was at the age of 11. She is in a 28/4 cycle with no irregularities or discomfort. At age 18, she had an induced abortion and has used oral contraceptives since then. In the recto-vaginal exam, you palpate a mass on the left side. An ultrasound confirms a complex cystic tumor 6cm in diameter on the left ovary. There is no free fluid in the pelvis. There is no family history of any malignant tumors. What is your therapeutic approach? Answer Choices 1 Laparoscopy and cystectomy 2 Ultrasound examination after the next period 3 Hormonal treatment with progesterone 4 Pelvic exam in 6 months 5 Ovariectomy

Explanation Since this patient is pre-menopausal, with no family history and the tumor is cystic, unilateral, asymptomatic, and there is no ascites, the possibility of it being malignant is very small. Most of these cysts regress within 2 menstruation cycles; therefore, ultrasonic control after the next period is a legitimate approach. If, after 2 cycles, the cyst persists or increases in size, laparoscopy and cystectomy would be indicated. Since the patient expressed the wish for children, it is very important to try leaving the ovary intact. Hormonal treatment with progesterone has not shown any advantage in retrogression of ovarian cysts. Even with no indication of a malignancy, a control period of 6 months is too long and should always include a sonogram. An ovariectomy should only be performed if laparoscopy shows a highly suspicious result of the tumor being malignant.

A 28-year-old woman in her 2nd trimester of pregnancy presents with a 3.5 cm painless mass on her left breast. She has no other associated symptoms. She has a positive family history of breast cancer. She is afebrile; pulse is 80/min, and BP is 110/72 mmHg. On examination of the breasts, you notice engorgement and hypertrophy. The mass appears solid on ultrasonography (USG). Question What is the next step in evaluation of this mass? Answer Choices 1 Magnetic Resonance Imaging (MRI) 2 Percutaneous biopsy under local anesthesia 3 Fine needle aspiration cytology (FNAC) 4 Sentinel lymph node sampling using blue dye 5 Avoid any intervention at this time

Explanation The correct answer is percutaneous biopsy under local anesthesia. This is used to obtain a histological diagnosis, which can be done safely during pregnancy with a sensitivity of around 90%1. During pregnancy, non-ionizing examinations are preferred to those needing ionizing agents1. Breast ultrasonography is the first diagnostic instrument used by clinicians when a breast mass and the axillary area need to be assessed in a pregnant woman; it is non-ionizing and has high sensitivity and specificity1. MRI is incorrect; it should only be used when it will alter clinical decision making and when ultrasonography is inadequate1. No well-designed studies of the efficacy and safety of MRI of the breast during pregnancy have been reported, and results of some studies have shown that gadolinium-based MRI contrast agents pass through the placental barrier and enter fetal circulation1. Fine needle aspiration cytology (FNAC) should be done only in case of a cystic mass1. Gestational and puerperal hormones induce physiological hyperproliferative changes of the breast, which could lead to a false positive or false negative result with FNAC1. Therefore, this procedure is not recommended during pregnancy1. Staging with sentinel lymph node biopsy is done once the diagnosis of breast cancer is established. Clinicians can safely use sentinel lymph node staging during pregnancy using technetium 991. However, blue dye is associated with a risk of an anaphylactic maternal reaction, and it would probably distress the fetus; therefore, the use of blue dye should be avoided during pregnancy. Although sensitivity and specificity of sentinel lymph node biopsies during pregnancy have not been established, researchers have successfully used technetium-based identification in pregnant women1. Staging examinations and tumor biology assessment affect management decisions in pregnancy and should not be avoided.

An obese woman pregnant with twins is concerned about gaining too much weight. How do you counsel her regarding the Institute of Medicine's (IOM) recommended weight gain in women carrying twins? Answer Choices 1 A 25 to 42 pound weight gain is recommended. 2 A 37 to 54 pound weight gain is recommended. 3 An 11 to 20 pound weight gain is recommended. 4 She should maintain her current pre-pregnancy weight as much as possible. 5 A 25 to 35 pound weight gain is recommended.

Explanation The correct response is a 25 to 42 pound weight gain is recommended. The IOM did not distinguish different weight recommendations for women carrying twins from those for women pregnant with a singleton until recently. According to IOM, women pregnant with twins should attempt to gain between 37 - 54 pounds. A woman with a normal BMI (18.5 to 24.9) carrying only 1 baby should attempt to gain 25 to 35 pounds in pregnancy. IOM recommends that pregnant women with a low BMI (< 18.5) should attempt to gain 28 to 40 pounds if carrying 1 infant; overweight women with a high BMI (25 to 29.9) should attempt gaining 15 to 25 pounds, or 31 - 50 pounds if carrying twins; and obese women (BMI > 29.9) should gain 25 - 42 pounds if carrying twins, and only 11 - 20 pounds if pregnant with a singleton.

A 17-year-old girl presents with a 1-month history of vaginal discharge. The discharge is described as thin, greenish-yellow, and malodorous. She also admits to vaginal irritation and postcoital bleeding. She currently has 1 sexual partner and denies the use of barrier contraceptive methods. Speculum exam reveals erythematous vaginal mucosa, purulent, thin, frothy discharge, and a 'strawberry cervix'. Question Based on the patient history and physical examination findings, what is your most likely diagnosis? Answer Choices 1 Chronic cervicitis caused by Neisseria gonorrhea 2 Chronic cervicitis caused by Mobiluncus curtisii 3 Acute cervicitis caused by Trichomoniasis vaginalis 4 Acute cervicitis caused by Gardnerella vaginalis 5 Foreign body cervicitis

Explanation The correct response is acute cervicitis caused by Trichomoniasis vaginalis. This patient has the classic findings associated with an acute trichomoniasis infection, which are purulent, malodorous, thin, frothy discharge with vaginal irritation, and punctate hemorrhages on the cervix (strawberry cervix). The case does not support the other diagnoses. Chronic cervicitis is typically noninfectious, and cervicitis caused by Gardnerella vaginalis is not associated with purulent discharge or strawberry cervix; no foreign body was identified on speculum exam, which would support a diagnosis of foreign body cervicitis.

A 29-year-old woman inquires when she should have her first mammogram. Her family history is significant for the death of her mother and grandmother due to breast cancer at the age of 32 years. Presently, the woman is asymptomatic. What is the recommendation for mammography for this patient? Answer Choices 1 Immediately 2 When she is 30 years old 3 When she is 32 years old 4 When she is 34 years old 5 When she is 36 years old

Explanation The correct response is immediately. Breast cancer is recognized as both an increasingly common disorder and a potentially hereditary disorder. Furthermore, early detection will provide a woman with her best chance at a cure of breast cancer. Given the patient's genetic burden of having both her mother and her grandmother die of metastatic breast cancer at the relatively young age of 32 years, it is reasonable to provide her with immediate and periodic mammography combined with instruction in breast self-examination. Women at high risk require a more aggressive schedule, frequent examinations, and other procedures such as ultrasound and MRI. The American Cancer Society (ACS) recommends annual mammography beginning at age 40 in average risk women in the absence of any positive family history. The American College of Obstetricians and Gynecologists (ACOG) and the American Medical Association (AMA) recommend mammography every 1 - 2 years in women age 40 to 49 years, and annually after the age of 50. Screening mammography annually or biennially significantly reduces the mortality from breast cancer. A prudent provider will recognize and explain that in average risk individuals, the risk of a false positive mammogram increases as the age of the patient decreases.

Your patient in the labor and delivery unit is a 26-year-old primigravida; she is at 36 weeks of gestation. She is experiencing fever and abdominal tenderness. Her pregnancy has been normal; however, 2 days ago she noticed clear fluid vaginal leaking. There are no changes in fetal movements, and there has been no vaginal bleeding. Today her temperature is 38.5 C, blood pressure is 120/80 mm Hg, heart rate is 92/min, and respirations are 18/min. Fetal heart rate is 160/min (tachycardia), with good variability. Contractions are registered every 10 minutes. On examination her abdomen is diffusely tender, the cervix is closed, and there is turbid vaginal pooling that shows an elevated pH and a ferning pattern. Question After administering antibiotics, what will be your next step in the management of this patient? Answer Choices 1 Cesarean section 2 Oxytocin 3 Magnesium sulfate 4 Indomethacin 5 Steroids

Explanation The correct response is oxytocin. Your patient most probably has chorioamnionitis secondary to the prolonged rupture of membranes. Delivery is indicated for the benefit of both mother and fetus. Her cervix is closed; therefore, the induction of labor is the best next step in the management. A cesarean section is not indicated in this case; instead, you should try vaginal delivery. You should only consider a cesarean section if there is significant maternal and/or fetal distress and a vaginal delivery is remote. Magnesium sulfate is not indicated; your patient has chorioamnionitis, and you do not want tocolysis. Indomethacin is not indicated; you do not want to delay the labor. Steroids (e.g., betamethasone) are used to improve fetal lung function and are given up to 34 weeks of gestation (the greatest benefit is at less than 32 weeks of gestation). Your patient is at 36 weeks of gestation; therefore, steroids are not indicated.

A 36-year-old woman presents for fertility counseling. She and her husband have been trying to conceive for over 1 year, during which time her periods have been irregular. Her last menses was 4 months ago. Serum hCG is negative. Serum levels of LH and FSH are elevated, and serum estradiol is low. What is the most likely diagnosis? Answer Choices 1 Tubal obstruction 2 Polycystic ovarian disease 3 Endometriosis 4 Premature menopause 5 Panhypopituitarism

Explanation The correct response is premature menopause. Low levels of estradiol associated with high levels of gonadotropic hormones indicate a functioning pituitary, therefore excluding panhypopituitarism, and primary ovarian failure. Average age of menopause in the United States remains at age 52, with a perimenopausal period averaging 5 years. Tubal obstruction, commonly the result of endometriosis, will result in infertility, but it does not affect menstruation.

A 40-year-old Caucasian woman presents with excessive menstrual bleeding, as well as bleeding between her menses; these symptoms have been occurring over her last 6 cycles. She also admits to associated "pressure and fullness" in her pelvis, dysmenorrhea, urinary frequency, and generalized fatigue. She denies being pregnant and has never had children. She denies any weight loss, fever, chills, chest pain, shortness of breath, abdominal pain, early satiety, nausea, vomiting, diarrhea, changes to her urine color or odor, flank pain, hematuria, or dysuria. Her physical exam is remarkable for a "lumpy-bumpy" cobblestone sensation above the symphysis pubis with an enlarged, mobile, and irregular contour upon bimanual palpation; no adnexal masses, cervical motion tenderness, or vaginal discharge is noted. Question What medication would be most beneficial in the initial management of this patient? Answer Choices 1 Leuprorelin (Lupron) 2 Raloxifene (Evista) 3 Clomiphene (Clomid) 4 Danocrine (Danazol) 5 Estradiol (Estraderm)

Explanation This patient's history and physical exam are most consistent with a diagnosis of uterine leiomyoma (fibroid). Pharmacologic inhibition of the growth of fibroids can be achieved by suppression of the hypothalamic-pituitary-ovarian axis through the use of gonadotropin-releasing hormone agonists (GnRH analogs) that decrease estrogen production, thereby reducing fibroid size by as much as 40-60%. It is also commonly used prior to a planned hysterectomy to reduce blood loss. Raloxifene (Evista) may help reduce fibroid growth. However, whether efficacy in reducing symptoms is comparable to that of other drugs is unclear. Clomiphene (Clomid) is indicated in the treatment of infertility, as it stimulates ovulation. Danocrine (Danazol) is an androgenic agonist and can suppress fibroid growth. However, it is not as successful as GnRH analogs and is associated with a high rate of adverse, androgenic effects such as weight gain, acne, hirsutism, edema, hair loss, deepening of the voice, flushing, sweating, and vaginal dryness; it is thus often less acceptable to patients. Estrogen replacement agents are not suitable as treatments for uterine fibroids since uterine leiomyomas are hormonally responsive benign tumors of the uterus that proliferate in response to estrogen.

A 32-year-old woman in her 34th week of pregnancy comes to the labor floor after 2 - 3 hours of vaginal bleeding. Labor and vaginal delivery of her previous 3 pregnancies were normal. She is frightened, but oriented. While taking her medical history, what points are relevant to ask? Answer Choices 1 Cigarette use 2 The latest pap smear result 3 Recent sexual intercourse 4 Desire for future pregnancies 5 Recent vulvar pruritus

Explanation You should ask about recent sexual intercourse. The woman in this case exhibits 3rd-trimester bleeding, which is an obstetric emergency. History, physical exams, and lab test should be performed to assess maternal and fetal well-being. The differential diagnosis consists of placenta previa, abruptio placenta, and local vaginal or cervical bleeding from trauma. In medical history, you should ask about bleeding problems and sexual intercourse, as they can cause cervical trauma. In physical examinations, Leopold's Maneuvers outline the fetal lie and indicate fetal malposition. Postural hypotension suggests major uncompensated blood loss. After the history and physical exams have been completed, measurement of blood loss and urine output should be done. However, a type and cross match is an essential precaution. At this time, amniocentesis is appropriate to look for blood in the amniotic fluid and to determine fetal lung maturity. Internal examinations must not be performed in women suspected of having placenta previa, unless immediate delivery is required. The use of oxytocin challenge test in the face of 3rd-trimester bleeding is contraindicated. The best management for this woman would be careful monitoring of vaginal bleeding. Blood should be set up in the blood bank. Cesarean section may be required later, but not at this point. The use of Ritodrine to inhibit early labor is not indicated. Magnesium sulfate is used to control preeclampsia, however it is thought that magnesium sulfate can worsen bleeding in a woman with placenta previa by preventing blood coagulation.

Which one of the following bacteria is carried by about 20% of women of child-bearing age and is a cause of sepsis and meningitis of the newborn? Answer Choices 1 Lactobacilli species 2 Bacteroides 3 Bifidobacterium 4 Group B streptococci 5 Clostridium

Explanation Group B streptococci are carried by about 20% of women of childbearing age in their vagina and are a cause of sepsis and meningitis of the newborn and are acquired during passage through the birth canal. Lactobacilli, Bacteroides, Bifidobacterium and Clostridium are normally found in the colon and do not cause sepsis and meningitis of the newborn.

A 14-year-old girl had primary amenorrhea, a webbed neck, a broad, shield-like chest with widely spaced nipples, and lack of secondary sexual characteristics. A karyotype of this patient is shown in Figure G3.6. The MOST likely diagnosis is which of the following? Answer Choices 1 Turner syndrome 2 hermaphroditism 3 androgen-insensitivity syndrome 4 Down syndrome 5 Sheehan syndrome

Explanation In classic Turner syndrome (ovarian dysgenesis), there are only 45 chromosomes, 1 sex chromosome being absent (45, X). This syndrome includes bilateral ovarian agenesis (streak ovaries), infantile genitalia, short stature, short webbed neck, high arched palate, cardiovascular and renal defects (such as coarctation of the aorta and horseshoe kidney), and cubitus valgus (an outward deviation of the extended forearm).

Examination of a newborn infant revealed the presence of hairy patch present on the lower back. Neural tube defect is most commonly associated with which of the following? Answer Choices 1 Teratogen exposure during second trimester 2 Radiation exposure during third trimester 3 Maternal infections during the third trimester 4 Increased maternal age at first pregnancy 5 Maternal folic acid deficiency in the first trimester

Explanation Neural tube defects are most commonly associated with maternal folic acid deficiency in the first trimester. Hence, 400mg of folic acid is recommended when attempting to get pregnant and during early pregnancy to prevent neural tube defects. Neural tube defects (NTDs) result due to the failure of closure of neural tube during the 4th week of embryogenesis. NTDs include anencephaly, spina bifida, and encephalocele and may also cause severe disabling birth defects. Congenital malformations of the spinal cord are the result of improper closure of the caudal neuropore. Malformations of the neural tube involving the spinal cord and vertebral arches are termed as spina bifida. The non-fusion of the spine can occur in varying degrees of severity ranging from a lack of bone fusion to protrusion of the meninges with or without nerve involvement. Severe type of spina bifida (meningomyelocele) involves protrusion of the spinal cord along with the meninges through a defect in the vertebral arch. The clinical manifestation vary depending on the level of the lesion; nerve involvement results in paralysis of the legs, urinary and fecal incontinence, skin anesthesia, and abnormalities of the pelvis, knees, and feet. Display of a hairy patch, a dimple, or hyperpigmentation may also be present. Early prenatal diagnosis by ultrasound and alpha-fetoprotein level estimation is essential for proper management. Though maternal folic acid deficiency plays a pivotal role in NTDs, the etiology of neural tube defect remains multifactorial. Maternal infections, radiation, and teratogen exposure predominantly in the early pregnancy are associated with NTDs. Increased maternal age is associated with Down syndrome.

A 16-year-old girl has had recent onset of painful menstrual periods. Age of menarche was at 12 years. Her periods were irregular for about 8-10 months, but have been fairly regular since then, with occasional mild crampy pains on the first 2 days of her period. She rates the pains now as being 8-9 on a scale of 10. Cramps will start about a half a day before the onset of her periods, worsen the following day, and then gradually subside over the next day. There is no change with bowel movements. She also describes having a headache, looser stools, mild nausea, and low back and thigh pain. Bleeding is moderate on the first 2 days. She is otherwise healthy without history of abdominal surgery. Privately she denies any sexual activity. She is an average student and has missed 1-2 days of school with each period for the past 3 months. She also denies any fever or dysuria. There is also no family history of gynecological problems. On exam she has diffuse midline lower abdominal tenderness with some mild soreness to lower back and thighs, bowel sounds are normal, and there is no rebound tenderness. An external genital exam is normal. Based on this information, which of the following is the most likely cause of this girl's dysmenorrhea? Answer Choices 1 Primary dysmenorrhea 2 Endometriosis 3 Irritable bowel syndrome 4 Imperforate hymen 5 Ovarian cysts

Explanation Primary dysmenorrhea is associated with ovulatory cycles and typically occurs at an earlier age than secondary dysmenorrhea, as 1/3 of adolescents continue to experience anovulatory cycles in the first few years after menarche due to the time required for the hypothalamic-pituitary-ovarian cycle to mature. Disability is common with a reported 14% of girls reporting missing school or work because of pain and nearly 50% of those have pain described as moderate or severe. The physiologic basis relates to cell membrane phospholipids, endomyometrial prostaglandins, and leukotrienes. Defining symptoms include crampy midline lower abdominal pain beginning with menstrual flow or a short time before. As in this patient, cramps are typically intense on the first or second day and resolve before the end of the menstrual flow. The pain may be referred to the lower back and anterior thigh. Nausea or vomiting may occur as well as near-syncope or dizziness and complaints of weakness. Breast tenderness, bloating, headache, and mood changes can also be seen. Management involves the use of NSAIDs, which reduce the production of prostaglandins. Oral contraceptives should be considered for teens that have not experienced sufficient relief of symptoms with NSAIDs. Endometriosis occurs when functioning ectopic endometrial glands and stroma are present outside the uterine cavity, causing pain, irregular bleeding, and frequently infertility, though it may also be asymptomatic. The pelvic pain often cycles with menstruation and is associated with dysmenorrhea, dyspareunia, and infertility. The pain can be a deep constant ache with bilateral patterns of distribution and radiate to the buttock and perianal region. Pain may be associated with bladder or bowel function and there can be menstrual spotting. There is usually nonspecific pelvic tenderness on exam. Unlike this patient, it occurs more frequently in first-degree relatives of women whose endometriosis has been confirmed surgically. The pain of irritable bowel syndrome in adolescents is similar to that described in adults, with paroxysmal periumbilical abdominal pain that, unlike in this patient, is relieved by defecation or associated with an irregular pattern of defecation, including a change in frequency or consistency of stool, straining or urgency, feeling of incomplete evacuation, passage of mucus, and bloating or abdominal distention. Left lower quadrant fullness over the left colon may also be present. Periods of diarrhea alternate with periods of constipation. Imperforate hymen is recognized as a membrane covering the vaginal opening seen on external exam not evident in this patient. This may be bulging or a have a bluish hue from blood retained. There is a history of cyclic abdominal pain and a midline abdominal mass. A large enough mass may cause urinary obstruction. Ovarian cysts are fluid or semisolid filled sacs that develop on or within the ovary and are typically functional and disappear on their own. These develop from follicles that fail to rupture and release an egg and instead of being reabsorbed, the fluid within the follicle persists and forms a cyst. This may cause a sudden onset of pain that is constant or intermittent, dull or sharp aching pelvic pain that occurs, unlike the pattern seen in this patient, shortly after beginning or ending a menstrual period. There may be abnormal uterine bleeding, either longer, shorter or absent, abdominal bloating or distention, or no symptoms. Large or persistent cysts may require surgical removal.

A 20-year-old female (G1, P0) presents to your office for her 36th week antenatal care visit. The woman's blood group is O, Rh negative. Her husband's blood type is B, Rh positive. The fetal ultrasound showed mild enlargement in the cardiac chambers and pericardial effusion. The spectrophotometric analysis of the amniotic fluid sample showed increased absorbance at 450nm. The most likely mechanism regarding these findings is Answer Choices 1 Cardiac malformation 2 Hemolytic disease in the fetus 3 Hepatic dysfunction in the fetus 4 Hypoglycemia in the fetus 5 Polycystic kidney

Explanation The fact that the mother is Rh negative and that the father is Rh positive means that the likelihood that the mother will be carrying an Rh positive baby can either 0% or 50% depending on whether the father is heterozygous or homozygous for the Rh gene locus. The antenatal screening of the mother at 36 weeks gestation showed dilatation of the cardiac chambers and pericardial effusion in the fetus, which are signs of heart failure. Since the mother is Rh negative and has likelihood of 50% of carrying an Rh positive baby, hemolytic disease of the newborn should be on the top of your list while working up the differential diagnosis in this case. The presence of hyperbilirubinemia in the amniotic fluid as detected by spectrophotometry also should steer you towards hemolytic disease in the fetus due to Rh incompatibility with alloimmunization of the mother. It is a common misunderstanding that the first pregnancy should be safe in an RH neg. mother carrying and RH pos. fetus. Feto-maternal transfusion on a small scale can occur during pregnancy and depending on how much has been exchanged between the maternal and the fetal circulation, RH antibodies can be found in third trimester of primigravidas, that is why re-screening for RH antibodies around the 28th week is recommended. The heart failure and jaundice are not signs of hypoglycemia in the fetus and the mother is not diabetic making this possibility highly unlikely. The ultrasonic evaluation of the fetus showed only mild signs of heart failure and did not indicate the presence of cardiac malformation in the fetus. In addition the Rh-negative status of the mother should alert you to the possibility of an Rh incompatibility specially knowing that the husband is Rh positive. Hepatic dysfunction also will be associated with increased bilirubin in the amniotic fluid but will not explain the cardiac findings. The ultrasound examination would have detected the polycystic kidneys if it were present.

You are the resident on call in the ER when a multiparous patient presents in the first stage of labor, with contractions less than one minute apart. The chief resident calls you to assist in the delivery of the child and states that "the fetal head is engaged." In reference to this phrase, which of the following is true? Answer Choices 1 the widest portion of the fetal head has successfully passed through the pelvic inlet 2 forceps will need to be used because the widest portion of the fetal head is engaged at the pelvic inlet 3 in a multiple birth, the head of the first fetus is not presenting first, but is positioned correctly 4 the fetus is presenting in a frank breech fashion with the head positioned correctly in the pelvis 5 the crown of the head of the fetus is seen to bulge the perineum indicating birth is imminent

Explanation When the fetal head is engaged, the widest portion of the fetal head has successfully passed through the pelvic inlet. This phrase signals that the delivery of the infant is proceeding normally, that the presentation is head first, and is not a breech presentation. After this stage, with the complete dilation and effacement of the cervix, the child will descend down the birth canal. Only at that time would the fetal head be noted to be crowning, e.g., presenting as a bulge at the perineum.

You are providing care to a woman who is at 33 weeks gestation. Her pregnancy is complicated by gestational diabetes. She is being provided education by the dietician, and has weekly obstetrical appointments. What fasting blood sugar (FBS) readings should necessitate switching from diet control to insulin therapy? A FBS > 70 mg/dL B FBS > 90 mg/dL C FBS > 95 mg/dL D FBS > 110 mg/dL E FBS > 126 mg/dL

FBS >95 mg/dL Gestational diabetes has different risks associated with it, in particular for the fetus. Stricter glycemic controls are recommended for pregnant versus non pregnant women by the ACOG and the ADA.

A 15-year-old girl presents due to primary amenorrhea. She was always healthy, does not take any medications, tobacco, or drugs, and denies intercourse. She has never experienced cyclic abdominal and/or pelvic pain. Her family history is negative for gynecologic or fertility problems, autoimmune diseases, and endocrinopathies; her mother's and female relatives' menarche presented in the ages from 12-14 years. Her breasts are absent. The rest of your examination is normal. The presence of her uterus is demonstrated by ultrasound. Question What should you obtain as your next step in diagnosis of this patient? Answer Choices 1 Estrogen levels 2 Folliculostimulating hormone (FSH) levels 3 Testosterone levels 4 Pregnancy test 5 Progesterone challenge test

FSH Explanation Primary amenorrhea is the absence of menstruation by the age of 14 if the patient has no secondary sexual characteristics, or by the age of 16 if the patient has secondary sexual characteristics present. The absence of breasts indicates inadequate estrogen production. The initial workup includes a pregnancy test and serum luteinizing hormone, follicle-stimulating hormone, prolactin, and thyroid-stimulating hormone levels. If history or examination suggests a hyperandrogenic state, serum free and total testosterone and dehydroepiandrosterone sulfate concentrations are useful. FSH levels will show where the lesion is. If there are no functional ovaries, FSH will be high; and if there is no FSH, ovaries will not be stimulated. The next step should be obtaining the karyotype. The lack of X chromosome, which is necessary for developing ovarial follicles, will point to the gonadal dysgenesis, and low FSH will point to hypothalamic dysfunction (normal ovaries are not stimulated to produce estrogen). Autoimmune oophoritis with anti-ovarian antibodies should be considered when previous tests are normal. Patients with autoimmune oophoritis are at risk for the development of adrenal insufficiency and other autoimmune endocrinopathies (thyroid and parathyroid, diabetes mellitus, myasthenia gravis, pernicious anemia, etc). There is no need to obtain estrogen levels; having no breasts, your patient most probably has no estrogen. The main question is why she does not have estrogen. FSH and karyotyping will help in the diagnosis.Testosterone levels should be obtained in the patient who has breasts but has no uterus in order to decide where the estrogen originates from; if testosterone levels are that of a normal female and ovaries are present, then the patient might have Mullerian agenesis (she also will have normal pubic and axillary hair and normal karyotype). If testosterone levels are that of a normal male, the estrogen source is probably the testes, as in androgen insensitivity syndrome, when a psychologically and physically female patient (with karyotype 46, XY) presents with primary amenorrhea with the lack of pubic and axillary hair and absent uterus. A pregnancy test is indicated in secondary amenorrhea. In the case of primary amenorrhea strongly suggestive for anatomical lesion or hormonal dysfunction, pregnancy test is not indicated as a first diagnostic step. Progesterone challenge test is usually performed in the case of secondary amenorrhea when other causes are excluded (pregnancy, hypothyroidism, prolactinoma, medications). Positive test is when a single dose of progesterone or 7 days of oral medroxyprogesterone causes withdrawal bleeding, as in anovulatory cycles. Negative test will demand further evaluation with estrogen-progesterone challenge (21 days of estrogen followed by 7 days of progesterone). Negative test will prompt endometrial problems (scars, adhesions) or outflow obstruction from other cause. Withdrawal bleeding will reveal inadequate estrogen levels, and your next step will be to find out why. This can be done by obtaining FSH levels.

An 18-year-old woman presents for advice regarding emergency contraception. She had sexual intercourse the day prior to presentation; she did not use contraception, and she wants to know if it is possible that she is pregnant. Her last menstrual period (LMP) began 5 days prior to presentation. She usually gets her menses once in 28 days and the period usually lasts for 5 days. The physical exam is normal. Question What hormone is responsible for the stimulation of the ovaries at this point of the menstrual period? Answer Choices 1 Estrogen 2 Follicle-stimulating hormone (FSH) 3 Inhibins 4 Luteinizing hormone (LH) 5 Progesterone

FSH Explanation The 5th day of the menstrual period corresponds to the follicular phase of the cycle, which starts from the 1st day of bleeding to the day of the LH peak. In this phase, the FSH, which is produced by the pituitary, stimulates the development of follicles in the ovaries; only 1 follicle is dominant at the end. During the 2nd part of this phase, the granulosa cells of the follicle begin to produce estrogen. Estrogen is produced by the granulosa cells of the follicle; they are very important in the follicular phase. Their level is very low at the beginning of this phase and reaches its maximal point at the middle of the cycle, just before the the LH peak. The cells are the feedback for the LH production, and in part, for the FSH production. Inhibins are also hormones produced by granulosa cells of the follicles; they are part of the negative feedback for the FSH. Inhibin B levels rise during the luteal-follicular transition, are highest during the mid follicular phase, go up again during the LH peak, and finally decrease in the late follicular phase. Inhibins A levels decrease during the late luteal phase. Ovulation occurs approximately at the middle of the menstrual cycle (day 14). At the end of the follicular phase a rise in the level of estrogens takes place, preceding the LH peak. Ovulation will occur an average of 30 hours after this peak. The 2nd 1/2 of the menstrual cycle corresponds to the luteal phase of the menstrual cycle which starts from the day of ovulation to the 1st day of menstruation. In this phase, the ovarian corpus luteum begins to produce progesterone. If there is fertilization of the ovum, the production of progesterone continues for 4 to 5 weeks until the placenta can produce a sufficient quantity of the hormone to maintain the pregnancy. If there is no fertilization, progesterone levels decrease and menstruation occurs.

A 20-year-old Lebanese woman presents to a family practice office because she wants to start birth control. She has never been sexually active, and she is engaged to be married in 2 months. She feels well and has no complaints. She thinks she wants "the pill". Her fiancé is also a virgin, and they are not interested in condoms or other barrier methods of contraception. She wants to delay childbearing for at least 2 years. A summary of her past medical history includes: Medications: occasional over-the-counter ibuprofen for menstrual cramps and headaches Allergies: Penicillin Surgical history: Tonsillectomy Medical history: No known conditions. OB/GYN history: Menarche age 12. Regular monthly menses, with mild-moderate dysmenorrhea. Family history: Patient's older sister had a blood clot in her lung that followed delivery of a child. Her paternal grandfather has diabetes and hypertension. Her maternal grandmother had a stroke. Her mother had a deep venous thrombosis in a leg, and her maternal grandfather had prostate cancer. Social history: The patient works as a waitress part-time and is attending college. She currently lives in a dorm. She denies the use of tobacco, alcohol, and recreational drugs. Vitals are obtained from your medical assistant prior to physical exam. Weight 123 lbs. Height 64" Body mass index (BMI) 21.1 Pulse 88 Blood pressure 134/86 Temperature 97.9 °F Question Based upon this patient's presentation, what test should be ordered before prescribing birth control pills? Answer Choices 1 Bleeding time 2 Complete blood count (CBC) 3 Complete metabolic panel (CMP) 4 Factor V Leiden 5 Pap smear

Factor V Leiden Explanation Generally, no screening tests are recommended prior to starting most patients on most forms of contraceptives. However, when risk factors suggest a possible contraindication, the clinician should delve deeper and individualize testing. In this case, the patient is healthy, but her family history is concerning for a heritable form of hypercoagulability. She should be tested for a Factor V Leiden mutation. Factor V Leiden mutations are estimated to be present in approximately 5% of the Caucasian population, and it is highest in those of Lebanese ethnicity, with a prevalence of over 14% of the affected population. This mutation is the most common of heritable causes of hypercoagulability. If the patient tests positive, then she should not use estrogens in either contraceptives or menopausal hormone therapy, due to increased risk of thromboembolism. This patient should be offered alternate methods of contraception. Bleeding time is a test of platelet function, not coagulation factors. Bleeding time is useful in evaluation of patients at risk of bleeding too much, not clotting. A complete blood count (CBC) is useful in evaluating several conditions, such as anemia and infection and alterations of platelet numbers. If the clinician ordered a CBC and it returned normal, it could provide false reassurance that this patient could safely use combination hormonal contraceptives; the CBC can be entirely normal in an individual with a Factor V Leiden mutation. There is another condition of inherited hypercoagulability, familial or essential thrombocytosis, which would show up on the CBC, but this condition is very rare; also, it often presents at birth, so this patient is unlikely to have it. A complete metabolic panel (CMP) provides information on liver and kidney function, glucose levels, and electrolytes. It does not provide information related to this patient's family history of hypercoagulability. Theoretically, the CMP could be done to ensure normal liver enzymes because many of the hormonal contraceptives are hepatically metabolized. However, routine testing of liver function is not recommended prior to starting birth control. The Pap smear (short for Papanicolaou) is a test for cervical cancer and pre-cancerous dysplasia. Many years ago, it was thought that birth control increased risk of cervical cancer. That myth has been disproved. There is no compelling reason to obtain a Pap smear in order to initiate birth control. Routine cervical cancer screening guidelines should be followed, which would indicate that this patient should wait until the age of 21 years before beginning Pap testing.

A 26-year-old female presents to the clinic for evaluation of her inability to conceive for 12 months. She menstruates monthly. Her past medical history is significant for PID x 2 and mild asthma. Her GC, Chlamydia, and pap smear are negative. Her TSH is 3.0 and her UCG today is negative. What is the most likely cause for her inability to conceive? A Anovulatory cycles B Hypothalamic state C Fallopian tube scarring D Hypothyroidism E Polycystic ovarian syndrome

Fallopian tube scarring C In a young and otherwise healthy female with a negative work up and a history of PID, the most common cause of infertility is fallopian tube scarring secondary to STD and PID.

A 29-year-old woman, G0P0, and her husband present to your office after 1 year of infertility. Their histories elicit irregular menses in the woman and that the man is an avid cyclist. Apart from semen analysis in the man, what laboratory and/or diagnostic studies should be pursued in the woman to further evaluate this couple's infertility? Answer Choices 1 Fasting prolactin, TSH 2 TSH, LH, T3, T4 3 Fasting prolactin, FSH 4 FSH, T3, T4 5 Fasting prolactin, TSH, FSH, LH, hysterosalpingogram

Fasting prolactin, TSH, FSH, LH, hysterosalpingogram Explanation Fasting prolactin, TSH, FSH, and LH should all be obtained in the female patient as her irregular menses more or less indicate a high probability of oligomenorrhea. It is also important to evaluate by hysterosalpingogram to look for evidence of uterine and/or tubal defects. It is also important to evaluate the male patient's semen to investigate male infertility related to oligospermia or hypomotility (1).

A 32-year-old, vegetarian female in mid-pregnancy complains of lack of energy and says she becomes easily fatigued. Upon any strenuous movement, her heart pounds rapidly and she becomes short of breath. Nutritional supplement of what mineral may alleviate the female's symptoms? Answer Choices 1 Mg2+ 2 Ca2+ 3 Fe2+ 4 Cu2+ 5 Zn2+

Fe2+ Explanation Pregnant females can experience iron deficiency anemia due to increased demands on their blood. Oxygen (O2) in the lungs binds to the iron ion, Fe2+ while complexed with the heme cofactor of hemoglobin in red blood cells. Iron supplements or foods with abundant iron, such as liver, lean meats, or vegetarian alternatives such as spinach, carrots, and raisins can alleviate anemic symptoms. The other minerals have biological roles but are not associated with anemic symptoms. Magnesium (Mg2+) coordinates with the negatively charged backbone of DNA and interacts with neurotransmitter receptors at excitatory synapses in the central nervous system. Mg2+ deficiency affects the nervous system, resulting in vasodilation, tremors, and depression. Calcium phosphate forms a hard material in bone and teeth. In addition, Ca2+ is a ubiquitous second messenger ion in cellular signaling coupled to G-protein signaling, hormone signaling, and ion channel activity. Ca2+ deficiency can give rise to muscle twitching or cramping and cardiac arrhythmias. Copper (Cu2+) participates in bone and blood formation and is an electron carrier in mitochondrial electron transfer proteins. Cu2+ deficiency is uncommon since the trace amounts needed are satisfied by most diets. Zinc (Zn2+) is a cofactor of many DNA and RNA binding proteins, including many transcription factors. Severe zinc deficiency can retard growth in children, can cause low sperm count in males, and can slow wound healing.

A nulliparous 25-year-old woman has had an uneventful, spontaneous labor at term. She has an epidural anesthetic and has been in the second stage of labor for 2 hours. The fetal heart tones show some signs of deterioration. You decide to expedite the delivery with the use of forceps. What criterion defines the application of low forceps? Answer Choices 1 Fetal head is on the perineum 2 Fetal skull has reached the pelvic floor 3 Fetal skull is at station +2, but not on the pelvic floor 4 Head is engaged, but above station +2 5 Scalp is visible at the introitus without manually separating the labia

Fetal skull is at station +2, but not on the pelvic floor Explanation Low forceps is defined by the skull being at station +2 or greater, but not on the pelvic floor. Outlet forceps includes situations where the scalp is visible, the fetal skull has reached the pelvic floor, or the head is on the perineum. In addition, rotation must not exceed 45 degrees. Midforceps is the head engaged but above a station of +2.

A 22-year-old female present complaining of a lump in her left breast. She noticed it two days ago while taking a shower. She is a non-smoker and has three to four drinks per week. PMH is negative and FMH is negative. On physical exam, vitals are normal, and a breast exam reveals a 1-cm discrete, soft, and rubbery lesion in the upper outer quadrant—it is non-tender, and the remainder of the breast exam is normal. What is the most likely diagnosis? A Abscess B Fibroadenoma C Phylloides tumor D Cyst E Carcinoma

Fibroadenoma B The most common mass in premenopausal women by far is a fibroadenoma. While any lesion needs to be followed, soft, mobile, non-tender, and small lesions in young women without family history are very characteristic of fibroadenoma.

A 24-year-old G1P1 presents to the office complaining of a red, tender area of her right breast. She is four weeks postpartum and is nursing her infant with good success. She complains of no other symptoms. On physical exam, her vitals are normal. Lungs CTA, CV RRR, left breast is normal, right breast has a 3-cm area that is warm with erythema, and no mass or area of fluctuance is noted. She has a MRSA mastitis. How did she most likely contract the infection? A Community acquired B Hospital acquired C From her infant D Self inoculated

From her infant Infants usually contract MRSA due to poor hand washing technique from the hospital staff, but it is then spread to the mother via the infant.

A 25-year-old female presents for an ultrasound after having a positive home pregnancy test. She has an unremarkable past medical history and physical exam. She states she has been feeling fine without any abdominal discomfort or vaginal bleeding noted. On ultrasound you determine she is 10 weeks pregnant. You note a noncomplex unilateral mass on her left ovary measuring 2 cm in diameter. What is the most likely diagnosis? A Benign cystic teratoma B Serous cystadenoma C Functional ovarian cyst D Hemorrhagic ovarian cyst E Torsion of the adnexa

Functional Ovarian cyst C More than 90% of unilateral, noncomplex masses that are measured to be less than 5 centimeters in diameter that are identified in the first trimester are functional ovarian cysts. Benign cystic teratomas (A) make up 21% of pathologic ovarian neoplasms and serous cystadenomas (B) make up 21%. A hemorrhagic cyst (D) is a non-functional cyst. Torsion of the adnexa (E) is most commonly seen between 6-14 weeks gestation or in immediate puerperium but symptoms include abdominal pain and tenderness.

You are examining the wet prep of a young woman who presented complaining of vaginal discharge. You see 15 to 20 WBCs, 2+ bacteria, and clues cells. The KOH prep is whiff positive. What type of vaginitis does this indicate? A Yeast vaginitis B Viral vaginitis C Gardnerella vaginitis D Trichomonas vaginitis E Atrophic vaginitis

Gardnerella vaginitis Gardnerella produces a positive amine test when the preparation is mixed with KOH. Viral Vaginitis, yeast, and Trichomonas do not produce a positive amine smell when mixed with KOH.

Your patient is a full-term newborn with facial defects affecting her eyes, nose, and upper lips. She is the first child of non-consanguineous parents. Her mother has a history of gestational diabetes, which began at the start of her pregnancy. She contracted German measles a month before the delivery. During the course of infection, her self-prescribed daily treatment was 3 tablets of aspirin, at least 6 cups of herbal tea, and a double dose of folic acid. Imaging studies showed prosencephaly. Question What risk factor is responsible for the condition of this child? Answer Choices 1 Rubella 2 Aspirin 3 Herbal tea 4 Gestational diabetes 5 Folic acid

Gestational diabetes Explanation The only possible risk factors during the period of the development of face and forebrain is gestational diabetes, because the forebrain is formed and the face begins to develop in the fifth and sixth weeks of human pregnancy. Other known risk factors for prosencephaly include transplacental infections, bleeding during the first trimester, a history of miscarriages, use of some drugs potentially unsafe in pregnancy (insulin, birth control pills, lithium, anticonvulsants, retinoic acid, cholesterol-lowering agents, and maternal hypocholesterolemia), and toxins (alcohol, nicotine). Rubella, or German measles, is not likely the cause of birth defects because infection occurred after organogenesis was finished. Rubella can cause miscarriage, stillbirth, or birth defects (most often deafness, brain damage, heart defects, and cataracts) if infection occurs during the first 16 weeks of pregnancy. Aspirin taken during the last trimester may increase the risk of bleeding in the newborn, but cannot cause birth defects. Herbal tea is not the cause of birth defects after organogenesis is finished. All women capable of pregnancy should take a daily vitamin supplement of folic acid to prevent neural tube defects. In the late pregnancy, however, folic acid can neither prevent nor cause birth defects of the brain and face.

Some temporary disease processes are unique to pregnancy and may occur in one or all of a women's pregnancies. Which of the following would raise suspicion for the first phase of toxemia referred to as pre-eclampsia? Answer Choices 1 Headache, visual disturbances, abdominal pain, vomiting 2 Sinus pain, chest pain, bowel disturbances 3 Chest pain, shortness of breath, pain in arms 4 Bowel disturbances, pain in arms, pain in back 5 Face swelling, edema of the limbs, renal failure

Headache, visual disturbances, abdominal pain, vomiting Explanation Pre-eclampsia, the first stage of toxemia, exhibits the symptoms of headache, visual disturbances, abdominal pain, and vomiting. These are often overlooked as normal pregnancy symptoms. The patient should be monitored for further symptoms. Chest pain, shortness of breath, and pain in the arms would raise the suspicion of cardiac problems and should not be ignored.

You are treating a 14-year-old female for gonorrhea. She has a past history of Chlamydia. Her LMP was 3 weeks ago. She is allergic to penicillin. Her only medication is oral combination birth control pills. Why should Levofloxin be avoided in this patient? A Her age B Her LMP status C Her history of Chlamydia D Her oral contraceptives E Her penicillin allergy

Her Age Fluoroquinolones are contraindicated in pediatric patients. The risk-benefit assessment indicates that levaquin is only appropriate in pediatric patients at least 6 months of age, for treatment of inhalational anthrax (post-exposure). There are other medications, both oral and injectable, with better safety profiles that are available.

A 30-year-old female patient comes into your family practice office to discuss the fact that her mother just tested positive for the BRCA 1 and BRCA 2 genes. She has also been tested and is positive. What is her risk for developing breast cancer by the age of 70? A Her risk is identical to the population. B Her risk is approximately 25%. C Her risk is approximately 55%. D Her risk is approximately 85%. E Her risk is approaches 100%.

Her risk ~85% Hereditary breast and ovarian cancers put both male and female patients at increased risk for multiple cancers, including breast, ovarian, prostate, and pancreatic. In this case, the risk of breast cancer by the age of 70 is approximately 85%. When in doubt, consult a genetic counselor.

A 37-year-old female presents to the clinic for her obstetrical appointment. She is a G5P4 African American female. On physical exam her uterus is larger than expected for dates by 5 cm. What is her risk of having twins increased by? A Her increasing age B Her increasing parity C Her race D Her use of clomifine E Her late age of menarche

Her use of clomifine Incidence of multiples is not related to age, parity, or menarche; the increase seen in the last few decades is solely related to the increased use of fertility drugs.

A 52-year-old woman presents because her menopausal symptoms have been extremely distressing. Over the past 4 months, she has experienced severe mood swings, hot flashes, night sweats, breast tenderness, and changes in her appetite. She has never smoked; she has an occasional drink. She had an IUD that was removed at 35 years of age. There is no family history of cancer. After a prolonged discussion, a decision to start hormone replacement therapy is made. Question What is a legitimate concern with the use of an estrogen-only supplement? Answer Choices 1 Vaginal dryness and itching 2 Worsening mood swings 3 Lowering of serum HDL level 4 Higher risk of vertebral fractures 5 Higher risk of endometrial cancer

Higher risk of endometrial cancer Explanation A higher risk of endometrial cancer is the right answer. When estrogen is administered as the sole therapy without cyclical progesterone, it causes unopposed proliferation of the endometrium. Over a period of time, this increases the chance of dysplasia, leading to endometrial cancer. To prevent this, estrogen is usually administered with cyclical progesterone. Vaginal dryness and itching is incorrect. Estrogen helps to maintain the epithelial lining of the vagina and increases lubrication, thereby decreasing the symptoms of postmenopausal atrophic vaginitis. Worsening of mood swings is incorrect. Hormone replacement therapy during menopause is beneficial in mitigating mood symptoms. When used, they are prescribed in the lowest dose possible for the shortest period of time. Lowering of serum HDL level is incorrect. Estrogen does exactly the opposite; it raises HDL and lowers LDL levels. It acts as a cardioprotective agent in that respect. Higher risk of vertebral fractures is incorrect. Estrogen influences bone metabolism and prevents osteoporosis.

A 27-year-old woman presents with cramping abdominal pain and vaginal bleeding. Further history reveals amenorrhea for the past 2 menstrual cycles. On examination, she is found to have left lower abdominal tenderness and an adnexal mass. Lab values reveal an elevated Beta HCG level. Question A further history would possibly reveal what associated risk factor? Answer Choices 1 History of PID 2 Diaphragm use 3 Use of condoms 4 Oral contraceptive pills 5 Multiple sexual partners

History of PID Explanation A history of pelvic inflammatory disease is the correct answer. The vignette describes a potential ectopic pregnancy, evidenced by the triad of amenorrhea, adnexal mass, and an elevated B-HCG. Inflammation of the fallopian tubes secondary to bacterial infection resolves with fibrosis and adhesion formation. The fertilized egg gets trapped on its transit through the fallopian tube, resulting in an ectopic pregnancy. Being thin walled, implantation in the fallopian tube invariably results in a rupture. Diaphragm use is incorrect. Diaphragms sit atop the cervix, preventing sperm entering the uterine cavity. They are used as a temporary measure and are not associated with ectopic implantation. Condom use is incorrect. It is used as a temporary measure, and condom use is not associated with ectopic implantation. Oral contraceptive use is incorrect. Regular OCP use is associated with an extremely low failure rate. In the event of contraceptive failure, there is no increased risk of ectopy with OCP use. Multiple sexual partners is incorrect. While a common risk factor for PID, by itself it is not a risk factor.

A 26-year-old athlete presents complaining of scant menses x 4 months. She is a G0P0, menarche was at age 13, and her menses have been mostly regular. She is a non-smoker and non-drinker, and has been trying to achieve pregnancy x 8 months. Physical exam reveals a thin, white female in no distress. Vitals are normal, BMI is 17.5, her pelvic exam is normal, and STD cultures are negative. A pregnancy test is negative in clinic. What is the most likely cause for her amenorrhea? A Idiopathic B Hypothalamic C Hypothyroid D Polycystic ovarian syndrome E Androgenic

Hypothalamic A young healthy woman with a low BMI, no other signs of virilization, and a history of normal menarche is most likely to have induced amenorrhea, due to suppression of the hypothalamic axis from low weight and fat index.

A 26-year-old athlete presents complaining of scant menses x 4 months. She is a G0P0, menarche was at age 13, and her menses have been mostly regular. She is a non-smoker and non-drinker, and has been trying to achieve pregnancy x 8 months. Physical exam reveals a thin, white female in no distress. Vitals are normal, BMI is 17.5, her pelvic exam is normal, and STD cultures are negative. A pregnancy test is negative in clinic. What is the most likely cause for her amenorrhea? A Idiopathic B Hypothalamic C Hypothyroid D Polycystic ovarian syndrome E Androgenic

Hypothalamic B A young healthy woman with a low BMI, no other signs of virilization, and a history of normal menarche is most likely to have induced amenorrhea, due to suppression of the hypothalamic axis from low weight and fat index.

A 20-year-old woman was just told by her new sexual partner that she needed to be checked for a sexually transmitted infection because he has developed dysuria and a profuse urethral discharge. She herself has had a subjective fever for the past two days, some nausea but no vomiting, diffuse lower abdominal pain, and a severe backache. On examination, she has a temperature of 100.5˚F, hypoactive bowel sounds, bilateral lower abdominal quadrant tenderness, a profuse mucopurulent cervical discharge and pronounced cervical motion tenderness. Serum pregnancy testing is negative. She is given an injection of ceftriaxone and a prescription for doxycycline for 14 days and an appointment for follow up the next day. Under which of the following conditions should metronidazole be added to her regimen? A If she does not appear improved by the following morning B If she develops vomiting or diarrhea C If she has a probable tubo-ovarian abscess D If she has an allergic reaction to the doxycycline E If she has had more than one partner in the past month

If she has a probable tub-ovarian abscess C Recommended regimens for treatment of pelvic inflammatory disease include ceftriaxone or another parenteral third-generation cephalosporin and doxycycline or cefoxitin, probenecid, and doxycycline. Metronidazole (or clindamycin) should be added to either regimen if a tubo-ovarian abscess is present. If being treated as an outpatient and she has not improved in 3 days (A), or if she develops vomiting (B), she should be hospitalized rather than given metronidazole. Metronidazole is not a satisfactory substitute for doxycycline (D). The patient's condition rather than the number of sexual partners (E) determines the appropriate therapy.

A 24-year-old gravid 3 para 0 ab 2 presents complaining of vaginal pressure every 2 hours, but no pain. She is at 26 weeks gestation. This pregnancy is uncomplicated to date. On physical exam she is in no acute distress and her vitals are normal. She is placed on a fetal monitor and no contractions are noted; fetal heart tones are 138. A sterile speculum exam reveals her cervix to be dilated to 4 cm. What is the most likely diagnosis? A Preterm labor B Group B Streptococcus C Incompetent cervix D Braxton Hicks contraction E Chlamydia trachomatis

Incompetent cervix While infection is a significant cause of premature labor, it is not a cause of incompetent cervix. Incompetent cervix is most likely found in the presence of recurrent pregnancy loss and painless dilatation.

A 50-year-old woman presents for her annual pelvic examination. She states her last menstrual period was over 6 months ago; the last few occurrences of menses were extremely irregular. The patient also describes having the sensation of intense heat in her face and trunk; the sensation is accompanied by sweating. She further states that these "heat episodes" have been occurring 1 or 2 times a week for the last several months. She has no other complaints at this time. She has received her annual pap and pelvic examination yearly, as well as a clinical breast exam, without any issues. During the pelvic examination, you note obvious vaginal thinning and excessive dryness; there is also apparent vaginal wall atrophy. Question Laboratory findings in this patient would include a decreased serum estradiol as well as what? Answer Choices 1 Decreased serum thyroid stimulating hormone (TSH) 2 Increased serum follicle stimulating hormone (FSH) 3 Decreased serum luteinizing hormone (LH) 4 Increased serum prolactin level 5 Decreased serum aldosterone level

Increased serum follicle stimulating hormone (FSH) Explanation This patient scenario above is most likely to be caused from menopause. In the most pure sense of explanation, menopause is a cessation of menstruation from either natural aging (usually amenorrhea for at least 6 months) or an external cause (surgical). There is usually a 1 - 3 year time period during which women will typically adjust physiologically to the diminished hormonal and menstrual actions as well as the effects this has on their body. These effects may include hot flashes, night sweats, vaginal dryness, and in the later stages, osteoporosis. The average age of menopause in western societies is around 51 years old. Laboratory findings that indicate natural menopause include elevated levels of FSH, LH, decreased levels of estradiol (estrogen), and normal levels of TSH and prolactin. Decreased levels of TSH would be seen in hyperthyroidism, which is also referred to as thyrotoxicosis. One of the many symptoms seen with hyperthyroidism includes menstrual irregularities; however, if TSH levels are corrected, the menstrual regularity will most likely return to its normal state. Serum prolactin levels will remain unchanged in naturally occurring menopause. Levels may be increased if amenorrhea is a consequence of a prolactin-secreting pituitary adenoma. Decreased levels of aldosterone in relation to amenorrhea may be found in patients who are experiencing toxemia of pregnancy, which is not likely in this patient.

A 25-year-old G1P1 Caucasian woman who recently delivered a healthy male infant presents with breast pain and tenderness on the right; associated symptoms include malaise and fever. The patient has been compliant with all of her prenatal and post-natal obstetrical appointments. No medical complications were incurred during or after the pregnancy. The patient is currently breast feeding. Physical inspection of the right breast shows focal breast erythema and tenderness. The patient has no known co-morbidities, and she denies significant family medical problems. The patient denies palpitations, shortness of breath, or syncope. Pulse is 78 bpm, and respirations are 16 per minute; blood pressure is 102/68 mm Hg, and temperature is 101.2°F. What is the most likely diagnosis? Answer Choices 1 Inflammatory breast carcinoma 2 Congestive heart failure 3 Venous thromboplebitis 4 Infection with Staphylococcus aureus 5 Simple breast engorgement

Infection with Staphylococcus aureus Mastitis is a condition that reflects cellulitis of the periglandular tissue in breast-feeding mothers, typically due to infection with Staphylococcus aureus. Clinical symptoms of mastitis include breast pain, skin redness, fever, and flu-like symptoms. Physical findings include focal breast erythema, swelling, and tenderness, while fluctuance suggests a breast abscess. Diagnosis rests on breast milk cultures and complete blood count analysis. If fluctuance is present on physical examination, then a breast ultrasound can be used to exclude abscess formation. Treatment for mastitis involves continued breast feeding and antibiotics such as dicloxacillin or erythromycin. The treatment of a breast abscess is incision and drainage. Inflammatory breast carcinoma can clinically resemble unilateral mastitis, but it would not resolve with antibiotic therapy. Additionally, inflammatory breast carcinoma would be extremely unusual in a 25-year-old woman. If the patient's breast erythema, swelling, and tenderness continues despite antibiotic therapy, then a mammogram with an associated punch skin biopsy should be considered to rule out inflammatory breast carcinoma. Congestive heart failure can lead to bilateral breast swelling with increased breast interstitial markings and trabeculations on mammography, but it would be unusual in this patient without a past medical history of congestive heart failure or complaints of palpitations, shortness of breath, or syncope. Thrombophlebitis of the superficial veins of the breast and the anterior chest wall (Mondor Disease), a rare condition, presents as a tender, cordlike structure that may be demonstrated best by tensing the skin and elevating the arm. The clinical symptoms described in this patient are more consistent with mastitis than Mondor's disease. Simple breast engorgement can present in patients as a swollen, firm, and diffusely tender breast, but usually produces a low-grade fever rather than a high fever, as in the patient above.

A 59-year-old woman presents with a firm, non-tender mass with a "rock-hard" consistency in her right breast when palpated by her gynecologist. On further examination, the right breast exhibits redness and dimpling of the skin and her right nipple appears slightly retracted. In addition, she reports a single episode of a bloody nipple discharge. A diagnostic mammogram reveals dense fibroglandular tissue with possible microcalcifications in the area of the palpable lump. A sonographic exam reveals an immobile 10 mm hypoechoic mass. Question What is the most likely diagnosis? Answer Choices 1 Benign cyst 2 Fibroadenoma 3 Galactorrhea 4 Gynomastia 5 Infiltrating duct carcinoma

Infiltrating duct carcinoma: Explanation Infiltrating (invasive) duct carcinoma is the most common type of breast cancer. It typically does not respect anatomic borders and invades the normal breast at random. Invasion of the fat tissue can be seen on gross examination as stellate white tissue extending into the yellow fat tissue. Invasive ductal carcinomas cause a dense fibroblastic response in the host tissue that gives these tumors a rock-hard consistency. The desmoplastic reaction caused by infiltrating ductal carcinoma resembles an irregular scar which deforms and causes a dimpling of the skin and retraction of the nipple (peau d'orange appearance). This patient has many classic signs of infiltrating ductal carcinoma:firm breast mass, dimpling of the skin and retraction of the nipple (peau d'orange) and a bloody discharge. A biopsy should be performed to confirm the imaging diagnosis. Benign cyst is incorrect. Benign cystic changes are typical of non-proliferative fibrocystic changes. These changes are found in some breasts of women older than 45 years of age and are probably related to normal aging of the female breast. The breast mass described in this case does not have the characteristics of a benign cyst. Fibroadenoma is incorrect. Fibroadenoma is the most common benign breast tumor. It is composed of elongated ducts surrounded by loose fibrous stroma. These well circumscribed tumors typically present as easily movable spherical masses in breasts of 20-35 year old young women which does not the describe the patient or tumor in this case. Galactorrhea is incorrect. Galactorrhea is a milky breast discharge in non-nursing women due to a prolactinoma, a pituitary adenoma, that causes an abnormal proliferation of lactotrophs and the secretion of excess amounts of prolactin. Prolactin stimulates milk synthesis in lactating women. Galactorrhea does not occur in men because men do not have enough breast tissue. Gynecomastia is incorrect. Gynecomastia is the excessive growth of male mammary glands. Physiologic causes include: in the newborn, due to elevated maternal estrogens during pregnancy; during puberty, due to an increased estrogen to androgen ratio; and in old age, due to combined effect of decreasing testosterone and increased estrogen due to peripheral aromatization of androgen to estrogens in adipose tissue.

A 27-year-old woman has a history of recurrent ovarian cysts; she is being treated with a combination oral contraceptive (norethindrone/ethinyl estradiol 1mg/35mcg). The norethindrone in this drug acts to suppress ovulation by what process? Answer Choices 1 Decreasing circulating sex-hormone binding globulin (SHBG) 2 Increasing gonadotropin production 3 Inhibiting release of follicle-stimulating hormone (FSH) secretion from the anterior pituitary 4 Inhibiting release of luteinizing hormone (LH) secretion from the anterior pituitary 5 Stimulating proliferation in the endometrial lining

Inhibiting release of luteinizing hormone (LH) secretion from the anterior pituitary Explanation Norethindrone is the progestin component of the oral contraceptive, which "primarily suppresses luteinizing hormone (LH) secretion (and thus prevents ovulation)."1 The LH 'surge' is responsible for triggering ovulation. The LH is released from the anterior pituitary and is suppressed in a negative feedback cycle when progestin levels are increased. The norethindrone would be expected to increase circulating sex-hormone binding globulin (SHBG), resulting in decreased free androgens in the serum.2 Both the estrogen and progestin components of this drug decrease gonadotropin (LH and FSH) production by negative feedback to the anterior pituitary. The estrogenic agents in oral contraceptives (ethinyl estradiol) suppress follicle-stimulating hormone (FSH), as well as provide stability to the endometrium and potentiation of the progestin's actions. The effect on FSH is primarily attributed to the estrogen, while the effect on LH is primarily attributed to the progestin, such as norethindrone. Estrogen stimulates proliferation in the endometrial lining. Progestins, such as norethindrone, are associated with endometrial thinning and a shift to the secretory phase.1Neither effect on the endometrium leads to ovulation suppression.

A 27-year-old woman and her male partner come to the emergency department for assistance with emergency contraception. They experienced condom failure during intercourse an hour ago and neither desires pregnancy. Her last menstrual period was approximately two weeks ago and her cycles occur every 28-30 days. Her medical history includes a deep venous thrombosis during labor and delivery 5 years ago. What is the most appropriate course of action at this time? A Administration of an ethinyl estradiol and levonorgestrel combination now and in 12 hours B Dilation and curettage C Insertion of a copper-containing intrauterine device D Serial beta hCG determinations E Testing for factor V Leiden

Insertion of a copper-containing intrauterine device Insertion of a copper-containing IUD is an effective means of preventing an unintended pregnancy in this case. The woman's history of clotting is a contraindication to use of combination oral contraceptives (A). Dilation and curettage (B) is neither appropriate nor effective for emergency contraception. Performing beta hCG determinations (D) would merely detect pregnancy if it were to occur. Given her history of clotting, testing for factor V Leiden (E) may be appropriate but will not affect management at this time.

A 37-year-old woman who takes no medication and is otherwise healthy has developed a spontaneous "bloody discharge" from her left breast. Examination reveals no tenderness, masses, dimpling, or asymmetry. Gentle pressure at the margin of the areola reveals single duct involvement. This most likely represents which of the following? A Fibroadenoma B Fibrocystic changes C Intraductal papilloma D Malignancy E Pituitary adenoma

Intraductal papilloma A unilateral serous or serosanguinous nipple discharge from a single duct is more likely a benign intraductal papilloma A less-likely intraductal malignancy (D), however, is possible and must be ruled out. Fibroadenomas (A) and fibrocystic changes (B) are not usually associated with nipple discharge. A pituitary adenoma (E) is usually associated with galactorrhea, rather than a bloody discharge, from multiple ducts in both breasts.

A mother brings her 16-year-old son to your medical office for a comprehensive history and physical examination. She tells you she is concerned about his immature physical development and insecure behavior. She thinks these characteristics are markedly different from her other children. His IQ is 70, and he is in special education for a language-based learning disability. On physical examination, he is tall and thin; he has sparse body hair and a high-pitched voice. Heart, lungs, abdomen, and neurologic exam are unremarkable. Pertinent positive findings include disproportionately long arms and legs, gynecomastia, as well as small testes and phallus. Question What is the most likely diagnosis? Answer Choices 1 Fragile X syndrome 2 Klinefelter syndrome (XXY) 3 Turner syndrome (XO) 4 Triple X syndrome (XXX) 5 XYY syndrome (XYY)

Kleinfelter Syndrome Explanation The combination of hypogonadism, long extremities, decreased intelligence, and behavioral problems makes Klinefelter syndrome (also referred to as XXY syndrome, 47,XXY, and Klinefelter's syndrome) the most likely diagnosis. The original syndrome, as described by Dr. Klinefelter, consisted of gynecomastia, testicular atrophy, and infertility. Intelligence profiles can range from specific learning disabilities (language learning or reading impairment most common) to frank mental retardation/intelligence disability (MR/ID). The only constant feature of the syndrome is testicular atrophy with resulting azoospermia and infertility. The atrophy of the testis is the result of fibrosis, which begins to appear in childhood and progresses until all the seminiferous tubules are replaced by fibrous tissue. In males presenting with gynecomastia, MR/ID, and eunuchoidism (i.e., loss of male secondary sexual characteristics, small penis, loss of body hair, and a high-pitched voice), Klinefelter syndrome should be at the top of the list in the differential diagnosis. Most patients with Klinefelter's syndrome have 47 chromosomes instead of the normal 46 chromosome karyotype. The extra chromosome is an X chromosome, making the sex chromosome constitution XXY instead of XY. Klinefelter's syndrome is one of the most common chromosome abnormalities seen in males and occurs in 1 in 300 of the male population. Patients with this syndrome show that the Y chromosome is strongly sex-determining; thus, a patient who has an XXY chromosome constitution may have the appearance of a normal male, with infertility being the only incapacity, while the loss of a Y chromosome leads to the development of a bodily form that is essentially feminine. Fragile X syndrome is incorrect, as it is characterized by prominent jaw, large ears with soft cartilage, and macroorchidism in pubertal male patients. Turner syndrome (XO) is incorrect, as it is a genetic condition of females patients; it is usually characterized by a short stature, increased distance between the nipples, low hairline, low set ears, a webbed neck, amenorrhea, and sterility. Triple X syndrome (XXX) is incorrect. This condition only occurs in female patients. XYY syndrome (XYY) is incorrect. In this condition, IQ is normal, and there is normal sexual development as well as normal fertility.

What hormone is responsible for ovulation? Answer Choices 1 Follicle stimulating hormone 2 Luteinizing hormone 3 Estrogen 4 Testosterone 5 Insulin-like growth factor

LH Explanation Luteinizing hormone is the hormone responsible for initiation of ovulation. Follicle stimulating hormone is responsible for follicle maturation. Estrogen increases the number of luteinizing hormone (LH) receptors, which also also leads to unregulation of LH. Ultimately, a rapid rise in LH (LH surge) occurs approximately 24 - 36 hours prior to ovulation. Estrogenis produced by the conversion of testosterone to estradiol by aromatase in the granulosa cells.

A 25-year-old woman, G0P0, presents to your office with 1-year history of oligomenorrhea. Her most obvious physical exam findings are hirsutism and obesity, having a calculated BMI of 30. What would you expect to find on her workup results? Answer Choices 1 LH:FSH ratio of 2:1 or greater 2 TSH less than 0.4 mIU/mL 3 Fasting glucose less than 100 mg/dl 4 Free testosterone level less than 0.6ng/mL 5 Prolactin level greater than 40ng/mL

LH:FSH ratio of 2:1 or greater Explanation The correct answer is LH:FSH ratio of 2:1 or greater as this is the most positive indicator of polycystic ovarian syndrome (1). TSH less than 0.4 mIU/mL would indicate the possibility of hyperthyroidism and therefore is incorrect. The only symptom this patient has that would correlate with hyperthyroidism is the oligomenorrhea. A fasting glucose less than 100 mg/dl would indicate insulin sensitivity and a patient suspected of having PCOS usually demonstrates insulin resistance. A free testosterone level less than 0.6ng/mL is incorrect as 25-50% of patients with PCOS have elevations in free testosterone (1). A prolactin level greater than 40ng/mL indicates hyperprolactinemia, which usually is not a finding in PCOS and is indicative of a separate disorder (1).

A 30-year-old woman comes in for evaluation of infertility. She and her husband have been having unprotected intercourse for the past year. Her menstrual cramps have become increasingly painful, and she has a severe low backache for several days before and during her menses. She complains that intercourse is painful when her husband "goes deep." Physical examination reveals multiple tender nodules of various sizes in the posterior vaginal fornix. Definitive diagnosis is best accomplished using what methodology? A Abdominal radiography B CA-125 measurement C CT scanning of the abdomen D Laparoscopy E Pelvic ultrasonography

Laparoscopy D This woman has a classic presentation for endometriosis. Definitive or final diagnosis can only be made at laparoscopy or laparotomy, allowing direct visualization of the endometrial implants. Abdominal radiography (A), computed tomography scanning (C), and ultrasonography (E) are not usually helpful in making a definitive diagnosis. CA-125 (B) may be elevated in endometriosis, but lacks specificity.

A 33-year-old gravida 3, para 1011, whose last menstrual period (LMP) was 6 weeks ago, presents with a 3-day history of vaginal bleeding as well as intermittent, non-radiating right lower quadrant pain. She states that her pain scale is a 7/10 and that the pain is worse today than it has been previously. Her past medical history is significant for gonorrhea at age 16; she was treated with antibiotics. Vital signs are: BP, 80/42 mm Hg; pulse is 120 BPM, and respirations are 22/min. The patient is afebrile, and oxygen saturation is 95% on room air. The abdomen is distended, and bowel sounds are absent. There is rebound tenderness and guarding. Pelvic exam demonstrates a mass in the right adnexa. The uterus is approximately 4 - 6 weeks size. A serum beta human chorionic gonadotrophin (HCG) level is 3,723 mIU/ml. Hematocrit is 24%. Ultrasound demonstrates free fluid in the pelvis, and there is a 6 cm complex mass in the right adnexa. There is no evidence of an intrauterine gestation. Question What is the most appropriate treatment for the patient at this time? Answer Choices 1 Laparoscopic salpingostomy 2 Methotrexate, 75 mg IM x 1 dose 3 Serial quantitative beta HCG levels every 48 hours 4 Laparotomy 5 Laparoscopic right salpingo-oophorectomy

Laparotomy Explanation Approximately 2% of all pregnancies are ectopic pregnancies, and most are located in the Fallopian tube. The triad of amenorrhea, vaginal spotting, and unilateral adnexal pain is strongly suggestive of an ectopic pregnancy. This patient is hemodynamically unstable. Such patients should undergo exploratory laparotomy as soon as is possible after the presumptive diagnosis has been made. Laparoscopic surgery of any kind is unacceptable in the face of a hemodynamically unstable patient with an ectopic pregnancy. Methotrexate is an acceptable alternative to surgical management of an ectopic pregnancy, but is contraindicated when an ectopic pregnancy is ruptured, or when the adnexal mass is >3.5 cm on ultrasound. Serial quantitative beta HCG levels may be obtained every 48 hours in order to confirm the diagnosis of ectopic pregnancy; however, in this case, the patient's unstable status makes this treatment plan dangerous and unnecessary.

A 29-year-old woman has Wilson's disease, for which she is taking D-penicillamine. Today, her laboratory findings (including liver function tests) are within normal limits; there are no neurological signs. Her work requires frequent (3 - 5 days per week) travel, including international overnight travel all over the world. She wants contraception that is both effective and convenient in terms of application. Question What is your advice? Answer Choices 1 Combined oral contraceptive pill 2 Progestin-only pill 3 Nonhormonal copper intrauterine device 4 Levonorgestrel-releasing Intrauterine system 5 Contraceptive patch

Levonorgestrel-releasing Intrauterine system Explanation Only spermicide, barrier contraceptives, and progesterone-only preparations can be considered in Wilson's disease. Levonorgestrel is a 2nd-generation progestin (synthetic progestogen). A levonorgestrel-releasing intrauterine system causes thickening of cervical mucus, thereby reducing sperm motility and penetration; over time it decreases proliferation of the endometrium. The local inflammatory response could be toxic to sperm, thus providing an additional contraceptive effect. Combination oral contraceptive pills contain both estrogen and progestins. Estrogen-containing contraceptives are not recommended since estrogen may interfere with copper excretion. The progestogen-only pill is efficient; however, it must be taken at or around the same time every day (within 3 - 12 hours). Therefore, the pill may be inconvenient for a woman with a job that requires international travel all over the world. Nonhormonal copper intrauterine devices should be avoided in a disease that has the main characteristic of copper accumulation in the body. Copper IUDs primarily work by disrupting sperm mobility and damaging sperm. Contraceptive patches also contain estrogen; therefore, they should be avoided in Wilson's disease.

A 34-year-old diabetic female complains of amenorrhea for the past 2 months. Last week she tested positive for strep pharyngitis and is currently being treated. She admits that she has not been using birth control. Her urine HCG is positive. Her current medications are listed in the choices below. Which of the following medications should you discontinue? A amoxicillin B lisinopril C acetaminophen D humalog insulin E methyldopa

Lisinopril B The correct answer is (B). Lisinopril, an ACE inhibitor, is contraindicated in pregnancy due to known problems with fetal toxicity and should be stopped as soon as possible once pregnancy is confirmed. If a patient is planning on becoming pregnant the ACE inhibitor should also be discontinued. ARBs should also be avoided. Choices (A), (C), and (D), and (E) can be used safely in pregnancy and are considered category B. Methyldopa is preferred in the treatment of hypertension in pregnancy and its safety is supported by evidence.

Breast cancer is one of the most frequently occurring cancers in women. Which of the following methods is the most effective for detecting breast cancer? Answer Choices 1 Mammography 2 Breast Self Examination 3 Yearly physical examination 4 Thermography 5 Ultrasonography

Mammography Explanation The breast is the most frequent site of occurrence for cancer in women. One out of nine women will have breast cancer in their lifetime. Breast self examination (BSE) is one of the effective early detection modalities for this disease, but recent studies show that mammography is the only clinically effective screening measure for the detection of breast cancer. The American Cancer Society recommends, in the United States, annual screening mammography with clinical examination in women aged 40 years. Research has shown that BSE plays a small role in finding breast cancer compared with finding a breast lump by chance or simply being aware of what is normal for each woman. Some women feel very comfortable doing BSE regularly (usually monthly) which involves a systematic step-by-step approach to examining the look and feel of one's breasts. Doing BSE regularly is one way for women to know how their breasts normally feel and to notice any changes. Thus, more than actual detection and decrease in deaths due to early detection of cancer, BSE helps create familiarity for the patient with her breasts and thereby report any changes earlier to the doctor.

A 24-year-old female presents with hyperpigmented macules on her cheeks, nose, and upper lip. They have been present for a couple of months. Her current medications include oral LoEstrin 24 Fe, cetirizine, and a multivitamin daily. What is the most likely diagnosis? A congenital nevus B melasma C post-inflammatory hyperpigmentation D café-au-lait macule

Melasma The patient is experiencing melasma secondary to the use of oral contraceptives. This is a frequent cause of melasma. Melasma can also be precipitated by hormonal changes that occur during pregnancy. The condition will resolve upon discontinuation of the oral contraceptive. A congenital nevus is a nevus that presents within the first year of life. It is monitored in the same way as acquired nevi. They can be larger than acquired nevi, with only a slight increase in chance of malignant change over time. Post-inflammatory hyperpigmentation includes darker areas of pigmentation that can result after inflammation on the skin. Common causes include acne and atopic dermatitis. The hyperpigmentation will resolve over time. A Café-au-lait macule is a type of birthmark. It is usually light tan to light brown in appearance, and can vary greatly in size. They are usually benign, but can be associated with neurofibromatosis when more than six, with a diameter greater than 1.5 cm, are present.

A 50-year-old woman presents for her annual pelvic examination; her previous examination took place about 18 months ago. She tells you that her last menstrual period was over 12 months ago; the last few occurrences of menses were extremely irregular. The patient also describes having multiple daily episodes of severe, intense heat in her face and trunk that is accompanied by sweating. She further states that these "heat episodes" have been occurring 4 - 6 times a day for the last 4 weeks. They have been interfering with her everyday activities, as well as her sleep. She has no other complaints at this time. She has received her annual pelvic examinations yearly as well as clinical breast exams and mammograms; there have been no significant findings. During the pelvic examination, you note obvious vaginal thinning and excessive dryness; there is also apparent vaginal wall atrophy. Question What pharmacologic intervention can be prescribed in order to assist in ameliorating the patient's symptoms? Answer Choices 1 Menopausal hormone therapy 2 Low-dose oral contraceptives 3 Phytoestrogens 4 Progestin-releasing intrauterine devices 5 Testosterone replacement therapy

Menopausal hormone therapy Explanation This patient scenario above is most likely the result of menopause. Menopause in the most pure sense of explanation is a cessation of menstruation from either natural aging (usually amenorrhea for at least 6 months) or an external cause (surgical). There is usually a 1 -3 year time period that women will typically adjust physiologically to the diminished hormonal and menstrual actions and the effects this has on their body. These effects may include hot flashes, night sweats, vaginal dryness, and in the later stages, osteoporosis. In western societies, the average age at which women achieve menopause is about 51 years old. The patient described in the scenario above is suffering from moderate-to-severe symptoms of the physiological menopausal transition. Given that this patient does not have any obvious risk factors, menopausal hormone therapy (MHT) is the best pharmacological option at this time. At one time, MHT was referred to hormone replacement therapy. Benefits of the use of MHT include reducing hot flashes, night sweats, and other related issues, such as poor sleeping and irritability. MHT will also treat vaginal symptoms related to menopause, such as vaginal dryness and discomfort. Another benefit to its use is slowing of bone loss and easing mood swings or depressive symptoms. An extremely critical component that must be remembered by the healthcare provider and the patient if MHT is started is that the use of MHT is only recommended for a short period of time and at the lowest effective dose. Low-dose oral contraceptives (or birth control pills) may be an option for menopausal symptom relief only if the patient is perimenopausal. Because this patient has not had her menstrual period in over 8 months, she is categorized as being menopausal, so this treatment would not be appropriate. Phytoestrogens are substances found in plants, such as soy and red clover extracts, that bind to estrogen receptors. They do not appear to significantly improve menopausal hot flashes, and they would not be the best option for this patient. Progestin-releasing intrauterine devices (IUDs) would not be an appropriate treatment option as this is only used for patients using MHT to reduce the incidence or dysfunctional uterine bleeding and endometrial carcinoma. Testosterone replacement therapy in women is reserved for those patients suffering from mainly hypoactive sexual desire disorder (HSDD); other signs and symptoms that may benefit from testosterone replacement therapy include hot flushes, pubic atrophy, muscle atrophy, and osteoporosis; however, this is not the correct treatment option for this patient.

A 27-year-old woman complains of years of menstrual irregularity and increasing facial and chest hair. PMH: significant for ovarian cyst and left cytectomy. She is a non-smoker and non-drinker. Labs include a negative uCG, elevated LH, and low FSH. She desires fertility and she has not responded to three cycles of clomiphene. What would be the next choice of medication that may return ovulation? A Insulin B Metformin C Dexamethasone D Spironalactone E Finasteride

Metformin B Dexamethasone, finasteride, and spironalactone all treat symptoms of hirsutism, but do not treat the underlying cause of PCO or improve fertility outcomes. PCO has an underlying insulin resistance that can be treated with oral hypoglycemics and improve sensitivity to insulin. Adding insulin does not improve the resistance.

A 24-year-old G2P2 delivered a viable female infant (8 lb 4 oz) via caesarean section, after a failed 20-hour induction for post date pregnancy. On day 2, she developed a postoperative fever of 101F (38.3C). She had slightly increasing abdominal cramping and pain, no change in loci, is voiding well, and has passed flatulence. Her WBC is 19,000. What is the mostly likely cause for her fever? A Urinary tract infection B Ileus C Metritis D Atelectasis E Tubo-ovarian abscess

Metritis C The patient is passing urine and flatulence well, making choices A and B less likely. Fever greater the 38C is the most important indicator of metritis. Fever is not usually indicative of mild atelectasis. Tubo-ovarian abscess is usually a complication from PID. The patient has many risk factors for metritis including c-sect, prolonged induction, and fever.

A 19-year-old presents complaining of vaginal discharge and itching for 3 days. She is sexually active and uses condoms most of the time. A physical exam reveals the following: vitals are normal, abdomen is soft and non-tender, + bowel sounds, the pelvic exam is notable for moderate discharge with no masses or tenderness. Her wet mount/KOH prep reveals 20 wbcs, 2+ bacteria, no hyphae, 5 to 7 clue cells. What is the most appropriate treatment? A Terconazole vaginal cream daily x 3 days B Cefixime 400 mg x one dose C Ciprofloxin 500 mg BID x 7 days D Metronidazole 500 mg BID x 7 days E Azithromycin 1 gram x 1 dose

Metronidazole 500 mg BID x 7 days D Clues cells with bacteria and white blood cells are indicative of Gardnerella vaginitis, which is best treated with metronidazole.

Septic arthritis in adults younger than 30 years is usually caused by A Neisseria gonorrhea B Staphylococcus aureus C Pseudomonas aeruginosa D Streptococcus pyogenes E Salmonella species

Neisseria gonorrhea In patients younger than 30 years, gonococcus is the most common cause of septic arthritis. When all patients are considered, Staphylococcus aureus is the most common cause. Patients with prevalent joint disease and intravenous drug users are especially susceptible to Staphylococcus. Pseudomonas is also a common cause of septic arthritis in intravenous drug users. Salmonella is not a common cause of joint infection.

While performing a Doppler ultrasound on a woman at 30 weeks gestation, you notice that the fetal heart rate is consistently within the range of 130 - 140 beats/minute. What can you conclude about the heart rate of the fetus? Answer Choices 1 Structural heart disease 2 Normal heart rate 3 Asphyxia 4 Bradycardia 5 Tachycardia

Normal HR Explanation The fetus has a normal fetal heart rate, as it is within the range of 110 - 160 beats/minute. The diagnosis of structural heart disease cannot be established by a fetal heart rate. The fetus has neither tachycardia (an early sign of fetal asphyxia), nor bradycardia (a late sign of asphyxia). Fetal bradycardia is defined as a decrease in the baseline fetal heart rate to <100 beats per minute. Suspicious tachycardia is defined as the heart rate being between 150 and 170; a pathological pattern is above 170 beats/minute.

A 36-year-old presents to the office for evaluation of painful breasts, which is worse before her period. She complains of them feeling fuller and lumpier before onset of her menses. She has tried acetaminophen and ibuprofen with minimal relief. Her symptoms resolve at the end of her menses. What is the most likely cause of the symptoms? A Normal cyclic hormone fluctuation B Abnormal cyclic hormone fluctuation C No cyclic hormone elevation D Hormone secreting tumor E Hypothalamic dysfunction

Normal cyclic hormon fluctuation A Age, cyclic history, and resolution are essential in the diagnosis of benign fibrocystic breast disease.

A 23-year-old G1P0 woman presents for her first prenatal visit. The first day of her last menstrual period (LMP) was February 23. Generally, she feels well and has no complaints. The patient has started a prenatal vitamin, is getting the appropriate amount of sleep, and is keeping up with a moderate exercise routine. A complete physical exam is performed, and there are no abnormal findings. Initial ultrasound documents fetal cardiac activity. Question Using Nägele's rule, what is the patient's estimated date of confinement (EDC)? Answer Choices 1 November 16 2 November 30 3 November 23 4 September 23 5 September 30

November 30 Explanation Nägele's rule is a mathematical way of calculating a pregnant patient's EDC using the first date of her LMP: EDC = LMP - 3 months + 7 days. Therefore, using this equation, this patient's EDC is November 30.

A 33-year-old G1P0 presents for evaluation of her inability to conceive a pregnancy for six months. She menstruates monthly. Her past medical history is significant for PID x 2, for which she was hospitalized for IV antibiotics. Her Chlamydia, GC, and pap smear are normal. When should a more comprehensive evaluation for her infertility begin? A Now B 6 months C 12 months D 18 months E 24 months

Now A A comprehensive work up should begin now, due to her advancing age and history of significant PID, which may require surgical treatment.

A woman who is pregnant suffered a spontaneous abortion at 12 weeks gestation. She is now a G2P1Ab1 and is Rh negative. When should she receive her next Rhogam (Rho D immune globulin) shot? A Now B In one month C At conception of her next pregnancy D 28 weeks gestation of next pregnancy E After delivery of her next viable infant

Now A Placental implantation occurred and separated with the spontaneous miscarriage. Therefore, there is a slight chance of isoimmunization, so Rhogam should be given now so that the mother does not develop antigens that can cross the placenta during the first half of the next pregnancy.

A 33-year-old G1P0 presents for evaluation of her inability to conceive a pregnancy for six months. She menstruates monthly. Her past medical history is significant for PID x 2, for which she was hospitalized for IV antibiotics. Her Chlamydia, GC, and pap smear are normal. When should a more comprehensive evaluation for her infertility begin? A Now B 6 months C 12 months D 18 months E 24 months

Now A comprehensive work up should begin now, due to her advancing age and history of significant PID, which may require surgical treatment.

A 51-year-old female presents to her primary care provider for her annual physical. She is a healthy white female and a non-smoker. She has mild HTN, but an otherwise negative health Hx. In addition, FHx is negative and ROS is negative. Her LMP was 6 months ago. Her last mammogram was 3 years ago. When should she have her next mammogram? A In 2 years B In 1 year C Now D Only if self breast exam reveals abnormality E Only if provider breast exam reveals abnormality

Now C The preponderance of data strongly supports the benefits of a screening mammography. New analyses of older randomized studies have suggested that screening may not work. While the design defects in some older studies cannot be retrospectively corrected, most experts, including panels of the American Society of Clinical Oncology and the American Cancer Society, continue to believe that screening conveys substantial benefit.

A previously healthy 30-year-old woman, G1 P1, presents with amenorrhea, weight loss, shortness of breath, and increasing abdominal circumference. Menstrual irregularity started about 1 year ago, and her last menstruation was 3 months ago. She has lost around 5 kilos over the last few months, but at the same time, her waist has enlarged. Shortness of breath started a week ago, and it is worse when she is lying down; she now sleeps using at least 2 pillows. An examination of the lungs shows dullness to percussion, decreased tactile fremitus, and inaudible breath sounds bilaterally. Physical examination of the abdomen shows bulging of the flanks in the reclining position, and there is a difference in percussion in the flanks that shifts when she is turned on the side. Pelvic examination shows a normal uterus and left adnexa; the right adnexum appears enlarged, smooth, and tender. Complete blood count and chemistry is normal, and a chest X-ray confirms the presence of pleural effusion on the right side. Fluid obtained from peritoneal cavity shows heterogeneously bloody content that clots; the leukocyte number is normal, and serum-ascites albumin gradient (SAAG) is 0.8 g/dL (low). The sample is negative of malignancy. Question What is the most likely cause of her symptoms? Answer Choices 1 Extrauterine pregnancy 2 Acute heart failure 3 Acute liver failure 4 Intra abdominal hemorrhage 5 Ovarian fibroma

Ovarian fibroma Explanation Ascites in a young woman is most often caused by liver, pancreatic or renal disease, pelvic or abdominal tumours, and infection. The SAAG less than 1.1 g/dL points to the non-portal hypertensive ascites. This case is Meigs syndrome; it is characterized by the triad of benign ovarian tumor (usually fibroma), ascites, and pleural effusion that resolves after resection of the tumor. Menstrual irregularity and amenorrhea are probably caused by a fibrous growth in this woman's ovary, which causes abnormal levels of sex hormone production. An extrauterine pregnancy will not present with the ascites and pleural effusion. The clinical presentation of ectopic pregnancy occurs at approximately 2 months after the last normal menstrual period; it typically causes vaginal bleeding and abdominal pain. Acute heart failure may present with shortness of breath, and ascites and ovarian tumor may be the accidental finding; however, SAAG >1.1 g/dL rules out non-portal hypertensive ascites. The SAAG is calculated by subtracting simultaneously obtained the ascitic fluid albumin value from the serum albumin value, and it correlates directly with portal pressure. Therefore, it is the best single test for classifying the ascites into the ascites caused by portal hypertension (SAAG >1.1 g/dL) and non-portal hypertension (SAAG < 1.1 g/dL). Its accuracy is about 97%. Acute liver failure is not likely when there is a normal CBC and chemistry panel with non-portal hypertensive SAAG. Visible bloody ascites requires the presence of more than 20,000 red blood cells/µL. It is usually a consequence of either malignancy or a traumatic tap. A traumatic tap results in a heterogeneously bloody fluid that will clot. Therefore, a traumatic tap is most probably the case of bloody peritoneal fluid.

A patient seen at the prenatal clinic develops Graves disease at 25 weeks' gestation. Which of the following is the most appropriate treatment? A PTU 100 mg po tid B methimazole 10 to 30 mg po qd C propranolol 80 mg po qid D radioactive iodine therapy (RAI, 131I) E levothyroxine 0.1 mg po qd

PTU 100 mg po tid In nonpregnant patients, PTU and methimazole are the drugs of choice for the management of Graves disease. During pregnancy, PTU has a lower incidence of crossing the placental barrier than does methimazole. It also is excreted into breast milk to a lesser degree than is methimazole. Propranolol will help with the symptoms of Graves but not treat it. It can also cause low birth weight in the infant. RAI is contraindicated in pregnancy. Levothyroxine will worsen a Graves patient's hyperthyroidism.

A 75-year-old woman presents to your office complaining of intense pruritis of the vulva and occasional bleeding. She is unsure if the bleeding is caused by her scratching in attempts to alleviate the itching. She has tried some OTC preparations to alleviate the itching and has not had any relief from them. She denies any vaginal discharge or dysuria. On physical examination you notice excoriations and some scattered lesions that look like eczema on the vulva and they do not scrape off. Also noted was inguinal lymphadenopathy. You decide to do a punch biopsy. The pathology report reveals large eosinophilic cells. Question What is the most likely diagnosis? Answer Choices 1 Lichen sclerosis 2 Vulvar carcinoma 3 Paget's disease 4 Melanoma 5 Candida albicans

Paget's disease Explanation Paget's diseaseis associated with intense pruritus of the vulva, along with lesions that resemble eczema. Pathology also characteristically shows large eosinophilic Paget's cells (1). Lichen sclerosis is not correct because the pathology would reveal changes associated with chronic inflammation. Vulvar carcinoma is incorrect as it generally shows squamous cell characteristics. (1). Melanoma is incorrect as the lesion will generally appear hyperpigmented (1). Candida albicans is incorrect because the lesions that appear on the vulva do not scrape off and she does not have any vaginal discharge. Candida albicans is also not associated with inguinal lymphadenopathy.

A 21-year-old female returns to clinic for treatment of her abnormal pap smear. She had been sexually active since age 18. She uses oral contraceptives for birth control. Her pap smear showed mild cervical intraepithelial neoplasm, with positive high-risk HPV. What is the most appropriate next step for management of her pap smear? A HPV testing in 6 months B Pap smear in 6 months C Colposcopy with directed biopsy D LEEP procedure E STD testing

Pap smear in 6 months B According to new ACOG guidelines, cancer screening should begin at age 21. Routine follow up is based on normal pap smear and no history of high-risk HPV. The presence of high-risk HPV necessitates the exclusion of a higher grade cervical lesion.

A 48-year-old woman presents complaining of vaginal fullness and difficulty passing stool. Upon exam she is found to have a stage 3 rectocele by the Baden Walker System. What is the most common cause for this? A Increasing age B Genetic disposition C Obesity D Pelvic floor injury E Constipation

Pelvic floor injury D While all of the answer choices are risk factors, the most common cause of pelvic organ prolapse remains pelvic floor injury, usually related to child birth or trauma.

A 37-year-old gravida 4, para 1, whose last normal menstrual period was 6 months ago, presents with 5-month history of irregular menses. She states her menstrual period is now 10 days in length. She uses approximately 10-12 pads or tampons daily during menses. She also has unpredictable bleeding between menstrual periods. This bleeding may begin anywhere from 2-12 days after the end of her regular period, and may last for between 2-15 days. She uses between 5-10 pads or tampons a day during this time. She denies syncope but admits to headaches and occasional dizziness. She also complains of bilateral pelvic pain that is present constantly and that radiates down both thighs. The pain is crampy, and it is relieved to some degree by naproxen sodium. On exam, the patient's vital signs are within normal limits. Abdominal exam demonstrates normal bowel sounds. There is a globular and irregular pelvic mass that extends about 5 cm above the umbilicus. The mass is nontender. There is no rebound or guarding. The pelvic exam reveals scant blood in the vaginal vault, no vaginal or cervical lesions, no cervical motion tenderness, and confirms the presence of the pelvic mass. Question What is the next best step in management for this patient? Answer Choices 1 Dilation and curettage 2 Pelvic ultrasound 3 CT scan of abdomen and pelvis, with and without contrast 4 Follicle-stimulating hormone (FSH), luteinizing hormone (LH), prolactin 5 Total and free testosterone

Pelvic ultrasound Explanation The patient's age and her history of menometrorrhagia, pelvic pain, and a globular, irregular pelvic mass are strongly suggestive of leiomyomata uteri. Fibroids are the most common solid pelvic tumors in women. The best study to evaluate the pelvic mass is ultrasound. Dilation and curettage is not indicated at this time. The patient should have an endometrial biopsy performed, because she is over the age of 35 and is experiencing abnormal vaginal bleeding, though it is probably due to leiomyomata uteri. If an endometrial biopsy is unsuccessful, then hysteroscopy and dilation and curettage would be indicated. CT scan of the abdomen and pelvis, with and without contrast, is not indicated at this time. If ultrasound exam were suspicious for malignancy, a CT scan of the abdomen and pelvis would be indicated to evaluate further the mass as well as to determine the extent of any lymphadenopathy. Hormone studies (FSH, LH, prolactin, free and total testosterone) are not generally useful in the diagnosis of a pelvic mass. They may be of use in a patient with abnormal vaginal bleeding whose pelvic exam does not reveal any masses.

A 58-year-old woman with no significant past medical history presents with a 6-month history of "heartburn", sometimes occurring after meals. There is associated fatigue, bloated abdominal sensation, early satiety, and alternating constipation and diarrhea. She denies fever, chills, changes in weight, chest pain, shortness of breath, abdominal pain, nausea, vomiting, melena, hematochezia, and vaginal discharge. Her last menstrual period was 4 years ago. She is unmarried and does not have any children. Her physical exam reveals normal vital signs and a normal cardiopulmonary exam. Her abdomen is protuberant; there is a shifting dullness and a fluid wave noted. The pelvic exam reveals a solid, irregular fixed lesion in the left lower abdomen. Question What is the most appropriate initial intervention for this patient at this time? Answer Choices 1 Perform a transvaginal ultrasound 2 Prescribe a proton pump inhibitor 3 Refer the patient for an upper endoscopy 4 Prescribe medication for irritable bowel syndrome 5 Observation and reassessment in 1 to 3 months

Perform a transvaginal ultrasound Explanation This patient's presentation suggests ovarian cancer. Doppler transvaginal ultrasonography is used in the initial evaluation, and it is the most useful initial investigational tool in the assessment of adnexal masses. Ultrasonography may define the morphology of the pelvic tumor. In addition, it can determine whether the suspected tumor has metastasized to other abdominal organs, such as the liver. Sonographic features suggestive of cancer include complexity with solid and cystic areas, extramural fluid, echogenicity, wall thickening, septa, and papillary projections. Proton pump inhibitors and irritable bowel therapeutics should not be initiated without adequate investigation into the presence of abdominopelvic malignancy. In patients with diffuse carcinomatosis and gastrointestinal symptoms, a GI tract workup and endoscopy may be indicated as an adjunctive modality to a transvaginal sonogram.

A 40-year-old woman presents for a consultation. She takes a multivitamin, iron, and calcium as advised. She has never smoked cigarettes and does not drink alcohol at all. After she gave birth to her fourth child about 7 years ago, she underwent bilateral tubal ligation because both she and her husband decided not to have more children. Afterwards, she started gaining weight (now her BMI is 30). In the last several months, her periods have come every 2 weeks and last about 5-7 days. Before that they were regular: every 28-30 days, with 3-5 days of bleeding. Her mother died from breast cancer at age 70. Physical examination reveals pallor and tachycardia. Pelvic examination is normal. Question What is the next best diagnostic step? Answer Choices 1 Prescribe oral contraceptive pills 2 Check FSH 3 Perform endometrial biopsy 4 Schedule mammogram 5 Check LH

Perform endometrial biopsy Explanation Your patient most probably has dysfunctional uterine bleeding (DUB). DUB is irregular uterine bleeding that occurs in the absence of pathology or medical illness. It is a diagnosis of exclusion. Laboratory studies for patients with DUB include human chorionic gonadotropin (HCG), complete blood count (CBC), Pap smear, endometrial sampling, thyroid functions and prolactin, liver functions, coagulation studies/factors, and testing for the presence of uterine fibroids or polyps and hormone assays when indicated. Most important, however, is to exclude endometrial hyperplasia or carcinoma because your patient has several risk factors: she is older than 35 years; she is obese; and she has prolonged periods of unopposed estrogen stimulation and probably chronic anovulation. Endometrial biopsy is the most important and most commonly used diagnostic test for DUB in such a patient. In DUB there is usually a hormonal imbalance (too much estrogen thickens endometrium, and progestrone causes excessive irregular bleeding). The effect of oral contraceptive pills on endometrium mimics that of an ovulatory cycle. However, before you start oral contraceptive pills, you should confirm the diagnosis of DUB, which means excluding the other reasons. Follicle-stimulating hormone (FSH) testing may be ordered when a woman's menstrual cycle has stopped or become irregular to determine if the woman has entered menopause. That is not the case in your patient. Mammography is indicated in this woman. The National Cancer Institute recommends that women age 40 or older should have screening mammograms every 1 to 2 years. However, it is not the most important test you should order at this point; instead, you should find a reason for bleeding because of all the consequences of losing blood and because of the risk of endometrial cancer. Checking luteinizing hormone (LH) will be indicated in women having difficulties getting pregnant, having irregular or heavy menstrual periods, or having symptoms of pituitary, hypothalamic, or ovarian disorders. Again, endometrial biopsy done in the office is probably more important.

A 54-year-old female presents to clinic complaining of vaginal fullness and leaking of urine for 10 years, which is progressively getting worse. She is a G4P4 and is postmenopausal. Nothing seems to improve her symptoms, and coughing or running makes them worse. On physical exam her vitals are as follows: Ht 5'4", Wt 135 lb, T 98°F, BP 130/72. Her abdomen is soft and non-tender with no masses, and her pelvic exam reveals the anterior wall to be at the level of the hymen. In addition, UA dip is negative. She is on no medication. What would the most appropriate initial management include? A Pessary B Surgery C Renal CT D Renal functions E Cystogram

Pessary A This is a stage 2 cystocele by the Baden Walker System, with urinary incontinence. It is best treated with conservative measures. If they fail, then surgery should be considered.

A 30-year-old woman presents to the office complaining of an inability to achieve pregnancy after over a year of trying. What is the couple's least likely cause for infertility? A Azoospermia B Ovarian failure C Endometrial disease D Tubal disease E Pituitary disease

Pituitary disease E In the vast majority of women who are infertile or subfertile, the issue is due to either ovarian dysfunction or structural abnormalities, with hormonal abnormalities accounting for only 10%.

A 37-year-old G3P2 female at 39 weeks gestation presents to the labor and delivery unit complaining of abdominal pain. Laboratory evaluation reveals anemia coagulopathy that is felt to be consumptive. What is the most likely diagnosis to have caused this? A Placental abruption B Placenta previa C Preeclampsia D Labor E Pre-existing anemia

Placental abruption Reproductive placental abruption is the most common cause of coagulopathy in pregnancy; the hemorrhage may be concealed and is not always evident. In the presence of pain, anemia, and coagulopathy, abruption should be assumed until proved otherwise.

A 23-year-old primigravida is referred to her obstetrician by a community nurse. The nurse noted 2 blood pressure readings of 150/90 and 154/90 taken 15 minutes apart. Physical examination reveals a uterus at roughly 24 weeks gestation and grade 2 pitting edema. Urine analysis is 1+ positive for albumin. A sonogram at eight weeks gestation showed a single live intra-uterine gestation. She has had regular antenatal check-ups, and has no past history of hypertension or diabetes. Both her parents and grandparents are hypertensive. She is unaware of whether her mother or grandmother faced similar problems during their pregnancies. Question What is the most likely initiating event for her condition? Answer Choices 1 Over activation of her renin-angiotensin system 2 Overproduction of B-HCG 3 Placental ischemia 4 Renal artery stenosis 5 Essential hypertension

Placental ischemia Placental ischemia is the correct answer. The vignette describes a woman with classic features of pregnancy induced hypertension, as evidenced by the blood pressure readings, edema, time of onset beyond 20 weeks and proteinuria. The initiating event appears to be abnormal cytotrophooblast invasion of the spiral arterioles, which leads to activation of the maternal vascular endothelium. The cascade that follows includes enhanced formation of endothelin and thromboxane, increased vascular sensitivity to angiotensin II, reduced nitric oxide, and prostacyclin (vasodilator) synthesis. Over activation of her renin-angiotensin system is incorrect. While it is an important component of the response, it does not initiate the process. Overproduction of B-HCG is incorrect. It may result in high blood pressure as seen in hydatiform mole and multiple gestations. However, there is no indication of either in this vignette. Renal artery stenosis is incorrect. It is a common cause of hypertension in young individuals. It would be present through the duration of pregnancy. Essential hypertension is incorrect. Essential hypertension is a pathology associated with aging, the pathogenesis of which is unclear. It is certainly unlikely in a 23-year-old. In addition, the patient would have a past history of hypertension.

A 24-year-old woman, gravida 0, presents with heavy bleeding between her periods, with cycles that seem irregular and more than 40 to 60 days apart. She has never had regular monthly periods. She is also concerned that she has acne and hair growth on her face, chest, and abdomen. She would like to become pregnant. She denies any chronic medical problems, although she states that her previous physician has advised her to lose weight because she has borderline diabetes. She is 5 feet 5 inches tall and weighs 240 lbs. Her gynecologic examination is unremarkable. Question What is the most likely explanation for her abnormal uterine bleeding? Answer Choices 1 A testosterone-secreting tumor of the ovary or adrenal 2 Increased exogenous estrogen 3 Increased exogenous progesterone 4 Loss of pulsatile GnRH due to thyroid dysfunction 5 Polycystic ovary syndrome (PCOS)

Polycystic ovary syndrome (PCOS) Explanation This patient has polycystic ovary syndrome. PCOS, also known by the name Stein-Leventhal syndrome, is an endocrine condition present in 5% to 10% of women of reproductive age. It is the most common cause of infertility in women. It is referred to as polycystic because most women with the condition have a number of small cysts in the ovaries; however, it is the characteristic constellation of signs, symptoms, and biochemical aberrations, rather than the presence of the cysts themselves, that is important in establishing the diagnosis, including dysfunctional uterine bleeding (DUB) due to estrogen breakthrough bleeding, hyperandrogenism, insulin resistance, and often obesity. Each plays a role in the evolution of an oligo-ovulatory state. It most commonly affects women ages 15 - 20 and is characterized by high estrogen and androgen levels, resulting in virilization (hair growth and acne), chronic menstrual irregularities, and infertility. Although the exact mechanism is yet to be determined, it is thought to involve a disorder of the hypothalamic-pituitary axis, in which access luteinizing hormone (LH) and androgen production causes virilization and anovulation with cyst formation in the ovary. Increased endogenous estrogen associated with obesity is likely to contribute to her anovulatory state, but there is no evidence to suggest that she is exposed to exogenous estrogen. Hyperandrogenism and, thus, virilization can be seen in the presence of a testosterone-secreting tumor of the ovary or adrenal, but her long history of oligomenorrhea makes this less likely. Thyroid dysfunction may also cause anovulation through dysregulation of a feedback loop that result in increased prolactin levels. However, her long history, including hyperandrogenism (acne and excess hair growth) and obesity, make this diagnosis less likely. A similar bleeding pattern could be seen with progestin-only contraception, such as Depo-Provera (depot medroxyprogesterone acetate/DMPA, or the "mini-pill," progestin-only birth control pill), but such hormonally-related bleeding would be due to progesterone breakthrough rather than estrogen breakthrough, as seen here. This patient desires pregnancy and is not exposed to exogenous progestin. Women with PCOS are at risk for cardiovascular illness and diabetes mellitus. They are also at increased risk for endometrial hyperplasia and endometrial cancer because of long-term unopposed estrogen stimulation of the endometrium.

A 23-year-old G1P0 presents to the office complaining of headache, nausea, swelling, and generally not feeling well. She is at 33 weeks gestation. A physical exam reveals a 5-pound weight gain in 2 weeks, BP 148/90, P 84, T 98.1°F, and UA concentrated with 1+ protein. What is the most likely diagnosis? A Pregnancy induced hypertension B Preeclampsia C Eclampsia D Gastroenteritis E Third space fluid retention

Preeclampsia B Preeclampsia has a classic triad of weight gain, elevated blood pressure, and proteinuria. A woman with this triad of symptoms has preeclampsia until proven otherwise.

Vaginal trichomoniasis has been associated with adverse pregnancy outcomes. What is an example of such an outcome? Answer Choices 1 Hyperemesis gravidarum 2 Premature rupture of membranes 3 Preeclampsia 4 Gestational Diabetes 5 Eclampsia

Premature rupture of membranes Trichomoniasis infection is associated with premature rupture of membranes, preterm delivery, and low birth weight. There is no proven association between trichomoniasis and hyperemesis gravidarum, preeclampsia, eclampsia, or gestational diabetes.

A 17-year-old female presents to the emergency department complaining of watery vaginal discharge for 6 hours. She is found to be at 35 weeks gestation. An external fetal monitor reveals fetal heart tones in the 130s, good variability, and no contractions. What is the most likely diagnosis? A Preterm labor B Preterm rupture of membranes C Premature rupture of membranes D Preterm labor and premature rupture of membranes E Preterm and premature rupture of membranes

Preterm and premature rupture of membranes Preterm rupture of membranes is defined as rupture before 37 weeks gestation; premature is defined as before the onset of labor. The absence of pain or contractions decreases the likelihood of labor.

A 23-year-old woman presents to clinic complaining of amenorrhea for 3 months. She also complains of increasing facial hair and weight gain. On exam, PMH: menarche age 13. Physical exam reveals a well-developed, slightly obese female with a BMI of 29. Her amenorrhea can likely be improved with which therapy? A Testosterone administration B Progesterone administration C Estrogen administration D Estrogen suppression E Testosterone suppression

Progesterone administration B Progesterone administration slows GnRH pulses, thereby improving FSH secretion and follicular maturation.

A 23-year-old woman presents to clinic complaining of amenorrhea for 3 months. She also complains of increasing facial hair and weight gain. On exam, PMH: menarche age 13. Physical exam reveals a well-developed, slightly obese female with a BMI of 29. Her amenorrhea can likely be improved with which therapy? A Testosterone administration B Progesterone administration C Estrogen administration D Estrogen suppression E Testosterone suppression

Progesterone administration Progesterone administration slows GnRH pulses, thereby improving FSH secretion and follicular maturation.

A 25-year-old woman presents to discuss her options of available contraception. You review her history and note menses onset at 12 years old, duration of menses typically around 6 days in length, and it occurs every 30 days. She is G0P0, and she has no history of abnormal pap smears or diagnosed STIs. The patient is a non-smoker, single, and in a monogamous relationship with 1 partner for the past year. All other medical history is noncontributory with the exception of the patient having a deep venous thrombosis at age 19 and a pulmonary embolism at age 21. Other than anticoagulation therapy for the appropriate amount of time, no other hematological evaluation was pursued after these events. Question Given the most likely inherited diagnosis, what would be the safest form of contraception at this time? Answer Choices 1 Estrogen/progestin combination oral contraception pill 2 Progestin-only oral contraception pill 3 Etonogestrel/ethinyl estradiol vaginal ring 4 Norelgestromin/ethinyl estradiol patch-transdermal 5 Estradiol/medroxyprogesterone monthly injection

Progestin-only oral contraception pill Explanation The correct response is progestin-only oral contraception pill. Our patient above is overall healthy 25-year-old female patient. However, her history of multiple hypercoagulable events without any substantial risk factors puts her in the likely category of possessing an inherited hypercoagulable state. The most common hypercoagulable state that is inherited currently is Factor V Leiden. Despite the fact that this patient is very highly likely to have Factor V Leiden, she can still be offered a type of contraceptive method to prevent unplanned pregnancy. The common pre-thrombotic contraceptive component is estrogen. For this reason, the patient above would be safest at beginning any method of contraception that only contains progestin. There are several types of contraception that are progestin-only: progestin-only implants, a monthly injection of only progestin, levonorgestrel based intrauterine device, and the progestin-only pills, which are also referred to as the "mini-pill." The estrogen/progestin combination oral contraception pill, the etonogestrel/ethinyl estradiol vaginal ring, the norelgestromin/ethinyl estradiol patch-transdermal, and the estradiol/medroxyprogesterone monthly injection all contain a form of estrogen, which use of this should be strictly avoided in patients such as the one described in the above scenario.

A 36-year-old woman presents for her annual gynecological examination. In addition to her routine testing, she is seeking advice on contraception. She currently takes no medication and has no known medical conditions. She smokes a 1/2 pack of cigarettes daily; she drinks 1 - 3 alcoholic drinks weekly, and she is currently sexually active with 2 partners. Her past PAP smears have all been normal. She is G2P1 and is uncertain whether she wants children in the future. Question What is your recommendation for contraception at this time? Answer Choices 1 Combination estrogen-progestin pill 2 Progestin-only pill 3 Ortho Evra (ethinyl estradiol/norelgestromin) patch 4 NuvaRing (etonogestrel/ethinyl estradiol) 5 Tubal ligation

Progestin-only pill Explanation Out of the above options, the patient should receive progestin-only pills. The patient is a smoker, which is a contraindication for estrogen containing products. The patient should be informed that she needs to take progestin-only pills at the same time daily; otherwise, she needs to use a back-up form of contraception. The patient cannot receive the combination estrogen-progestin pill, the Ortho Evra (ethinyl estradiol/norelgestromin) patch, or the NuvaRing (etonogestrel/ethinyl estradiol); they all contain estrogen. The provider should not recommend tubal ligation because the patient is uncertain whether she wants children in the future; tubal ligation is considered a permanent sterilization option.

Your patient is a 26-year-old mother of two young children whose second child was just delivered two weeks ago and she is breast-feeding. She complains of insomnia and depression. She denies the use of any drugs or alcohol. You would like to hold off on pharmacologic therapy unless her symptoms progress. Which of the following will be your first line of treatment? A Encourage more time with the baby B Encourage her to begin planning to get back to work C Promote adequate sleep D Suggest she get out of the house more often E Suggest her husband help more around the house

Promote adequate sleep C The correct answer is to promote adequate sleep (C). Postpartum "blues" are very common. Depression can occur in some cases, but most resolve without therapy. When required, SSRIs may be used even when a woman is breast feeding, though no studies have been done.

Your patient is a 26-year-old mother of two young children whose second child was just delivered two weeks ago and she is breast-feeding. She complains of insomnia and depression. She denies the use of any drugs or alcohol. You would like to hold off on pharmacologic therapy unless her symptoms progress. Which of the following will be your first line of treatment? A Encourage more time with the baby B Encourage her to begin planning to get back to work C Promote adequate sleep D Suggest she get out of the house more often E Suggest her husband help more around the house

Promote adequate sleep The correct answer is to promote adequate sleep (C). Postpartum "blues" are very common. Depression can occur in some cases, but most resolve without therapy. When required, SSRIs may be used even when a woman is breast feeding, though no studies have been done.

A woman brings her 13-year-old daughter to clinic, concerned about the fact that she has not yet had her first menstrual cycle. The patient is PMH negative, Social Hx negative, FMH: mothers' age of menarche was 13. Physical exam reveals the child to be a well-developed, well-nourished female, height and weight at the 50% for age and gender. Thelarche is present and has sparse pubic hair. The remainder of her physical exam is normal for age. The most appropriate course of action is? A Reevaluation in 6 months B Thyroid evaluation C Pelvic exam D Pelvic ultrasound E hCG

Reevaluation in 6 months A The child is in the expected age for puberty, and she shows signs of hormonal changes. Thelarche- onset of breast development

A 28-year-old G2P2 woman has just delivered a term male infant via normal spontaneous vaginal delivery. There was spontaneous rupture of the membranes 2 hours prior to delivery. Meconium stained amniotic fluid and thick particulate meconium was noted. The infant is brought under the radiant warmer where he appears peripherally cyanotic. He is crying and moving vigorously, and his heart rate is 160 beats per minute. Question What is the most appropriate next step in the resuscitation of this infant? Answer Choices 1 Bulb suction of the oropharynx and nasopharynx 2 Positive pressure ventilation via bag-valve mask 3 Removal of meconium by bulb suction 4 Routine neonatal evaluation since infant is vigorous 5 Suction the trachea via an endotracheal tube

Routine neonatal evaluation since infant is vigorous Explanation The recommendations have changed with the publication of the 2005 AHA/AAP Neonatal Resuscitation Program guidelines. Although the presence of meconium in the amniotic fluid should alert the caregiver to the possibility of neonatal distress, a vigorous infant is a good sign and further intervention is not warranted. Routine suctioning of the nose and mouth as the head is delivered is no longer recommended because it does not reduce the risk of meconium aspiration syndrome. In the presence of meconium stained fluid, suctioning is recommended if the infant has a low heart rate (<100 bpm), depressed respirations, or poor muscle tone. In an infant with one or more of these signs of distress, suctioning should be performed. Prior to drying, the infant should have visible, residual meconium removed from the nose and mouth by bulb suction. Then, under direct visualization, the trachea should be suctioned with an appropriately sized endotracheal tube. The infant in this case scenario is clearly vigorous and therefore does not require suctioning. He should only receive routine care (standard positioning, warming, etc.).

A 28-year-old pregnant woman who is currently at 33 weeks gestation is being evaluated for contractions at labor and delivery in the hospital. Laboratory testing on vaginal secretions from this patient reveals the following: Fetal fibronectin Positive Dried secretions on glass slide Ferning present Nitrazine paper Color change of yellow to blue Question What do these tests indicate? Answer Choices 1 Severe chorioamnionitis is present. 2 Fetal lungs are mature. 3 Rupture of membranes has taken place. 4 Estimated date of delivery (EDD) is confirmed. 5 The patient has a mild preeclampsia.

Rupture of membranes has taken place Explanation While direct observation of fluid leaking from the cervix is the best support, the above 3 tests (positive fetal fibronectin, ferning present on microscope slide and increased pH on nitrazine paper) indicate presence of amniotic fluid and thus rupture of membranes. Fetal fibronectin is not present in normal vaginal secretions. Normal vaginal pH during pregnancy is 4.5 - 4.7.1 However, amniotic fluid is 7 - 7.5, so after rupture of membranes, an alkaline pH is noted that causes corresponding color changes in the nitrazine paper. The laboratory testing that would most aid in a diagnosis of severe chorioamnionitis includes Gram stain for organisms and a white blood cell count of the amniotic fluid. 1 Often, the diagnosis is confirmed after pathology examination of the placenta. Fetal lung maturity is best evaluated with a lecithin to sphingomyelin (L:S) ratio. Some laboratories use different values for a level corresponding to fetal lung maturity (>/= to 2.0 or >3.0). A very low L:S ratio indicates fetal lungs are immature. 1 Confirmation of estimated date of delivery (EDD) cannot be accomplished with laboratory testing. Very early in a pregnancy, human chorionic gonadotropin (hCG) levels rise in a predictive manner in normal singleton, intrauterine pregnancy. 1 However, the level of hCG can neither be used to predict nor confirm the EDD. The best methods of confirming an accurate EDD include calculation based upon known date of last menstrual period (in a woman with regular menstrual cycles) and ultrasound in the 1st trimester.2 Preeclampsia is a syndrome in pregnancy consisting of hypertension and proteinuria. The testing that indicates a diagnosis of preeclampsia includes a blood pressure measurement and evaluation of the urine for protein, preferably from a 24-hour urine collection.3 None of the above listed tests on vaginal secretions indicate preeclampsia; furthermore, they do not indicate severity level.

A 51-year-old woman presents after discovering a new breast mass on her left side. It has not been tender, and she discovered it 2 weeks ago. Her last exam and mammogram were normal 2 years ago. Her past medical history is significant only for being perimenopausal. She takes no medications. Your physical exam confirms a firm, non-tender, fixed 1.5 cm mass in the left breast. Right breast exam is normal. Question What is the next most appropriate step for the management of this breast mass? Answer Choices 1 Refer for lumpectomy 2 Schedule a bilateral diagnostic mammogram 3 Order BRCA test 4 Reassure patient and re-examine in 6 months 5 Order follicle stimulating hormone (FSH)

Schedule a bilateral diagnostic mammogram The next most appropriate step in the management of this patient is to schedule a bilateral diagnostic mammogram because the mass has characteristics suspicious for malignant breast disease (such as being unilateral, non-tender, and fixed). Especially in patients over 40 years of age, the workup of a breast mass should include mammography. A mammogram can further characterize the breast mass and potentially detect additional smaller lesions and lymph node involvement that were not detected by physical exam. Ultrasonography may add additional diagnostic information in the evaluation of this mass. The imaging results can guide the next best treatment approach, such as fine-needle aspiration if the mass appears to be a simple cyst, or open biopsy with node sampling if the mass appears to be cancerous. Eventually, pathology on the tissue will be needed for definitive diagnosis. At this point, a referral for lumpectomy could have injurious effects regardless of whether the breast mass is benign in nature or if it is malignant and invasive. If the mass turns out to be a simple breast cyst, the patient would have been subjected to an invasive procedure when watchful waiting or needle-drainage may have sufficed. If the mass turns out to be invasive with lymph node involvement, the lumpectomy alone would have missed the opportunity to surgically sample lymph nodes, leading to an incorrect breast cancer staging or an additional surgery to remove additional cancer. BRCA testing is a serum test (for BRCA1 and BRCA2) to detect chromosomal mutations (on chromosomes 17 and 13) that are associated with an increased risk of breast and ovarian cancer. The results of a BRCA test will not influence the need to evaluate the symptomatic patient. Only about 5%-10% of breast cancer cases have a known genetic mutation. If the patient tests negative for BRCA, she still has a chance that her breast mass is malignant; conversely, a patient who tests positive for the BRCA mutation(s) may develop benign breast masses. Reassurance of the patient may be helpful, but because of the suspicious characteristics of the mass and the patient's age, delaying evaluation could be deleterious to her outcome. Further workup is warranted immediately. Checking the patient's FSH level might yield some information as to her ovarian status. The FSH rises as menopausal ovarian production of estrogen and progestin declines. However, the level does not directly influence management of the breast mass.

A 65-year-old man presents with a bulge in his lower abdomen that has been present for 5-10 years. He can usually push this bulge back, but, about 4 hours earlier, it came out and could not be pushed back. There is an aching pain, but no nausea or vomiting. He admits to straining at the stool and getting up twice a night to urinate. He smokes 1 pack of cigarettes per day. On physical examination, his vital signs are within normal limits; his pulse oxy is 91%. There is no jugular vein distention. Chest shows an increased A-P diameter, but breath sounds are good. Heart sounds are distant, and the rate and rhythm indicate normal sinus. There is an obvious bulge in the right lower quadrant extending into the right scrotal sac. The abdomen itself is soft, flat, and non-tender. Bowel sounds are slightly hyperactive; there are no rushes or tingling sounds. The bulge cannot be reduced and is tender. Bowel sounds are present within the mass. A somewhat enlarged, non-tender prostate is detected on rectal examination. Brown stool is obtained and is negative for blood. There is no abnormality on EKG. Chest x-ray shows some hyperinflation, but no infiltrates. An x-ray, flat and upright, of the abdomen shows some dilated small bowel extending into the right scrotal sac, but signs of obstruction are absent. Routine lab and urine studies are within normal limits. You start an intravenous line of 0.9 N/S to run at keep-open rate. Question What would be the next logical course of action? Answer Choices 1 Order an immediate surgical consultation 2 Admit the patient to the observation unit for 4 hours and reassess 3 Sedate the patient well and attempt to manually reduce the mass 4 Pass an intestinal tube to reduce small bowel distention 5 Pass a Foley catheter to reduce bladder distention

Sedate the patient well and attempt to manually reduce the mass The most likely diagnosis in this patient is incarcerated indirect hernia. Direct hernias do not usually incarcerate and come straight through the abdominal wall. Indirect hernias, by definition, come down the inguinal canal into the scrotal sac. Intestinal contents accompany the sac in either one. This patient's lung signs and signs of prostate enlargement are both impediments to emergency surgery, and they are additional reasons to delay repair to a more elective time if possible. Every reasonable attempt should be made to reduce this hernia using manual compression after adequate sedation. A warm compress on the area may help reduce muscle spasm and relax the abdominal musculature. There are times when either a nasogastric tube and/or a long intestinal tube are warranted. This patient has no signs of intestinal obstruction (vomiting, constipation, etc.). The minimal dilatation of the small bowel is an accompanying sign of an incarcerated hernia. An obstructed segment of bowel may become incarcerated and/or strangulated; when reduced or replaced in the peritoneum, signs and symptoms of mechanical obstruction occur. Of course, should it not be possible to reduce the hernia, obstruction and strangulation both may develop. Strangulation occurs when blood flow (venous, arterial, or both) is closed off, and the bowel segment dies. Then an emergency exists, and repair must occur upon recognition. A Foley catheter has no value unless there is urinary outlet obstruction.

A 54 year-old female returns for pathology results after being diagnosed with ovarian cancer. Which of the following is the most likely cause of ovarian epithelial malignancy? A Serous tumors B Endometrioid tumors C Mucinous tumors D Clear cell tumors E Transitional cell tumors

Serous Tumors A The most common of the ovarian epithelial malignancies are serous tumors (50%); tumors of mucinous (25%) (C), endometrioid (15%) (B), clear cell (5%) (D), and transitional cell (E) histology or Brenner tumors (1%) represent smaller proportions of epithelial ovarian tumors. Over half of all epithelial ovarian cancers have serous histology. The second most common histologic type of epithelial ovarian cancers are endometrioid adenocarcinomas.

The patient is a 19-year-old woman who presents due to lower abdominal pain, chills, and vaginal discharge. She admits to being sexually active and has a history of irregular menstruation. Her pelvic examination reveals adnexal and cervical motion tenderness, as well as purulent discharge seen at cervical os. Question What other factor in the patient's case would influence you to admit her for inpatient intravenous antibiotic therapy as opposed to outpatient antibiotic therapy? Answer Choices 1 Oral temperature of 100.8 ° F 2 Lower abdominal tenderness on exam 3 Serum β-hCG level of 28,000 mIU/mL 4 Evidence of cervical infection with N. gonorrhoeae 5 Elevated C-reactive protein

Serum B-hCG level of 28,000 mIU/mL Explanation Given the patient's history and physical examination, the most likely diagnosis is pelvic inflammatory disease (PID). Most cases of PID are treated on an outpatient basis, but there are certain criteria that warrant inpatient intravenous antibiotics. One of the indications for hospitalization is pregnancy, and a serum β-hCG of 28,000 mIU/mL is laboratory evidence that this patient is at least 5 weeks pregnant. In addition, the chance for ectopic pregnancy as a cause of the symptoms and elevated serum β-hCG still exists. Ectopic pregnancy is treated surgically, so this would be an added reason to treat the patient as an inpatient. Oral temperature of 100.8°F is not abnormal for a patient with PID. In fact, one of the additional criteria used to diagnose PID is an oral temperature over 101°F. If a patient has a high fever associated with the disease, then the health care provider may opt to treat as an inpatient. High fever may be a sign that there is a pelvic abscess or another undiagnosed surgical emergency, such as appendicitis. Lower abdominal tenderness is a symptom of PID and not a criterion for admission. N. gonorrhoeae can be found in the cervical culture of a patient with PID, and a C-reactive protein can be elevated. Neither is necessary for diagnosis, but both are common in patients with PID and are not reasons for inpatient treatment in themselves.

A sexually active, 17-year-old patient presents alone to your office and requests contraceptive counseling. Because she is under the age of consent, what is an ethical concern of which you must be aware in seeing her? Answer Choices 1 You are obligated to refuse to see her. 2 You may only see her with parental consent. 3 You do not need parental consent if she is accompanied by an adult. 4 She can give consent for contraceptive counseling and evaluation of STDs. 5 You must counsel abstinence as contraception at her age.

She can give consent for contraceptive counseling and evaluation of STDs. Explanation This case highlights several ethical issues. Autonomy indicates that an individual may make his/her own health care decisions based on their own values. It is usually limited by a determination of whether the individual is competent and, in pediatrics, at least in part determined by the age of the individual. Competence is an individual's ability to understand the possible consequences of his/her decision and the available alternatives. Patients are generally considered to have a right to confidentiality, i.e., trust that information about them will not be disclosed. Depending on the age of the patient, there may be a conflict of interest between the physician's duty to the patient and the physician's duty to the parents. States either have laws allowing individuals who would otherwise be considered minors to obtain contraceptive services or have no law precluding the provision of such services.However, most states have placed the interests of the adolescents above those of the parents in issues relating to the management of sexually transmitted diseases and other issues of sexuality. Physicians have a duty to tell the truth to their patients. This duty can create quite a conflict when other family members, especially parents, request information about another family member that the physician is obligated to keep confidential.

A 35-year-old Caucasian female patient who has been diagnosed with cervical cancer receives chemotherapy as an outpatient. You send her home with a prescription for metoclopramide. What do you need to tell her about the drug? Answer Choices 1 Nausea and vomiting are common side effects 2 She may experience dry mouth 3 She may experience drowsiness 4 The drug should be taken after meals 5 If she misses a dose to just skip it and go back to schedule with the next dose

She may experience drowsiness Explanation Drowsiness is a known side effect of metoclopramide, therefore you need to tell the patient not to operate equipment before she knows her response to the drug. Nausea and vomiting are the indications for prescribing the drug. The effectiveness of the drug is not affected by food. Since nausea and vomiting often occur with food intake, you should advise the patient to take the medication before meals. Doses of metoclopramide should not be skipped. If forgotten it should be taken as soon as possible, except if it is already time for the next dose. As metoclopramide is an anti-emetic which acts directly on the gastrointestinal tract there is no known side effect of dry mouth.

A 40-year-old white male patient presents to your office to ask for advice regarding the inability of his wife to conceive. She is younger than he is by eight years and is believed by him to be in good health. Additionally, she has two children from a previous marriage, and both pregnancies were without complications. He was also married once before but his first wife did not conceive children. He has not yet undergone any fertility tests. The most important factor concerning male fertility is Answer Choices 1 Lung and liver dysfunctions 2 Spermatogenesis, varicocele, infections, drugs, and heat 3 Mucous and sperm interactions 4 Endometrial cavity and shape 5 The size of the genital tract 6 Personal and social history as it regards smoking, drugs and eating habits 7 Past infections of the male genitalia

Spermatogenesis, varicocele, infections, drugs, and heat Explanation Factors involved in fertility: The male factor---Spermatogenesis, infections (prostatitis, epididymis, mumps) varicocele, heat (decrease sperm count and motility) drugs (marijuana, tobacco, alcohol). The ovarian factor---Ovulation The cervical factors---Mucus and sperm interaction The uterine factor---Endometrial integrity and cavity size and shape The tubal factor---oviductal potency and anatomic relationships to the ovary The coital factor---Insemination

A newborn boy was observed to have a small cyst located midline in the lumbar region of his back. No neurological symptoms were observed, and the sac contained only meninges and cerebral spinal fluid. X-ray revealed the absence of a vertebral arch at L5. What is this type of anomaly? Answer Choices 1 Spina bifida with meningocele 2 Spina bifida occulta 3 Spina bifida with meningomyelocele 4 Spina bifida with myeloschisis 5 Spinal dermal sinus

Spina bifida meningocele Explanation The failure of the 2 halves of the vertebral arches to fuse during development results in an anomaly referred to as spina bifida. The most minor, clinically insignificant type (spina bifida occulta) is asymptomatic and may be revealed only with imaging of the vertebra involved. Sometimes a small dimple or tuft of hair overlying the defect may be present at the level of the malformation, usually in the lumbar or sacral regions. More serious anomalies, categorized as spina bifida cystica, show a cyst-like protrusion at the level of the defect. When the cyst or sac contains meninges and cerebrospinal fluid, it is referred to as spina bifida with meningocele. If the spinal cord and/or nerve roots are included with the meninges and CSF in the sac, it is referred to as spina bifida with meningomyelocele. The most severe type of vertebral arch defect is one in which not only did the arches fail to fuse in the midline, but the neural plate failed to fold and fuse at the level of the anomaly, resulting in a flattened plate of nervous tissue, called spina bifida with myeloschisis. Spina bifida cystica show varying degrees of neurological deficits.

A 26-year-old woman who is nursing presents to clinic complaining of 2 to 3 days of increasing pain and redness in her left breast. She continues to feed her infant and has good milk supply. She is 4 weeks postpartum. She complains of some general fatigue, but no headaches, body aches, or fever. On physical exam, results are as follows: P 80, T 99.1°F, and BP 120/70, CV RRR, and lungs CTA. Her left breast has a 5-cm area of induration, in the upper outer quadrant a 2-cm mass is noted, which is tender to the touch. In addition, milk expressed is non-purulent. What is the most likely cause of the infection? A Staphylococcus aureus B Streptococcus viridians C Blastomycosis D Sporotrichosis E Streptococcus pyogenes

Staph aureus A The discrete nodules indicate a breast abscess, which can commonly occur in nursing women. Fungal infections can occur, but are rare, and Streptococcal infections are not as likely as staph infections.

A 23-year-old Asian woman presents for follow-up after being seen in the emergency department. She was seen 2 months ago in the emergency department for a 3.2 cm simple ovarian cyst, which had resolved on repeat ultrasound in 6 weeks. She reports that she had severe pain with the cyst, but all of her symptoms resolved after cyst resolution. However, she had the same experience about 18 months prior to that due to another simple ovarian cyst of 3.5 cm, and her severe pain resolved after cyst resolution. She is not sexually active, and she does not use contraception. She has no chronic medical conditions, and she takes no medications. Medical records confirm her history. The physical exam is unremarkable, showing a patient of normal body habitus, without acne and hirsutism. She is very interested in preventing future recurrences of these painful ovarian cysts. Question What should be done for this patient who wishes to prevent future ovarian cysts? Answer Choices 1 Insert a levonorgestrel intrauterine system (Mirena) 2 Perform monthly ovarian ultrasound 3 Start her on a folic acid supplement 4 Start her on a moderate-dose combined oral contraceptive pill 5 Test her CA-125 levels

Start her on a moderate-dose combined oral contraceptive pill Explanation This patient's described ovarian cysts are simple and related to follicular development. Moderate-dose combined oral contraceptives have been shown to reduce ovarian cysts in population studies. Although a mechanism of oral contraceptives is suppression of ovulation, birth control pills do not treatexisting ovarian cysts. Furthermore, the lower-dose regimens on the market currently do not appear to lower the overall incidence of ovarian cysts and achieve prevention of cysts. Starting this patient on a moderate-dose combined oral contraceptive pill is reasonable, especially if indicated for prevention of pregnancy or other reasons. Performing a monthly ultrasound may identify early cysts, but doing so will not prevent them. Ultrasounds often show incidental functional cysts in the ovaries of asymptomatic women. Ultrasound would be indicated in a symptomatic patient or to follow a concerning cyst to resolution. Once resolved, this imaging is neither necessary nor cost-efficient. Insertion of a levonorgestrel intrauterine system (Mirena) is indicated for prevention of pregnancy and menorrhagia. The primary effect of the progestin, levonorgestrel, is intrauterine; therefore, ovarian function is generally unaffected. A levonorgestrel intrauterine system would not reduce ovarian cyst recurrence. Folic acid supplementation has long been recommended for women with cervical dysplasia, but few studies have actually confirmed that supplementation reverses dysplasia. Furthermore, there are neither recommendations nor evidence for folic acid supplementation preventing ovarian cysts. Testing for CA-125 should be done in women with current ovarian cancer or an ovarian mass suggestive of an ovarian cancer. This patient denies current symptoms; she is young and described simple ovarian cysts, which resolved.

A 44-year-old woman G5P5 presents for her annual pelvic examination. Her menses are regular, and she is currently mid-cycle. She notes 'leaking urine' when she coughs, sneezes, or strains. There is a bulge into the anterior vaginal wall; it is exacerbated when the patient is asked to 'bear down'. A urinalysis is unremarkable. There is no cervical motion tenderness or discharge noted on pelvic examination. The patient is afebrile and in no distress. No lesions are noted on the external genitalia and the pelvic examination is unremarkable except for the bulge noted previously. Vaginal cultures for gonococcus (GC) and chlamydia are pending. Question What is the most likely diagnosis? Answer Choices 1 Bladder infection 2 Menopause 3 Multiple sclerosis 4 Pelvic inflammatory disease 5 Stress urinary incontinence

Stress urinary incontinence Incontinence and incomplete emptying of the bladder are both common symptoms in patients with multiple sclerosis because the disease involves a dysfunction of muscle coordination and strength. The patient does not give a history of previous symptoms suggestive of MS, but a workup may be warranted if symptoms worsen or persist without another obvious etiology. The presence of an anatomical defect suggests incontinence resulting from a cystocele in this patient. Pelvic inflammatory disease may also cause dysuria, often in association with cervical wall motion tenderness. GC and chlamydia cultures are pending; if there is a suspicion of PID, empiric treatment should be considered. There are no lesions consistent with herpes or chanchroid, and the pelvic examination does not suggest the presence of an infection.

At her routine annual pelvic exam, a 39-year-old female presents to the clinic complaining of pelvic pressure and bloating for several months. She is a G3P2 who delivered vaginally. She is a nonsmoker. Her maternal aunt had a history of ovarian cancer. Her pelvic exam reveals an 8-cm ovarian mass in the right adnexal area. What is the most appropriate evaluation of the ovarian mass? A Pelvic ultrasound B Pelvic CT scan C Surgical evaluation D CA 125 level E Repeat pelvic exam 1 month

Surgical evaluation C The patient is high risk, as she is premenopausal, has a family history of cancer, and the mass is large. Therefore, surgical evaluation should be undertaken. CA 125 can be negative in early disease, and pelvic US and CT are not sensitive enough. Repeat examination should be reserved for low risk women with smaller ovarian masses.

A 65-year-old female presents to clinic for her annual pap smear. She is in good health but has mild hyperlipidemia, which is controlled with diet. She had a hysterectomy more than 10 years ago for dysfunctional uterine bleeding. How often should she have a pap smear? A Annually B Every 2 years C Every 3 years D Symptomatically

Symptomatically D According to the new 2010 ACOG guidelines, women who have no high-grade lesions or cervical cancer history, and are over 65, may discontinue cervical cancer screening due to the decrease risk and slow progression if disease does occur.

A 38-year-old man presents with a 2-day history of a mass and severe pain in his scrotum. Physical examination reveals that his right testicle appears much larger than his left. On palpation, you note a small hole in his inguinal canal, and you are unable to place the contents into the canal. The contents of the hernia appear ischemic. What is the best description of the hernia? Answer Choices 1 Reducible 2 Recurrent 3 Irreducible 4 Strangulated 5 Incarcerated

The clinical picture is suggestive of an inguinal hernia. The hernia has become ischemic, which is better known as a strangulated hernia. In a reducible hernia, the contents would be able to be placed back into the abdominal cavity with simple manipulation. A recurrent hernia is a hernia that has been previously repaired surgically and has now returned. There is no history of a previous hernia in this patient. An irreducible (also known as incarcerated) hernia is when the hernial contents cannot be returned to their normal site with simple manipulation. This type of hernia would cause edema and entrapment due to impaired venous return. An irreducible (also known as incarcerated) hernia is when the hernial contents cannot be returned to their normal site with simple manipulation. This type of hernia would cause edema and entrapment due to impaired venous return.

A 42-year-old gravida 1, para 1 presents with a 4-month history of menorrhagia. She is having shortened menstrual cycles that are sometimes only 15 days in length, with menstrual bleeding for 5 - 6 days. She is using approximately 12 - 14 pads or tampons per day. She admits to fatigue, headaches, and occasional dizziness, but denies syncope. There is no dysmenorrhea. A thyroid-stimulating test last month was within normal limits. Abdominal and pelvic exams are normal. What statement is true regarding dysfunctional uterine bleeding? Answer Choices 1 It is seen in all age groups equally 2 Dilatation and curettage is the only treatment 3 The condition is most common after 40 years of age 4 The condition is not seen in adolescents 5 It is never associated with ovulatory cycles

The condition is most common after 40 years of age Explanation Dysfunctional uterine bleeding (DUB) is a condition of irregular uterine bleeding in a patient who does not have an anatomic uterine lesion. It is most common above the age of 40 years (50% of the cases), but it is also seen in adolescents (20%), in whom it is associated with anovulatory cycles. Anovulatory cycles are characterized by abnormal levels of estrogen and may be due to estrogen withdrawal or breakthrough. A deteriorating ovarian follicular function is responsible for anovulatory bleeding during the climacteric. Other etiologies, such as polycystic ovarian disease, fibroids, and thyroid disease need to be ruled out before making the diagnosis. An endometrial biopsy or dilation and curettage (if the patient cannot undergo an endometrial biopsy in the office) can be diagnostic, but it is not curative or even therapeutic in a patient with DUB. Medical therapy, including estrogens, progestational agents, progesterone-impregnated intrauterine devices, and combination oral contraceptives are used to treat the condition.

A 45-year-old Caucasian woman presents with a 2-month history of a lump in her right breast. She has had no children and has been on oral contraceptive pills for 20 years. Her menstrual periods began at the age of 15 and are still regular. She is a successful business woman who does not smoke cigarettes and drinks 6 cups of coffee per day. On examination, she has a nontender lump in the upper outer quadrant of her right breast. Several matted right axillary lymph nodes are also palpable. Question Given her history, what is her greatest risk factor for developing breast cancer? Answer Choices 1 Use of oral contraceptive pills 2 Menarche at 15 years of age 3 Menopause at age 45 years of age 4 Excessive caffeine consumption 5 Nulliparity

The correct response is nulliparity. The clinical features are suggestive of breast cancer. Although it occurs mainly in women, a small percentage develops in men. Breast cancer has no single cause, but multiple risk factors have been identified. These include: Family history of premenopausal breast cancer in a first-degree relative like one's mother, daughter, or sister. Having the first full-term pregnancy after the age of 30 Prior personal history of breast cancer Chest exposure to high dose of ionizing radiation Nulliparity Early menarche at or before 12 years Late menopause after 50 years Current or recent use of oral contraceptive pills (for women who stopped using oral contraceptive pills 10+ years ago, there is no apparent increase in risk).

A 20-year-old woman (G1, P0) presents for her 28th week antenatal care visit. The woman's blood group is O and is Rh-. Her husband's blood type is B and is Rh+. Her indirect Coombs test in the first antenatal care visit was negative; however, the indirect Coombs test at 28 weeks is positive. What does the mother's positive indirect Coombs test mean? Answer Choices 1 The fetus is Rh- 2 The fetus has antibodies against the mother's red cells 3 The mother has formed antibodies against Rh antigens 4 The mother has an autoimmune disorder 5 The mother is at risk for hemolysis

The mother has formed antibodies against Rh antigensExplanation The mother's positive indirect Coombs test means that she has formed antibodies against Rh antigens. The fact that the mother is Rh- and that the father is Rh+ means that the likelihood that the mother will be carrying an Rh+ baby can be either 50% or 100% depending on whether the father is heterozygous or homozygous for the Rh gene locus respectively. The antenatal screening of the mother at 28 weeks gestation was negative (showing anti-Rh antibodies), indicating her alloimmunization. The indirect Coombs test evaluates for the presence of circulating antibodies in the mother's serum and in this case showed the presence of anti-Rh antibodies in the mother's blood. Since the mother's Coombs test was negative initially and then positive at 28 weeks of her pregnancy, indicating that her child is Rh+ and has transported some of its red cells to her circulation, likely through a minor fetomaternal hemorrhage, which is most likely to occur in third trimester. Upon encounter of the Rh+ red blood cells, the mother formed anti-Rh antibodies in her blood, which were detected by the indirect Coombs test. The presence of the anti-Rh antibodies in the mother does not mean that she is at risk for hemolysis since her red cells are Rh-. The presence of an antibody in the mother's blood against a foreign antigen upon exposure to it is a normal immune response and does not represent an autoimmune disorder. The indirect Coombs test is performed on the mother and therefore can only show the presence or absence of the antibody under study (in this case anti-Rh) in the mother's blood.

A patient presents to the office at 5 weeks gestation. She has been spotting for several days, and the flow is increasing slightly. She has mild, crampy pain; no fetal heart tones are auscultated. Ultrasound reveals an intrauterine gestational sac with a fetal pole. What is the most likely diagnosis? A Threatened abortion B Spontaneous abortion C Incomplete abortion D Septic abortion

Threatened abortion A While she is at risk for a spontaneous abortion, one cannot determine fetal viability with one exam. Serial exams are necessary. Fetal heart tones are not always present at 5 weeks.

An 18-year-old woman comes in for evaluation of a bad smelling vaginal discharge. She says that it is so heavy that she must wear a pad to absorb it and that it is "yellowy green and bubbly." She also has a great deal of itching of vulva. Evaluation confirms the presence of a discharge and a vaginal pH of 5.5. Her vagina and cervix are diffusely erythematous with scattered petechiae. What is the most likely diagnosis? A Atrophic vaginitis B Bacterial vaginosis C Candidiasis D Gonorrhea E Trichomoniasis

Trichomoniasis E Trichomoniasis vaginitis is the most common STD in the United States that is not caused by a virus. It causes a profuse and frothy discharge that is greenish and can be foul smelling. Pruritus may occur. The vaginal pH is usually more than 5.0 and the vagina and cervix are red and may demonstrate small petechiae. Atrophic vaginitis (A) occurs primarily in postmenopausal women and is characterized by a thin watery or serosanguinous discharge, while bacterial vaginosis (B) has a greyish homogeneous discharge with a characteristic "fishy" odor released when potassium hydroxide is mixed with vaginal secretions. The discharge of candidiasis (C) is typically white and clumpy and is accompanied by intense itching in the vulva. More than three quarters of women who have gonorrhea (D) have no symptoms, but they may have a mucopurulent discharge.

A 30-year-old G1P0 woman who is 15 weeks pregnant undergoes "triple screening." The maternal alphafetoprotein (AFP), human chorionic gonadotropin (hCG), and unconjugated estriol (uE3) levels are all lower than normal. This suggests which of the following possible problems with the fetus? A cystic fibrosis B Down syndrome C homocystinuria D G6PD deficiency E trisomy 18

Trisomy 18 E The triple screen detects possible Down syndrome and trisomy 18. When the fetus has Down syndrome (B), the AFP and uE3 are low and the hCG is high, while trisomy 18 is suggested by low values in all three. A positive screen must be followed up by fetal karyotyping. Cystic fibrosis (A), homocystinuria (C), and G6PD deficiency (D) are not detected by the triple screen.

A 16-year-old sexually active girl is seen for a 2-month history of amenorrhea. She denies having unprotected sex, but always relies upon her partner to use a condom. She has vomited in the early morning twice in the past week. She has also had vaginal spotting for 3 days, accompanied by cramping lower abdominal pain that became sharp. Onset of menses was at 12 years, with normal, regular periods since then. There is no history of sexually transmitted disease. Physical examination revealed normal vital signs. Slight right and left lower quadrant abdominal tenderness without guarding and rebound was present. The cervix was closed. No blood was seen in the vaginal vault. The uterus was not palpable. Serum β-HCG: 5,200 mIU/ml. Vaginal spotting has increased, and abdominal pain has become more frequent. Repeat examination 3 days after the initial visit is unchanged. The uterus is still not palpable. Repeat serum β-HCG is 6,800 mIU/ml. Transvaginal ultrasound failed to reveal an intrauterine pregnancy or gestational sac. Question What is the most likely diagnosis? Answer Choices 1 Cervical ectopic pregnancy 2 Choriocarcinoma 3 Pseudocyesis 4 Tubal ectopic pregnancy 5 Very early intrauterine gestation

Tubal ectopic pregnancy Explanation The absence of fetal pole or gestational sac in the uterus with elevated serum β-HCG indicates an ectopic pregnancy, of which a tubal ectopic is the most common. The full triad of abdominal/pelvic pain, amenorrhea and irregular vaginal bleeding is present in only one-half of patients. Abdominal/pelvic pain may be unilateral or bilateral, usually worse on the affected side. Cervical ectopic pregnancies are rare. Serum β-HCG is not adequately elevated to suggest choriocarcinoma. Serum β-HCG is not elevated in pseudocyesis (false pregnancy), a condition in which the female patient truly believes she is pregnant and may exhibit signs and symptoms of true pregnancy such as amenorrhea, hyperemesis, breast swelling and tenderness, weight gain, abdominal tenderness, and even quickening. Transvaginal ultrasound failed to reveal even a fetal pole or gestational sac in the uterus, which should be detected by 36 to 40 days.

A 19-year-old female patient presents to her family practice office for her annual Pap test and her first dose of the quadravalent human papillomavirus (HPV) vaccine. As her physician assistant, when would you schedule her to come in for her second dose of the HPV vaccine? A two weeks B two months C three months D four months E six months

Two months B People are frequently confused by differences between the quadravalent and bivalent versions of this vaccine. But dosing schedules are not one of the confusing issues. The Centers for Disease Control and Prevention (CDC), Advisory Committee on Immunization Practices (ACIP), and the manufacturers of both the HPV4 and the HPV2 vaccines all agree: The dosing and administration schedules are the same for HPV4 and HPV2. Each dose is 0.5 mL, administered intramuscularly, preferably in a deltoid muscle. The vaccines are administered in a three-dose schedule. The second dose is administered one to two months after the first dose, and the third dose is administered six months after the first dose.

A 29-year-old man is seen in clinic for groin pain. He works as a cashier at a local supermarket. The pain increases through the day. When asked to point out its location, he localizes it to his left groin fold. Examination of his inguinal region reveals an ill-defined tortuous swelling that increases with standing and coughing. Question What is the best next step in diagnosis? Answer Choices 1 Diagnostic laparoscopy 2 Ultrasound 3 Venography 4 Clinical exam 5 MRI

Ultrasound Ultrasonography is the correct answer. Ultrasound reliably diagnoses varicoceles. Being quick, reliable and non-invasive, it is the diagnostic test of choice. Sensitivity increases with the concomitant use of Doppler. Pampiniform plexus venous dilation of over 2 mm with standing or Valsalva maneuver is considered diagnostic. Diagnostic laparoscopy is incorrect. The pathology lies in the inguinal region extending to the scrotum, and it does not need laparoscopy. Venography is incorrect. While it is highly accurate, it remains an invasive technique; there is exposure to ionizing radiation. It is rarely (if ever) necessary in the age of ultrasonography. Clinical exam is incorrect. It has been shown to miss the diagnosis of smaller varices, particularly when they are bilateral. MRI is incorrect. It is unnecessary because ultrasonography is of such high sensitivity.

A 25-year-old gravida 1 woman who is HIV positive arrives at the hospital in early labor. Membranes are intact and the cervix is 50% effaced and 3 to 4 cm dilated. Fetal heart rate is 150 beats/min. Which of the following procedures is contraindicated during labor? A amniotomy B augmentation of labor with oxytocin C external monitoring D operative delivery E use of fetal scalp electrodes

Use of fetal scalp electrodes E Use of fetal scalp electrodes and scalp sampling is contraindicated in the HIV-positive woman because it increases the risk of vertical transmission of the human immunodeficiency virus to the infant. While ruptured membranes for more than 4 hours is associated with an increased risk of vertical transmission, amniotomy per se is not contraindicated. Augmentation of labor, external monitoring, and operative delivery are not contraindicated and, in fact, may be indicated in specific instances for the well-being of the infant and/or mother.

A 23-year-old primiparous woman with an estimated gestational age of 27 weeks presents following a motor vehicle accident. Another car struck the vehicle that she was driving on the passenger side at 40 mph. She was wearing her seat belt. She cannot recall striking her head, but her chest hurts where the belt was, and her chin is scraped from the air bag. Review of her prenatal charts shows that her laboratory work is up to date. Her Rh factor is negative and her initial antibody screen was negative. She has gained 18 pounds during the pregnancy, and her blood pressure and urine dipstick readings have been normal. Her dates were confirmed by an ultrasound done at 12 weeks, and the fundal height measurements have fit with the dates throughout the pregnancy. Her physical exam reveals a blood pressure of 130/85 mm Hg, pulse of 96 BPM, respirations of 16/minute, and temperature of 98 °F (36 °C). There is an irregular abrasion on the chin. All extremities move fully without pain. The fundal height is 28 cm and the abdomen is non-tender. The fetal head is ballotable above the symphysis. Question What would be a major warning sign of minor trauma in this patient? Answer Choices 1 Fetal heart rate of 150 beats/minute 2 Presence of short and long-term fetal heart rate variability 3 2 fetal heart rate accelerations of about 22 beats per minute within a 20 minute strip 4 2 uterine contractions per hour 5 Vaginal bleeding

Vaginal bleeding Explanation Pregnant women who are involved in motor vehicle accidents need to be evaluated for the likelihood of injury that could have adverse effects on the pregnancy. The workup should include a careful physical exam and lab testing to check for maternal or fetomaternal hemorrhage. Rhesus-negative mothers may need Rh° (D) immune globulin. The most important component of the evaluation is fetal heart monitoring of at least 4 hours duration. After this initial monitoring period, if the mother is stable and has no specified warning signs, she may be discharged. Vaginal bleeding may be indicative of placental abruption and doing a speculum examination to evaluate for the presence of amniotic fluid and/or blood is prudent. An urgent ultrasound to assess for abruption is not likely to be helpful (sensitivity is 40% and specificity is 25). Differentiating between normal 'venous lakes' and extravasation, indicating abruption, is difficult. Frequent uterine contractions with or without vaginal bleeding are a good indicator; later, a detailed biophysical profile might be helpful if initial cardiac monitoring is equivocal. In a recent related study, frequent uterine contractions of greater than 4 contractions/hr are associated with poorer outcomes (especially within the initial 4 hours after trauma). Those with fewer than 4 uterine contractions per hour had the same live birth rates and Apgar scores as the control patients. Evidence of a reactive fetal strip would include the presence of short and long-term beat-to-beat variability, and the presence of at least 2 accelerations in fetal heart rate of at least 20 beats/minute within a 20 minute strip. In addition, a fetal heart rate of between 160/minute and 120/minute (above or below which would be classed as a fetal tachycardia and fetal bradycardia, respectively). Other warning signs would include the presence of abdominal tenderness and the presence of fetal decelerations, especially late decelerations.

a 23-year-old primiparous woman with an estimated gestational age of 27 weeks presents following a motor vehicle accident. Another car struck the vehicle that she was driving on the passenger side at 40 mph. She was wearing her seat belt. She cannot recall striking her head, but her chest hurts where the belt was, and her chin is scraped from the air bag. Review of her prenatal charts shows that her laboratory work is up to date. Her Rh factor is negative and her initial antibody screen was negative. She has gained 18 pounds during the pregnancy, and her blood pressure and urine dipstick readings have been normal. Her dates were confirmed by an ultrasound done at 12 weeks, and the fundal height measurements have fit with the dates throughout the pregnancy. Her physical exam reveals a blood pressure of 130/85 mm Hg, pulse of 96 BPM, respirations of 16/minute, and temperature of 98 °F (36 °C). There is an irregular abrasion on the chin. All extremities move fully without pain. The fundal height is 28 cm and the abdomen is non-tender. The fetal head is ballotable above the symphysis. Question What would be a major warning sign of minor trauma in this patient? Answer Choices 1 Fetal heart rate of 150 beats/minute 2 Presence of short and long-term fetal heart rate variability 3 2 fetal heart rate accelerations of about 22 beats per minute within a 20 minute strip 4 2 uterine contractions per hour 5 Vaginal bleeding

Vaginal bleeding may be indicative of placental abruption and doing a speculum examination to evaluate for the presence of amniotic fluid and/or blood is prudent. An urgent ultrasound to assess for abruption is not likely to be helpful (sensitivity is 40% and specificity is 25). Differentiating between normal 'venous lakes' and extravasation, indicating abruption, is difficult. Frequent uterine contractions with or without vaginal bleeding are a good indicator; later, a detailed biophysical profile might be helpful if initial cardiac monitoring is equivocal.

A 62-year-old diabetic hypertensive woman is evaluated for incontinence. She is found to have a cystocele and grade 3 uterine prolapse. Her diabetes is poorly controlled; so is her hypertension. On examination, the uterus can be repositioned with firm digital pressure. In addition to being declared unfit for surgery, she is not keen on undergoing any procedure. She has no history of any urinary infections or pelvic inflammatory disease. Question What is the next best step in management? Answer Choices 1 Anticholinergic agents to reduce bladder reactivity 2 Vaginal pessary placement 3 Laparoscopic ventral suspension of the uterus 4 Topical estrogen creams 5 Daily manual replacement of uterus

Vaginal pessary placement Explanation Vaginal pessary placement is the correct answer. Vaginal pessaries are the mainstay of conservative management of uterine prolapse. Pessaries are manually placed around the cervix, which prevents the descent of the uterus. Acute pelvic inflammatory disease and recurrent vaginitis are the only real contraindications to pessary placement. Anticholinergic agents is incorrect. Anticholinergic agents are used in the management of overactive bladders and act by reducing the bladder tone. Cholinergic nerve supply is responsible for detrusor contraction; thus, anticholinergics are useful in reducing bladder tone. Laparoscopic ventral suspension of the uterus is incorrect. While the treatment of choice for symptomatic prolapse is surgery, it is contraindicated in patients that are unfit for surgery, as is this patient. Topical estrogen creams is incorrect. It does not help uterine prolapse. Estrogen creams are useful in treating vaginal dryness and atrophic vaginitis.

A progestin-only contraceptive, or "minipill," would be most appropriate for which of the following patients? A a 25-year-old woman in excellent overall health B a 28-year-old woman with a history of epilepsy C a 32-year-old woman with a history of pelvic inflammatory disease D a 37-year-old woman who smokes 2 packs per day and has a history of hypertension E a 38-year-old woman with a history of asthma and bronchitis

a 37-year-old woman who smokes 2 packs per day and has a history of hypertension In the majority of cases, a combined hormonal contraceptive (ie, one that contains both an estrogen and progestin) is the preferred method of oral contraception because of its efficacy when used perfectly (>99%). However, for women older than 35 years of age who are smokers or are obese, or who have a history of hypertension or vascular disease, progesterone-only contraceptives are recommended. Ethinyl estradiol (EE), the most common estrogen found in combined hormonal contraceptives, has been associated with an increased risk of myocardial infarction in women older than 35 years of age who are smokers. Additionally, EE has also been shown to cause increases in blood pressure in both normotensive and mildly hypertensive women. Progestin-only contraceptives, however, tend to be less effective than the combined hormonal contraceptives.

A 28-year-old G1P0 at 37.5 weeks gestation complains of a thin, watery discharge for the last 5 hours. She has soaked 3 pads. She has no pain and the fetus continues to be active. An external fetal monitor reveals heart tones in the 140s, with variability and no contractions. The pH of the vaginal fluids is 8. What does this indicate? A Normal vaginal secretions B Amniotic fluid C Yeast vaginitis D Bacterial vaginitis E Urine

amniotic fluid B Premature rupture of membranes is the rupture of membranes before the onset of labor (within 2 hours); preterm rupture is the rupture prior to 37 weeks gestation. The absence of contraction on the monitor, in addition to no complaints indicates no labor. Urinary incontinence is common, but the ph of 8 indicates amniotic fluid.

A 37-year-old female presents to the labor and delivery department complaining of intermittent pain and contractions. Upon arrival, she also complains of vaginal bleeding. She is a G3P2 at 39 weeks gestation; no other prenatal complications are noted. She is a non-smoker. A physical exam reveals the following: P 90, BP 130/80, T 98.7°F, abdomen gravid, positive bowel sounds, and left lower quadrant tenderness noted. A sterile speculum exam reveals the cervix to be dilated 8, fetus is cephalic, and membranes are intact. The fetal monitor reveals heart tones in the 140s with mild, decreased variability and good quality contractions noted. Delivery is felt to be imminent, and vaginal delivery has been determined to be the best course of action. What will likely decrease bleeding and shorten time to delivery? A Increased activity level B Amniotomy C Oxytocin therapy D Epidural placement E IV sedation

amniotomy If the fetus is mature and vaginal delivery (versus c-section) has been determined to be the best course of action, then amniotomy may diminished amnionic fluid volume. This might also allow for better spiral artery compression, and serve to both decrease bleeding from the implantation site and reduce entry of thromboplastin into the maternal circulation.

A 22-year-old female presents to her obstetrical appointment at 39 weeks gestation. Her pregnancy to date has been uncomplicated. She is concerned that her infant may be larger than average, as her fundal height measures 41. On physical exam, her fetus is in a cephalic presentation, her cervix is soft and 1-cm dilated, and the fetus is at a -3 station. Her membranes are intact, she is not contracting, and her vitals are normal. She requests to be induced. For decreased risk of complication and optimal fetal outcome, when should she expect to be induced? A Now B At 40.5 weeks C 41.5 weeks D 42.5 weeks E Not expected

at 41.5 weeks In the absence of complication, the recommendation from ACOG is to wait for labor to occur. Large for gestational age is not an indication for induction in the absence of diabetes. Gestations greater than 42 weeks increase risk of fetal stillbirth.

A 32-year-old woman presents with a 2-day history of having a vaginal "bump"; the bump is painful to sit on. She has never had this problem before, and she has been monogamous with the same sex partner for 7 years. On physical exam (refer to image), you notice a solitary 2 cm smooth, slightly tender mass at the introitus. A KOH/wet mount demonstrates squamous cells with no white blood cells (WBCs), hyphae, or motile organisms. Whiff test is negative. Question What is the most likely diagnosis? Answer Choices 1 Vaginal candidiasis 2 Herpes simplex, type 2 3 Lichen sclerosis 4 Bartholin gland cyst 5 Vaginal intraepithelial neoplasia (VAIN)

bartholin gland cyst Explanation This patient most likely has a Bartholin gland cyst. The Bartholin glands are located at approximately the 4:00 and 8:00 positions at the introitus, and secretions help with vaginal lubrication. A cyst may form when the gland outlet is obstructed, and occasionally infection develops, leading to an abscess. Both Bartholin cysts and abscesses can be painful. Initial conservative treatment consists of warm compresses to encourage drainage. If this is unsuccessful, incision and drainage is necessary. If Neisseria gonorrhea is implicated as a causative organism, appropriate antibiotic treatment should be prescribed. Vaginal candidiasis (also known as a "yeast infection") would be characterized by symptoms of vaginal pruritus and discharge. Vulvar pain is common, and patients may note pain upon sitting. However, a physical exam would reveal vaginal and vulvar erythema, thick white discharge, and hyphae on the wet mount. The fungal hyphae are especially noticeable when potassium hydroxide (KOH) is applied to the microscope slide to lyse the other cells. Vaginal candidiasis is not associated with a solitary vulvar mass. Herpes simplex, type 2 is typically the strain associated with genital herpes. A symptomatic patient with herpes may describe vulvar lesions causing pain with sitting. When lesions are visible, herpes is characterized by multiple, painful vesicles and/or ulcerated lesions. Herpes lesions are typically much smaller than the described single mass. Lichen sclerosis is a chronic, progressive dermatological problem; it often involves the vulvar and perineal epithelium. It is characterized by pruritus, dyspareunia, and fissuring of the skin. Lichen sclerosis is more common in post-menopausal women, and physical exam reveals thin, white-appearing tissue. Masses are not associated with lichen sclerosis. Vaginal intraepithelial neoplasia (VAIN) is typically located at the upper 1/3 of the vagina. Its presence is usually detected by an abnormality on the Pap pathology from asymptomatic patients. This patient's 2 cm mass at the introitus could potentially be a malignant tumor, but VAIN is an unlikely diagnosis. Bartholin gland carcinoma is a possibility, but it is very rare.

You are caring for a 33-year-old G3P2 women. Her PMH is significant for obesity and allergies. She receives routine obstetrical care. When is she due for her glucose challenge testing to rule out gestational diabetes? A Between 12 and 16 weeks gestation B Between 16 and 20 weeks gestation C Between 20 and 24 weeks gestation D Between 24 and 28 weeks gestation E Between 28 and 32 weeks gestation

between 24 and 28 weeks gestation Routine surveillance for gestational diabetes of a pregnant women is between 24 and 28 weeks; it is not modified due to obesity or other risk factors for diabetes. Gestational diabetes is a hormone-mediated intolerance. Surveillance is modified in the presence of pre-existing diabetes.

A 32-year-old woman is 2-hours status post cesarean delivery of a twin gestation at 36 weeks. Her pregnancy was uncomplicated. She presented in early labor, which became prolonged despite oxytocin infusion. A cesarean section was performed when her labor became non-progressive. In the recovery area, she notes nausea and lightheadedness. On exam, her heart rate is 133 beats per minute, and blood pressure is 76/42 mm Hg. Significant vaginal bleeding is noted; abdominal palpation reveals a soft uterus. Question What intervention would be most appropriate? Answer Choices 1 Administration of intravenous magnesium 2 Bimanual uterine massage 3 Immediate surgical exploration 4 Transfusion with fresh frozen plasma 5 Transvaginal ultrasound

bimanual uterine massage Explanation The most likely cause of bleeding in this patient is uterine atony. Uterine atony occurs when the uterine myometrium fails to contract following delivery. Contractions of the uterine muscles after delivery normally tamponade bleeding from uterine arterioles. Absence of this response causes continued bleeding, which usually becomes evident early after delivery. Bimanual uterine palpation revealing a soft, 'boggy' uterus confirms the diagnosis. Initial treatment involves bimanual uterine massage, which helps promote uterine contractions. In addition, uterotonic agents are administered. The first-line of therapy is intravenous oxytocin. Second-line therapies including ergot alkaloid derivates and prostaglandins (e.g., Hemabate) are used when oxytocin therapy is unsuccessful. When these measures are unsuccessful, surgical interventions are required, the most common of which is bilateral uterine artery ligation. Uterine atony can also occur as a result of retained products of conception; these products inhibit uterine contraction. In this case, manual or surgical extraction is necessary. Intravenous magnesium is a uterine relaxant used to inhibit uterine contractions in settings of premature labor. It has the opposite effect than that desired to treat uterine atony. Transfusion with fresh frozen plasma may be indicated as a temporizing measure in the setting of disseminated intravascular coagulation accompanied by severe bleeding or massive uncontrolled hemorrhage from uterine rupture, neither of which is suspected in this clinical scenario. Transvaginal ultrasound is rarely needed to diagnose postpartum hemorrhage, and it is not the most appropriate next step in light of the patient's condition.

A previously healthy 26-year-old primiparous woman presents after 2 days of prolonged rupture of the membranes at 36 weeks of gestation with breech fetal presentation. Because of frequent fetal heart rate decelerations, the child is delivered by caesarean section. Prophylactic antibiotics were not given. Post-operatively, the woman starts complaining of increasing fatigue and muscle pain. On the 3rd day post op, she starts experiencing headache, insomnia, anxiety, generalized abdominal pain, and fever. Her temperature is 41 C. Pulse rate and respiration rate are increased (120/min and 20/min respectively). On examination, there are no signs of infection of the abdominal wall; however, there is a red rash, with swelling and tenderness, on both of her thighs. Question What is your initial step? Answer Choices 1 Antibiotics IV 2 Blood culture x 3 3 Painkillers as needed 4 Ultrasound examination 5 Antipyretics IV

blood cultures x 3 Explanation The patient most likely has puerperal sepsis. Puerperal sepsis is an infection of the genital tract that occurs at any time between the onset of the rupture of the membranes or labor and the 42nd day postpartum. Diagnostic essentials are also systemic signs of infection, abdominal and/or pelvic pain, vaginal discharge, peritoneal signs, abnormal smell/foul odor of discharge, and a delay in the rate of reduction of size of uterus (<2 cm/day during first 8 days). Collecting blood cultures prior to antibiotic administration is mandatory to identify the organism that caused sepsis. Most organisms that cause puerperal sepsis are considered normal vaginal flora, but they can cause puerperal infection in a patient with predisposing factors like: prolonged (more than 24 hours) or traumatic labor, Caesarean section, frequent and unsanitary vaginal examinations, unsanitary delivery practices, retained products of conception, hemorrhage or certain maternal conditions (anemia, malnutrition etc.), and prolonged and premature rupture of the membranes. Premature rupture of membranes happens when rupture of the membrane of the amniotic sac and chorion happens more than 1 hour before the onset of labor; it is prolonged when it occurs more than 18 hours before labor, and it is preterm when it occurs before 37 weeks gestation. This patient was at high risk for developing puerperal sepsis, and she was a good candidate for prophylactic antibiotic therapy (it was not given). You should definitively give antibiotics in the case of sepsis, but failure to check blood cultures prior to an antibiotic regimen can affect the growth of blood borne bacteria and prevent a culture from becoming positive later. You may give her painkillers, but it is essential that you deal with the cause of the sepsis. Ultrasound examination should be considered after you stabilize the patient - it is not your 1st step in the management. Antipyretics are also indicated in this patient, but blood culture is the 1st step in management of sepsis.

A 31-year-old woman presents with fever, malaise, and a mass in the right breast. On physical examination, a 4 cm (in diameter) cystic mass is present in the right breast under the nipple. The mass is tender to palpation. Her vital signs are pulse 95/min, BP 120/80 mm Hg, and temperature 99.9°. A fine needle aspirate is performed on the breast cystic lesion. A representative field of the aspirated material is represented in the image. What is the most likely diagnosis? Answer Choices 1 Breast abscess 2 Fat necrosis 3 Fibroadenoma 4 Ductal adenocarcinoma of the breast 5 Lobular adenocarcinoma of the breast

breast abscess Explanation The aspirated material is predominantly purulent with numerous polymorphonuclear leukocytes, debris, and few scattered benign epithelial cells. Abscess of the breast occurs commonly during lactation due to injury to the mammary ducts. Microscopically, the abscess cavity consists of numerous polymorphonuclear leukocytes and secretions surrounded by inflamed and (eventually) fibrotic breast tissue with obliteration of the lobular pattern. Clinically, a localized abscess may mimic carcinoma. The abscess is treated by incision and drainage with administration of antibiotics. Fat necrosis most often presents with a hard mass in the breast that can mimic carcinoma; it results from trauma to the breast or following a surgical procedure. A history of trauma can be elicited in 1/2 of the cases, usually 1 - 2 weeks prior to the occurrence of the fat necrosis. Microscopically, it is characterized by broken-down fat surrounded by clusters of foamy macrophages, with the presence of occasional hemosiderin-laden macrophages and dystrophic calcification. Fibroadenoma is a common benign breast lesion, usually occurring in patients between the ages of 20 and 35 years. It increases in size during pregnancy and tends to regress as the age of the patient increases. It is usually a single lesion, but in 20% of the cases there are multiple lesions in the same breast or bilaterally. Microscopically, the fibroadenoma varies from case to case depending on the relative amounts of glandular and stromal tissue. The fine needle aspirates of fibroadenoma show branching monolayers of epithelium, with well-defined outlines and stromal elements represented by amorphous background material and numerous naked oval stromal nuclei in the background. The epithelial cells are cytologically bland. Breast adenocarcinoma, whether ductal or lobular, is ruled out by the inflammatory background and the benign bland cytologic appearance of the few epithelial cells present in the smear.

You note an irregular hard mass with ill-defined margins on the right breast of a 37-year-old woman. In addition, the nipple has bloody discharge. What is the most likely diagnosis? Answer Choices 1 Mastitis 2 Breast cancer 3 Paget's disease 4 Fibroadenoma

breast cancer Explanation Breast cancer is manifested as a single, irregular, stellate, fixed and firm nodule as is presented in the scenario. A mastitis is warm and erythematous along the ducts, and it is usually associated with breast feeding. Paget's disease presents with a nodule at the epidermis of the nipple/areola, with an eczema rash, scaling, and excoriation. Fibroadenoma is a single, nontender, freely movable, round, and lobular nodule; it typically has clear margins.

A 56-year-old male patient is diagnosed with prostatitis. Which of the following is the least appropriate antibiotic to prescribe in the family practice setting? A ceftriaxone B doxycycline C levofloxacin D trimethoprim-sulfamethoxazole (TMP-SMX) E All are appropriate antibiotics for this patient in this setting.

ceftriaxone A The response to antibiotics in acute bacterial prostatis is usually prompt, perhaps because drugs penetrate readily into the acutely inflamed prostate Antibiotic selection should be guided by results of urine cultures and susceptibility results. Appropriate empiric antibiotics include a fluroquinolone (i.e.levofloxacin 500 mg once daily) or TMP/SMX (one double-strength tablet every 12 hours). Patients who are too ill for oral therapy or are septic on presentation should be hospitalized for initial parenteral treatment (intravenous quinolones with or without an aminoglycoside). Ceftriaxone would not be recommended as first-line.

A hydatidiform mole is removed from the uterus of a 20-year-old woman. Subsequent to evacuation, her serum human chorionic gonadotropin (hCG) concentrations are monitored, which continue to rise. What is most likely diagnosis? Answer Choices 1 Choriocarcinoma 2 Ectopic pregnancy 3 Ovarian failure 4 Pituitary insufficiency 5 Sarcoma of the uterus

choriocarcinoma Explanation Hydatidiform mole is a form of gestational trophoblastic disease. It is a benign proliferation of trophoblastic tissue with cystic (hydropic) swelling of chorionic villi. It commonly occurs in women younger than 20 or over 45. Once a hydatidiform mole has been diagnosed, it must be removed. This can be done by either evacuating the uterus (an option preferred in younger women) or by hysterectomy. Since most trophoblastic cells secrete hCG, its levels are indicative of tumor mass, a persistently high or increasing level of hCG after removal of the mole indicates development of an invasive mole or choriocarcinoma. Hydatidiform moles are essentially due to a failed pregnancy; a second, ectopic pregnancy is extremely unlikely. Since only trophoblastic tissue produces hCG, none of the other mentioned conditions would cause an increase in its level.

The placenta has 2 portions that are identified with either the fetus or the mother. Each portion has its origin from different cells or tissues. Which of the following is the fetal portion of the placenta? Answer Choices 1 Chorion 2 Amnion 3 Yolk sac 4 Allantois 5 Decidua

chorion Explanation The fetal portion of the placenta is derived from the chorion which contains the chorionic plate and chorionic villi. The maternal portion is formed by the decidua basalis. The yolk sac develops in the first trimester to aid in the transfer of nutrients. It also has a role in blood development, is the primitive gut, and the walls contain the primordial germ cells. Usually the yolk sac detaches by the 6th week, but can persist in the adult as Meckel's diverticulum. The decidua is the endometrium found in the pregnant uterus and is shed at partition. There are several forms of decidua that occur during pregnancy and a decidual cast that can occur independent of pregnancy. The amnion can be found surrounding the fetus and amniotic fluid. It is derived from trophoblast and can be seen in the first trimester on a sonogram. Allantois is found in most mammals and serves a different function in each species. In humans its blood vessels give rise to those found in the umbilical cord. In the term fetus the allantois becomes the median umbilical ligament.

A 30-year-old G1P1 Caucasian woman with a desire to become pregnant. She has been having unprotected intercourse for 12 months without pregnancy. She typically has 3 - 4 menstrual cycles a year. She reports that she had similar problems becoming pregnant with her 1st child. Her prior doctor did a complete workup for her infertility and amenorrhea. Records have been sent for your review. She successfully conceived on a medication 3 years ago, but she cannot recall the name of the medications. She has never used any method of contraception. This patient would like you to treat her infertility. Question What is the drug of choice for this patient? Answer Choices 1 Clomiphene 2 Danazol 3 Leuprolide 4 Mifepristone 5 Spironolactone

clomiphene Explanation The drug of choice for this patient is clomiphene (Clomid and others), which has estrogen agonistic and antagonistic effects, which increase gonadotropin release and follicular maturation. This medication also improves the luteinizing hormone release and estradiol secretion. Spironolactone is the medication that inhibits aldosterone and dihydrotestosterone, functioning as an androgen receptor blocker. Its primary use in gynecology is for acne, hirsutism, and polycystic ovarian syndrome. Leuprolide is a medication that causes inhibition of gonadotropin release, which reduces ovarian steroidogenesis. It is used in women for the treatment of uterine fibroids and endometriosis; it treats prostate cancer in men. It produces the opposite desired effect when given to a patient with infertility. Mifepristone is a medication with progesterone and glucocorticoid antagonistic effects, which reduce endometrial development. It is used for early pregnancy termination; off-label, it is used for endometriosis, uterine fibroids, and postcoital contraception. This medication should not be given to this patient. Danazol is an older medication that suppresses follicle stimulating hormone (FSH) and luteinizing hormone (LH). It reduces estrogen production and is weakly androgenic. This medication would worsen the patient's chances of conception.

A 34-year-old multiparous woman presents for a routine PAP smear after being "too busy" to have annual exams for the past 7 years. 3 Pap smears in her 20's have all been normal. She has had 1 episode of venereal warts in her late teens; there was no recurrence. She had 2 vaginal deliveries. She does not smoke. Remainder of her history is negative. Her Pap smear is reported as "atypical squamous cells of undetermined significance". What is the most appropriate next step in regards to evaluating the patient? Answer Choices 1 Colposcopy 2 Cone biopsy 3 Hysterectomy 4 Loop electrosurgical excision procedure (LEEP) 5 Repeat PAP in one year

colposcopy Explanation Under the Bethesda system of reporting Pap smear results, the term atypia is reserved for abnormalities which are neither clearly reactive in nature nor meet the criteria for squamous intraepithelial lesions. The evaluation then must obtain tissue for further specificity. Colposcopy with directed biopsies of any identified abnormality is the appropriate next step. In a patient with a negative history for HPV disease and annual Pap smears, and who can be counted on for compliance, a follow up PAP smear at a shorter interval (6 months) would be sufficient surveillance after a first atypical Pap smear. Cone biopsy and LEEP are not appropriate until a more specific tissue diagnosis is available. Hysterectomy is not indicated for an as yet undetermined cervical lesion.

A 23-year-old complains of chronic pelvic pain. It is worse with intercourse, several days before her period, and throughout her period. She is a GOPO, LMP two weeks ago. She is married and has had one partner for the last three years. Her symptoms have been increasing over the last year. What would the most appropriate initial management of her symptoms include? A Expectant management B Combined oral contraceptives C Surgical intervention D Acetaminophen E Androgen therapy

combined oral contraceptives B Endometriosis treatment is aimed at reducing pain and preserving fertility. Surgical interventional is not first line therapy until medication has been tried. NSAIDS are used, as opposed to acetaminophen, to lower the prostaglandin levels. Combined oral contraceptives suppress ovulation, decrease menstrual flow, and decidualize implants.

A 23-year-old woman comes to the office for a gynecologic examination. This is her first visit, and she has no complaints. She tells you that she has not had a Pap smear in several years. Her menarche was at 12 years, and she has had regular cycles since then. She has had several sexual partners in the past, but has been with her current partner in a monogamous relationship for 1 year. She reports that she had a chlamydial infection that was treated several years ago, but she denies a history of other sexually transmitted diseases. She has never been pregnant. On physical examination, her cervix appears friable with a slight area of ulceration. There are several perineal and vaginal lesions, which appear as small cauliflower-like projections. The results of the Pap smear, which return in 1 week, show a low-grade squamous intraepithelial lesion (mild dysplasia, CIN I). What factor in this patient's case is most closely correlated with the abnormal finding on the Pap test? Answer Choices 1 Condyloma acuminata 2 Condyloma lata 3 Early age at menarche 4 History of chlamydia 5 Nulliparity

condyloma acuminata A Pap smear should be obtained routinely during the pelvic examination at the initial visit. The Pap smear as a screening test has been extremely successful for the prevention of cervical cancer. Detection of dysplasia allows for successful treatment and prevention of cancer. Risk factors for cervical cancer include: Early age of first coitus Multiple sexual partners Immune suppression Low socioeconomic status Lack of Pap smear screening This patient has more than 1 risk factor at this young age. Condyloma acuminata are soft, fleshy warts that are caused by the HPV. Cervical dysplasia and carcinoma in situ are likely caused by types 16, 18, 31, 33, and 34. It is sometimes difficult to differentiate from squamous cell carcinoma. It is possible to treat these warts with a topical application if they are small in size. If they are large warts, cryotherapy may be required.

A 27-year-old woman presents in active labor. She is G3P2 and at 39 weeks of gestation. She has been receiving prenatal care since 6 weeks gestation, and her pregnancy has been uncomplicated. Both of her prior births were normal spontaneous vaginal deliveries. Her cervix is 6 cm, 90% effaced, mid-position, and soft. The fetus is not engaged and is thought to be vertex. Initial fetal monitoring shows a heart rate in the 140s with good accelerations, and it is reassuring. Contractions are 4 minutes apart, and she is comfortable. 20 minutes later, she experiences a large gush of clear fluid, and severe variable decelerations appear on the fetal heart rate monitor. Question What is the most likely diagnosis? Answer Choices 1 Cord prolapse 2 Placental abruption 3 Placenta previa 4 Uterine rupture 5 Vasa previa

cord prolapse Explanation The most likely diagnosis is cord prolapse. Cord prolapse occurs in 2 out of 1000 deliveries. It is diagnosed when fetal monitor recordings show that severe variable decelerations or bradycardia occur after membrane rupture. The cord is often palpable in the vagina. Cord prolapse happens most often at 5-cm cervical dilatation and in nonvertex presentations. 3 types of prolapse can occur. Overt cord prolapse is diagnosed when the membranes are ruptured and the umbilical cord falls through the cervix into the vagina ahead of the fetal presenting part. Funic presentation describes loops of umbilical cord between the presenting part and the cervical os prior to rupture of membranes. Occult cord prolapse is diagnosed when the cord is palpable alongside the presenting part on digital cervical exam. Treatment: Immediate delivery is essential to prevent fetal compromise. Cesarean delivery is generally preferred when the cervix is not fully dilated. Vacuum or forceps delivery may be attempted if the cervix is completely dilated, although manual elevation of the fetal part and emergent cesarean are the most common management. Uterine rupture is less likely since the patient has no history of cesarean delivery. Abruption is often associated with tetanic contractions and bleeding. An increase in uterine tone is seen between contractions. Vasa previa usually presents with painless bleeding at rupture of membranes. The cervical exam is not consistent with a diagnosis of placenta previa.

Upon inspection of an 18-year-old man's scrotum, you note that the left side is underdeveloped, and a testis is not palpable. There is no scrotal tenderness, swelling, or nodularity. What is the patient's condition most likely due to? Answer Choices 1 Cryptorchidism 2 Acute epididmitis 3 Hydrocele 4 Testicular cancer 5 Strangulated inguinal hernia

cryptorchidism An underdeveloped scrotum, either unilateral or bilateral, is likely due to undescended testicles, which is also known cryptorchidism. Acute epididymitis and strangulated inguinal hernia are usually tender to the touch and cause swelling. Hydroceles are commonly accompanied by scrotal swelling. If painless nodules are palpated in the testes, testicular cancer should be ruled out, especially in men between the ages of 15 to 35.

A 30-year-old G 2 P 1 woman comes to the emergency department at 37 weeks gestation with a chief complaint of worsening abdominal pain for the past two hours. On examination, the uterus is tense and tender. External fetal monitoring indicates frequent uterine contractions with late decelerations. An ultrasound suggests a retroplacental hemorrhage. What is the most appropriate course of action at this time? A Emergency operative delivery B Expectant management at home C Monitoring in the labor suite with the mother on her side D Tocolytic administration until the infant is 38 weeks gestation E Vaginal delivery with induction

emergency operative delivery A Indications for emergency cesarean delivery in a woman with placental abruption include fetal heart tracings that are not reassuring. Furthermore, a retroplacental hemorrhage carries a direr prognosis than a subchorionic one. Expectant management (B), monitoring (C), attempting to stop labor (D), and attempting a vaginal delivery (E) put both mother and fetus at severe risk of death.

A 49-year-old woman has had irregular menses for more than two years. Most of the time she requires only 1-2 mini-pads per day to handle the flow, but recently she soaked through a dozen maxi-pads in a day. On physical examination, she appears well and her pelvic exam in unremarkable. Her serum progesterone is low and her hematocrit is 39%. A transvaginal ultrasound reveals an endometrial stripe of 11 mm. Which of the following is the most appropriate next step in her management? A Abdominal hysterectomy B Endometrial biopsy C Fractional curettage D Hysteroscopy

endometrial biopsy This woman's excessive bleeding and endometrial stripe > 5 mm raises the suspicion for endometrial hyperplasia or cancer. The next step is an endometrial biopsy, which can be performed without anesthesia in the outpatient setting. Hysterectomy (A) is inappropriate without first establishing a diagnosis for the bleeding. Fractional curettage (C) is the definitive procedure for diagnosis of endometrial carcinoma, but requires anesthesia with its attendant risks. Hysteroscopy (D) is more invasive than endometrial biopsy and may spread tumor cells into the peritoneal cavity. Watchful waiting (E) is inappropriate given with size of the endometrial stripe.

A 25-year-old Caucasian woman and her husband have been trying to have a baby for the last 2 years. The patients' medical history included cyclical pelvic pain, dysmenorrhea and dyspareunia. The physical exam reveals the following: diffuse abdominal or pelvic pain of variable location, nodular thickening and tenderness along the uterosacral ligaments, on the posterior surface of the uterus, and in the posterior cul-de-sac, scarring and narrowing of the posterior vaginal fornix, and adnexal enlargement and tenderness. Question What is the most likely diagnosis? Answer Choices 1 Endometriosis 2 Ectopic pregnancy 3 Adnexal mass 4 Pelvic relaxation 5 Leiomyomatous uterus

endometriosis Explanation Endometriosis is the growth of tissue outside the uterus. The ectopic endometrial tissue can proliferate, and infiltrate, and spread to remote sites elsewhere in the body. Sites that endometriosis is most often found are, ovarian, pelvic peritoneum, anterior and posterior cul-de-sac, uterosacral, round, and broad ligaments, and fallopian tubes. An ectopic pregnancyis one that develops at any site other than the endometrium. Symptoms of an ectopic pregnancy are abdominal pain, possibly a missed period, abnormal bleeding at the time of presentation is not uncommon. Physical characteristics of an adnexal mass that may give a presumptive diagnosis: consistency (cystic, solid or both), size (in centimeters) morphology and surface contour (smooth, nodular, vague or sharp borders), location around the uterus, mobility (free or fixed to adjacent structures) tenderness, unilateral or bilateral. The structures supporting the urethra (urethrocele), bladder (cystocele), uterus, posterior wall of the vagina (enterocele), and rectum (rectocele) weaken. Often symptoms fo this pelvic relaxation occur at or after menopause because of the lack of hormonal effect, causing atrophy of these tissues. Each of these structures has different presenting symptoms. A leiomyoma is a benign uterine tumor. Other names for leiomyoma include myoma, fibroid, and fibromyoma. Symptoms associated with leiomyoma are bleeding, pressure, and pain.

A 27-year-old G1 P1 woman has recently given birth by caesarean section to a 36-week male infant. She did well throughout her pregnancy up until 34 weeks gestation. At that time, she presented with fever, abdominal pain, and wetness. She was diagnosed with preterm premature rupture of membranes (PPROM) and chorioamnionitis. She was treated with steroids and antibiotics, stabilized, and she then delivered by C-section with no complications. On postpartum day 7, she presents with sore breasts from breast-feeding and a sore abdomen. She also admits to an odorous vaginal discharge, but she denies any associated vaginal bleeding. On physical examination, she is having a moderate amount of lochia alba, and her temperature is 101.2°F. Question What is the most likely diagnosis at this time? Answer Choices 1 Deep vein thrombosis (DVT) 2 Endometritis 3 Infected retained placenta 4 Mastitis 5 Septic pelvic thrombophlebitis

endometritis Explanation Postpartum endometritis is most likely in this patient given her history of chorioamnionitis. Prolonged rupture of membranes, cesarean delivery, prolonged labor, and multiple cervical examinations are all risk factors for postpartum endometritis. The presence of intra-amniotic infection increases the risk of postpartum endometritis further. Antibiotics are not routinely continued for chorioamnionitis after a delivery because the "source" of the infection (the placenta) has been removed. Whenever fever occurs in the immediate postpartum period, endometritis should be suspected. Broad-spectrum antibiotics should be administered promptly by the parenteral route. Similar to chorioamnionitis, multiple bacterial organisms (usually normal vaginal and perineal flora) are likely to be responsible for this infection. Therefore, uterine cultures are unlikely to be helpful for guiding antibiotic therapy. Parenteral therapy should be continued until the patient has been afebrile for at least 24 hours. Mastitis is characterized by a swollen, firm, tender breast with systemic symptoms including inflamed breast, fevers, chills, and flu-like symptoms. Staphylococcus aureus is the typical pathogen. However, in the immediate postpartum period, breast engorgement without infection is the most likely reason for the patient's sore breasts. Pregnancy and the postpartum period increase a woman's risk of thrombogenesis. However, DVT is not a likely source of the fever. Septic pelvic thrombophlebitis is a diagnosis of exclusion and is usually entertained when fever spikes continue following treatment for endometritis. Infected retained placenta is unlikely in the absence of vaginal bleeding. Lochia This is the uterine discharge following delivery and lasts for 3 or 4 weeks. Foul-smelling lochia suggests infection. Types: (1) Lochia rubra. This blood-stained fluid lasts for the first few days. (2) Lochia serosa. This discharge appears 3 to 4 days after delivery. It is paler than lochia rubra because it is admixed with serum. (3) Lochia alba. After the 10th day, because of an admixture with leukocytes, the lochia assumes a white or yellow-white color. Puerperium: This period of 4 to 6 weeks starts immediately after delivery and ends when the reproductive tract has returned to its non-pregnant condition. Multiple anatomic and physiologic changes occur during this time; the potential exists for significant complications, such as infection or hemorrhage. Puerperal infection is defined as any infection of the genitourinary tract during the Puerperium accompanied by a temperature of 100.4oF (38 oC) or higher that occurs for at least 2 of the first 10 days postpartum, exclusive of the first 24 hours. Prolonged rupture of the membranes accompanied by multiple vaginal examinations during labor is a major predisposing cause of puerperal infection.

A 23-year-old Hispanic female delivers a healthy male child by normal vaginal delivery. She had a small episiotomy which was clean cut. She is discharged home after 48 hours but returns to the ER on the fifth post partum day with fever and chills. She says she has had foul-smelling vaginal discharge and lower abdominal pain for the last 24 hours. On exam, she has a temperature of 102° F, BP 110/60 mm of Hg, pulse 118/min, and SPO2 92%. Lungs are clear, and she has no pallor, cyanosis, or icterus. Abdominal exam demonstrates tenderness in the suprapubic area and both lower quadrants. Pelvic exam shows foul smelling purulent vaginal discharge and tender uterus. The episiotomy site is non-tender. Labs show Hb 0f 11g/dl, WBC 13,500, bands 11%, and platelets 350,000/uL. Urinalysis is pending. Question The most likely diagnosis is: Answer Choices 1 Endometritis 2 Pyelonephritis 3 Wound cellulitis 4 Cervicitis 5 Cystitis

endometritis Explanation This female patient is suffering from post partum endometritis as obvious by recent delivery, fever, foul-smelling vaginal discharge, and tender pelvic exam. Other risk factors for endometritis include invasive gynecological procedures, retained products of conception, intrauterine devices, submucosal fibroids, multiple sexual partners, unprotected intercourse with infected partner who may be asymptomatic, etc. Leukocytosis with bandemia supports diagnosis. Blood cultures, urine culture, and cervical culture may be done but treatment should not be delayed. Broad spectrum antibiotics to cover beta lactamase and anaerobic organisms like clindamycin with an aminoglycoside are given. Pyelonephritis is unlikely since she has no back pain or costovertebral angle tenderness, and foul vaginal discharge is absent in pyelonephritis. UA and urine culture should rule it out. Wound infections are rare in a small, clean-cut episiotomy. Fever with foul vaginal discharge does not occur, and the wound area will be obviously erythematous, indurated, and tender. Even in third or fourth degree lacerations antibiotic therapy is usually unnecessary. Cervicitis is either asymptomatic or has purulent vaginal discharge without fever, chills, tender abdominal exam, or leukocytosis. Cystitis may be associated with low grade fever and suprapubic tenderness, but dysuria is predominant and vaginal discharge absent. UA confirms diagnosis.

A 49-year-old woman presents with intermittent vaginal bleeding and anemia for 3 months. After her endometrial biopsy, you confirm that she has endometrial carcinoma. What is the largest risk factor for endometrial cancer? Answer Choices 1 Estrogen exposure 2 Oral contraceptive pill 3 Polycystic ovary 4 Granulosa-theca cell ovarian tumor 5 Obesity

estrogen exposure Increased risk of endometrial cancer has been associated with the following: Estrogen exposure is the largest risk factor: Sources of estrogen exposure may be endogenous or exogenous. There is a significant correlation between use of oral estrogen and endometrial cancer, when estrogen therapy is administered without the protective effects of cyclic progestin. Obesity: Women who are 20 - 50 lbs. over their ideal body weight have a 3-fold increased risk for endometrial cancer. Polycystic ovary disease: This is the disease risk attributed to stimulation release of unopposed estrogen from the ovaries. Granulosa-theca cell ovarian tumors: A hormonally active estrogen-producing stromal tumor of the ovary.

A 26-year-old woman presents with a 3-month history of amenorrhea. She has been sexually active with her husband, and she has been trying to conceive for the past 1 year. A pregnancy test is obtained, and it is negative. On examination, she appears overweight; there is acne and hirsutism. Her past menstrual history is significant for irregular menstrual cycles; they are not associated with dysmenorrhea or menorrhagia. Question What is the best management for her infertility? Answer Choices 1 Weight loss and exercise 2 Androgen receptor antagonist 3 Metformin 4 Estrogen receptor modifiers 5 InVitro fertilization

estrogen receptor modifiers Explanation Estrogen receptor modifiers is the correct answer. Clomiphene citrate is a commonly used drug in the management of anovulatory infertility. The vignette describes a classic case of polycystic ovarian syndrome, which is the most common cause of anovulatory infertility. It acts centrally by inhibiting the negative feedback of estrogen on the release of gonadotropins. This results in higher levels of follicle stimulating hormone (FSH) and luteinizing hormone (LH), which promotes the development and release of an ovum. Weight loss and exercise is incorrect. They are key to the management of PCOS, and they do so by reducing Insulin resistance and improving glycemic control. However, they do not correct anovulation. Metformin is incorrect. Metformin reduces insulin resistance and improves peripheral glucose uptake while promoting weight loss. However, it does not correct infertility. In vitro fertilization is incorrect. It is a viable option, and it is often used in infertility after medical management has failed. It is expensive, invasive, and time consuming; there is no guarantee of success. Therapy with clomiphene results in fertility in a number of women with PCOS and should be the first line of therapy.

For the past year, a 30-year-old woman and her husband have been trying unsuccessfully to become pregnant. Over-the-counter ovulation tests have indicated that she is ovulating. Neither partner smokes cigarettes, uses any mind-altering drugs, or has a history of sexually transmitted infection. She has no history of abdominal surgery or pelvic procedures, diethylstilbestrol (DES) exposure, or major illness. On examination, she is 66" tall, weighs 135#, and appears healthy. Her thyroid is nonpalpable, and pelvic examination is unremarkable. What is the most appropriate next step in evaluation of this couple's infertility? A Endometrial biopsy B Examination of the husband C Pelvic ultrasonography D Performing a hysterosalpingogram E Referral to an infertility clinic

examination of the husband Male factors account for 25-40% of infertility. Since the woman's initial evaluation appears normal and ovulation tests are positive, the next step is to evaluate the husband before embarking on a more detailed evaluation of the wife. An endometrial biopsy (A) may be appropriate during the late luteal phase to evaluate the endometrial lining. Pelvic ultrasonography (C) may help identify ovarian cysts, endometrial implants, leiomyomas and other treatable conditions. Hysterosalpingography (D) is useful in evaluating the patency of the fallopian tubes and the size and shape of the uterine cavity. Since this woman is only 30 years of age and many other studies can be performed in ob-gyn setting, referral to an infertility clinic (E) is premature.

A 65-year-old woman is concerned about a non-tender lump she discovered in her right breast last month. When performing a breast self-exam this month, she noticed the lump persists, but it is unchanged. On examination, you feel a firm, unfixed, non-tender 2 cm mass in the right upper outer quadrant; there are no palpable axillary nodes. Mammography and a fine-needle aspirate (FNA) are inconclusive. What is the next most appropriate step? Answer Choices 1 Repeat mammography in 3 months 2 Ultrasound 3 Partial mastectomy 4 Tamoxifen therapy 5 Excisional biopsy

excisional biopsy Explanation Establishing a diagnosis under these circumstances requires excisional biopsy and pathological examination of the lesion. In this case, the mammogram and FNA do not explain the clinical findings despite the usual sensitivity of the FNA (>90%). Ultrasound studies may add some information but are typically not useful in establishing a definitive diagnosis. Initiating surgical or medical therapy is inappropriate as would be delaying a diagnosis for 3 months.

A 19-year-old G1P0 presents for her routine obstetric exam. She is at 34 weeks gestation. When completing the physical assessment you perform a Leopold maneuver. What does this maneuver assess? A Fetal lie and station B Fetal position and presentation C Fetal lie and position D Fetal lie and presentation E Fetal position and station

fetal lie and position C The maneuver is used to examine the abdomen and determine the lie (first) and position (second). While the abdominal exam can be used to determine breech versus cephalic, it does not differentiate between, face, brow, and footling presentations. A vaginal exam must be done to determine the presenting part and decent into the pelvis.

A 23-year-old primigravida is admitted to your ward because of vaginal bleeding and cramping at 26 weeks. The ultrasound shows fetal heartbeat, and you try to stop a miscarriage by ordering terbutaline. What do you tell the nurse to watch for? Answer Choices 1 Constipation and dry mouth 2 Headache, tachycardia, and nausea 3 Sedation, urinary retention, and dry mouth 4 Anxiety, headache, and mydriasis 5 Cold extremities, arrhythmia, and insomnia

headache, tachycardia and nausea Explanation Terbutaline relaxes smooth muscle in the bronchial system and the uterus by stimulating β2-receptors. It is a tocolytic drug, and most commonly used in patients with bronchospasms caused by reversible obstructive airway disease. Headache, tachycardia, and nausea are common side effects of beta-mimetic drugs, which relax the uterine muscle. Other side effects are shown in the following table. Central nervous system Drowsiness, dizziness, headache, tremor, nervousness Gastrointestinal Nausea, vomiting Cardiovascular Palpitations, tachycardia, arrhythmia Respiratory Paradox bronchospasm Metabolic Hypokalemia Skin Diaphoresis Other Dry mouth and throat Constipation and dry mouth are side effects of skeletal muscle relaxants, like cyclobenzaprine, which is used as short-term treatment of muscle spasms. Sedation, urinary retention, and dry mouth are side effects of antihistamine drugs, which effect the peripheral H1-receptors. Urinary retention and sedation however are side effects of the "older" drugs like brompheniramine, chlorpheniramine, clemastine, promethazine, and triprolidine and not the "newer" ones like fexofenadine, loratadine, and cetirizine. Anxiety, headache, and mydriasis are signs of atropine overdosage. Atropin is an anticholinergic that inhibits acetylcholine at the parasympathica neuroeffector junction. It blocks vagal effects on the sinuatrial and atrioventricular nodes thereby enhancing conduction through the AV node and increasing the heart rate. It is used to treat bradycardia and to dilate the pupils. Cold extremities, bronchospasm, and insomnia are side effects of beta-blockers. Beta-blockers (or symppatholytics) decrease myocardial contractility, heart rate, blood pressure, and cardiac output (less blood getting to the periphery, which explains cold extremities). Myocardial oxygen use gets reduced. Since it decreases the effect of catecholamines on the bronchial system, it can trigger an asthma attack by increasing contraction of bronchioli and alveoli.

A mother brings in her 20-month-old female child to the office because she noticed pubic hair growing. On examination, the clinician notices that the clitoris is enlarged; the rest is unremarkable. Which of the following is an expected laboratory finding on this patient? A increased aldosterone B increased estrogen C increased androstenedione D increased luteinizing hormone

increased androstenedione C Infant girls presenting with signs of precocious puberty need to be screened for congenital adrenal hyperplasia (CAH). CAH most commonly presents with pseudohermaphroditism in females—urogenital sinus, enlarged clitoris, or other signs of virilization. In males, there tends to be isosexual precocity in older males and salt-losing crisis in infant males. Both children show increased linear growth and skeletal maturation. The most common type of CAH is a deficiency in the enzyme 21-hydroxylase and laboratory tests demonstrate increased urinary and plasma androgens (DHEA, androstenedione). There may be elevated progesterone, but typically there is no effect on estrogen. There is also decreased aldosterone and elevated urinary ketosteroids. There is also no effect on the levels of leuteinizing hormone or follicle-stimulating hormone. Treatment usually involves glucocorticoids, mineralocorticoids, and reconstructive surgery, if needed.

A 19 year old female presents for her first pelvic examination and is noted to have a palpable, non-tender right ovary. She is sexually active, and uses condoms for birth control and protection from STDs. Her LMP was 20 days ago. She underwent transvaginal ultrasound and was found to have a 3-cm follicular cyst. What does this right ovarian mass represent? A Retained hemorrhagic products B Intrafollicular fluids C Solid intracellular material D Germ cell layers E Keratinized squamous epithelium

intrafollicular fluids B Follicular cysts occur prior to ovulation and cause expansion of the follicular antrum, serious fluid collection, and subsequent follicular cyst formation. Corpus luteum cysts form after ovulation and may accumulate blood products. Germ cell layers and keratinized squamous epithelium are solid components of teratomas.

A 12-year-old boy presents with a 3-hour history of extreme, severe pain in the right testis. It started suddenly, is 8/10 in intensity, and does not radiate. It is associated with nausea and scrotal swelling. He never had such pain in his lifetime, and he denies any problem in urination. He has never been operated on, and he denies any history of trauma. He is allergic to penicillin. On physical exam, the child is in visible distress. Temperature is 37.0°C, heart rate is 95, blood pressure is 120/70 mm Hg, and respiratory rate is 20 per minute. Genital examination reveals enlargement and edema of the entire scrotum. The right testicle is erythematous and tender to palpation; it appears to sit higher and lies horizontally in the scrotal sac relative to the left side. The cremasteric reflex is absent ipsilaterally, and there is no relief of pain upon elevation of the scrotum (Prehn's sign). Abdomen is non-tender and tympanic to percussion in all 4 quadrants. Bowel sounds are audible. Chest auscultation shows normal vesicular breathing with mild crepitations over the lower lung fields. Cardiac exam reveals normal S1 and S2, without rubs, murmurs, or gallop. His initial labs show a hemoglobin of 14.5 g/dL, WBC of 13,000/mm³, platelets of 210,000/mm3, sodium of 140 mmol/dL, potassium of 3.8 mmol/dL, chloride of 95 mmol/dL, urea of 25 mg/dL, and creatinine of 0.9 mg/dL. Question What sign or symptom is the most sensitive for the diagnosis of this condition? Answer Choices 1 Tenderness 2 Edema 3 Horizontal lie 4 Prehn sign 5 Loss of cremasteric reflex

loss of cremasteric reflex Testicular torsion is a true urologic emergency and needs to be differentiated from other causes of testicular pain (e.g., trauma, epididymitis/orchitis, incarcerated hernia, varicocele, idiopathic scrotal edema, and torsion of the appendix testis). The finding of an ipsilateral absent cremasteric reflex is the most accurate and sensitive sign of testicular torsion. This reflex is elicited by stroking or pinching the medial thigh, causing contraction of the cremaster muscle which elevates the testis. The cremasteric reflex is considered positive if the testicle moves at least 0.5 cm.

The pregnant mother of a 2-year-old patient is concerned about exposure to environmental neurotoxins affecting her developing fetus. She asks you questions about whether the placenta would block any transfer of toxins to a growing fetus. Which environmental neurotoxin transfer to the fetal system is actually enhanced by the placenta? Answer Choices 1 Mercury 2 Cadmium 3 Polychlorinated biphenyls 4 Insecticides 5 Environmental tobacco smoke

mercury Explanation Methyl mercury is absorbed almost completely and crosses the placenta easily and then becomes stored in the fetus. Fetal blood concentrations exceed maternal concentrations by 50-100%. High doses can result in mental retardation, spastic paralysis, and death. Low doses are linked to deficits in neuromotor performance, cognition, memory, and language. The placenta effectively blocks the transfer of cadmium. Studies have shown cadmium concentrations in umbilical cord blood to be significantly lower than that in maternal blood and placental cadmium concentration to be highest. Polychlorinated biphenyls (PCP) have contaminated sport fish, particularly bottom-feeding species from water contaminated with PCPs. Also during the 1940s and 1950s the inside of concrete silos on many farms in the Midwest were coated with sealants containing PCPs that over time have peeled off and become mixed with silage to feed beef and dairy cattle. Incinerators and other PCP-disposal facilities or hazardous waste sites are other sources of PCP exposure. PCPs are synthetic hydrocarbons that are lipophilic, have a long half-life, and can cross the placenta easily. Fetuses and neonates are more sensitive to PCPs because the hepatic microsomal enzyme system that facilitates metabolism and excretion are not fully functional. Insecticides or pesticides appear to cross the placenta relatively easily being lipophilic and having a low molecular weight. Residue levels of DDT and its metabolites were detected in maternal blood, placenta, and umbilical cord blood of mother/child pair studies. A correlation exists between pesticide concentration and age, dietary habits, and area of residence of pregnant women. Environmental tobacco smoke (ETS) exposure during fetal development is one of the most ubiquitous and hazardous of environmental exposures. Placental vascular resistance is often increased when women smoke during pregnancy and there are subsequent alterations of protein metabolism and enzyme activity in cord blood. Maternal uterine blood flow is lowered with reduced flow of oxygen from the uterus to the placenta. There are increased levels of carboxyhemoglobin in both maternal and fetal blood when the mother smokes that can lead to fetal hypoxia with consequent chronic fetal hypoxic stress.

A 13-year-old girl presents with concerns about excessive vaginal discharge without itching or burning for the past 3 days. She denies ever being sexually active. Her last menstrual period was 10 days ago. On examination you find a thin, white, homogeneous discharge that has a distinct amine odor when potassium hydroxide is added. On saline wet mount, epithelial cells are covered with bacteria. What is the most appropriate management of this patient? Answer Choices 1 Metronidazole 2 Tricholoroacetic acid 3 Clotrimazole 4 Fluconazole 5 Nystatin

metronidazole Explanation This patient has the classic signs of bacterial vaginosis (BV), including the thin discharge with the fishy odor on KOH prep, and clue cells on wet prep. Standard treatment of BV is metronidazole 500 mg po bid for 7 days. Tricholoroacetic acid is used for the treatment of genital warts. Clotrimazole, fluconazole, and nystatin are all used in the treatment of candidiasis.

A sexually active 19-year-old woman presents with clusters of painful vesicles on an erythematous base on the vulva and cervix, accompanied by temperature of 100°F and mild malaise. She reports a history of a similar outbreak last month, which resolved in 10 days. Microscopic examination of cells from the basement of a blister treated with Giemsa stain is likely to reveal A multinucleated giant cells B gram-positive cocci in clusters C gram-positive cocci in chains D gram-negative rods E hyphae and buds

multinucleate giant cells A The clinical presentation is consistent with herpes simplex. The appropriate microscopic study is a Tzanck smear, prepared by staining cells from the floor of a vesicle using Papanicolau, Giemsa, or Wright methods. The Tzanck smear will show multinucleated giant cells. It has a sensitivity of 60% to 70% and as a result should be confirmed by viral culture. Gram-positive cocci are consistent with staphylococcal or streptococcal infection and gram-negative rods are usually enteric pathogens. Hyphae and buds are seen on KOH prep with candidal infection.

A 45-year-old woman status-post total abdominal hysterectomy and bilateral salpingo-oophorectomy secondary to Stage I ovarian cancer. She has completed a course of cisplatin and cyclophosphamide, and she presents for follow-up. Serial CA-125 has returned to normal. What is the most common site of metastasis for this tumor? Answer Choices 1 Omentum 2 Lung 3 Liver 4 Bone 5 Brain

omentum Explanation The most common route for spread of ovarian carcinomas is transcelomic, along with normal flow of peritoneal fluid, in which case common sites for metastasis include the pericolic gutters, the omentum, small and large bowel. Only rarely do ovarian tumors spread via hematogenous dissemination to the lung, liver or bone, making follow-up imaging of these areas unnecessary. The brain is not considered a site for ovarian metastasis.

Serosanguineous ascites is generally associated with which of the following processes? Answer Choices 1 Ovarian cancer 2 Lymphoma 3 Congestive heart failure 4 Cirrhosis 5 Infection

ovarian cancer Explanation The correct answer is ovarian cancer. Bloody or serosanguineous ascites is associated with intraperitoneal cancers, of which ovarian cancer is one. Lymphoma is incorrect because it will generally produce a chylous ascites. Congestive heart failure is incorrect because it will generally produce serous ascites. Cirrhosis is incorrect because it will generally produce serous ascites. Infection is incorrect because it will generally produce cloudy ascites.

A 21-year-old female presents to clinic complaining of mild, low abdominal ache and intermittent dysuria. She denies N/V/D, and she is sexually active and uses condoms some of the time. Her LMP was 10 days ago, and she is a G0P0. Physical exam reveals a healthy female in no acute distress. Vitals are as follows: P 70, BP 120/80, T 99.9°F. Lungs are clear, CV RRR, abd soft non-tender, + BS. Pelvic exam reveals normal external genitalia, scant discharge, moderate cervical motion tenderness, and no adnexal masses. What is her most likely diagnosis? A Tubo-ovarian abscess B Gastroenteritis C Ectopic pregnancy D Cervicitis E Pelvic inflammatory disease

pelvic inflammatory disease Suspicion for PID should be very high in a young, healthy, and sexually active woman with cervical motion tenderness. She is not spotting and just menstruated, making ectopic pregnancy much less likely.

A 30-year-old patient presents to labor and delivery complaining of bright red vaginal bleeding. She has no pain. The fetus is still active. She is 37 weeks pregnant. PMH is significant for in vitro fertilization. What is the most likely diagnosis? A Placental abruption B Placenta acreata C Placenta previa D Disseminated intravascular coagulopathy E Active labor

placenta previa C The most likely diagnosis is placenta previa, as the bleeding is bright red and painless. Labor and abruption are associated with discomfort and pain.

A 29-year-old woman, G4P2011, LMP 9 months prior, presents with severe lower abdominal pain. The pain is sharp and tearing. She began to have vaginal spotting prior to presentation. She denies any contractions. There is no history of medical problems or surgery, and she is on no medications. All previous deliveries were vaginal. She has smoked 1 pack of cigarettes a day over the past 10 years, denies alcohol use, but does admit to a remote history of heroin abuse by insufflation. On physical examination: T = 100.4° F; BP= 110/70 mmHg; P= 85/min; RR= 20/min. Pertinent findings on the PE were relegated to the pelvic exam; fundal height measures 39 cm, and there is profuse bleeding from the vagina. Fetal monitor shows contractions every minute with elevated baseline uterine tone. Fetal tachycardia is evident at 180 beats/minute, and late decelerations are also present. Question What is the most likely diagnosis? Answer Choices 1 Cervical cancer 2 Chorioamnionitis 3 Placenta previa 4 Placental abruption 5 Uterine rupture

placental abruption Explanation This patient has placental abruption. Abruption is a leading cause of 2nd and 3rd trimester bleeding. A meta-analysis has demonstrated that smoking increases the risk of abruption by 90%. Maternal and paternal smoking increases the risk of abruption 2-fold. Risk increases 5-fold when both parents smoke. Women with a history of abruption have a 15% increased risk during future pregnancies. Placenta previa, cocaine use, preeclampsia, and preterm premature rupture of membranes (PPROM) are also associated with placental abruption. Placental abruption is defined as complete or partial separation of the placenta prior to delivery. The incidence of abruption is 5 or 6 out of 1000 deliveries. Obvious vaginal bleeding occurs if the hemorrhage develops between the membranes and the uterus. Concealed presentations occur when blood collects behind the placenta. Abruption is a leading cause of 2nd and 3rd trimester bleeding, and it causes significant maternal and neonatal morbidity and mortality. Classically placental abruption presents as painful third-trimester bleeding. Treatment:Emergent cesarean is generally indicated when the patient is not in labor and hemodynamic compromise is present. If delivery is imminent and the abruption is mild, vaginal delivery may be attempted. Cervical cancer is incorrect.Although cervical cancer can coexist with pregnancy, it is very unlikely in this case. The severity of the signs and symptoms in this case would be incongruent with achieving 9 months gestation. Chorioamnionitis is incorrect.Clinical features include maternal fever and uterine tenderness in the presence of confirmed premature rupture of membrane (PROM). Profuse vaginal painful bleeding is not a finding. Placenta previa is incorrect, as it classically presents with painless 3rd trimester bleeding. Uterine rupture is incorrect. A uterine rupture typically occurs during active labor. As this patient denies any contractions, it is very unlikely.

You are providing care for a 28-year-old who presents to clinic for her 27-week obstetrical check. She has no complaints and the fetus is active. Physical exam reveals the following: P 88, BP 142/90, FHT 148, UA negative. CBC is normal. Her BP pre-partum was 110/70. What is the most likely diagnosis at this point? A Pregnancy induced hypertension B Preeclampsia C Eclampsia D Chronic hypertension E stress

pregnancy induced htn HTN is one of the triad of symptoms of preeclampsia, but at this point there are no other symptoms or proteinuria. She will need to be carefully monitored, as 50% of these presentations will progress to preeclampsia.

A 20-year-old woman is brought to the emergency room (ER) by the police. She was assaulted and raped 3 hours ago. Her last menstrual period (LMP) was 22 days ago, and her period lasts for 4 - 5 days with a 28-day menstrual cycle. She is not using any contraceptive method. What hormone is mainly produced by the ovary at this point of her menstrual cycle? Answer Choices 1 Estrogen 2 Follicle stimulating hormone (FSH) 3 Inhibins 4 Luteinizing hormone (LH) 5 Progesterone

progesterone Explanation The correct response is progesterone. The 5th day of the menstrual period corresponds to the follicular phase of the cycle, which starts from the first day of bleeding to the day of the LH peak. In this phase, the FSH, which is produced by the pituitary, stimulates the development of follicles in the ovaries, with only 1 follicle dominant at the end. During the second half of this phase, the granulosa cells of the follicle begin to produce estrogen. Ovulation occurs approximately at the middle of the menstrual cycle (day 14). At the end of the follicular phase, a rise in the level of estrogens takes place, preceding the LH peak. The ovulation will succeed by an average of 30 hours after this peak. The 22nd day of the menstrual period corresponds to the luteal phase of the menstrual cycle. The luteal phase starts from the day of ovulation to the first day of the menstrual period. In this phase, the ovarian corpus luteum begins to produce progesterone. If there is fecundation fertilization of the ovum, the production of progesterone continues for 4 to 5 weeks, until the placenta can take over and produce enough of the hormone to maintain pregnancy. If there is no fertilization, progesterone levels decrease and the menstruation occurs. Estrogens are produced by the granulosa cells of the follicle. They are very important in the follicular phase; their level is very low at the beginning of this phase and reaches a maximum point at the middle of the cycle, just before the LH peak. They are the feedback for the LH production and (in part) for the FSH production. Inhibins are also hormones produced by granulosa cells of the follicle, and they are part of the feedback for the FSH. Inhibin B levels rise during the luteal-follicular transition, are highest during the mid follicular phase, then go up again during the LH peak, and finally decrease in the late follicular phase. Inhibins A levels decrease during the late luteal phase.

A 31-year-old woman is being evaluated for irregular, infrequent menstrual periods. On further questioning, she complains of headaches, fatigue, and breast discharge. She takes ibuprofen only occasionally. Which of the following labs would most likely be elevated in this patient? A BUN and creatinine B luteinizing hormone (LH) and follicle-stimulating hormone (FSH) C oxytocin D prolactin E TSH

prolactin This patient's symptoms are consistent with a pituitary adenoma. Prolactinomas account for about half of all functioning pituitary tumors and may secrete PRL, GH, and ACTH.

A 21-year-old woman presents for a Pap smear. On speculum exam, the cervix is in a normal position. During a bimanual examination of the pelvis, the body of the uterus is unable to be palpated by the abdominal or pelvic hand. When a rectovaginal exam is performed, the body of the uterus is palpable. What position is this patient's uterus in? Answer Choices 1 An anteverted uterus 2 A retroflexed uterus 3 A retroverted uterus 4 A second-degree prolapsed uterus 5 A third-degree prolapsed uterus

retroflexed uterus Explanation When the uterus that cannot be palpated by a bimanual exam but can be felt rectovaginally, it is tilted backwards. A retrodisplaced uterus wherein the cervix is normally positioned, as in this case, is in a retroflexed position. A retrodisplaced uterus with a forward-facing cervix is in a retroverted position. An anteverted uterus is a forward-facing uterus, the most common position found. In a 2nd-degree prolapsed uterus, the cervix is in the vaginal introitus with the body of the uterus descending from its normal placement in the pelvic. In a 3rd-degree prolapsed uterus, the cervix and uterus are outside the vaginal introitus.

A 28-year-old woman presents with abdominal pain and vaginal bleeding. The pain began last night as a dull ache in the right lower quadrant, but this morning it became much more severe. She also complains of dizziness and nausea. She cannot recall when her last menstrual period was, but she says that she began bleeding yesterday and has a light menstrual flow. On examination, the patient is afebrile; pulse is 100/min, BP is 86/60 mm Hg, and RR is 20/min. Physical exam reveals moderate to severe tenderness in the right lower quadrant with rebound. Pelvic exam reveals a small amount of blood at the cervical os with cervical motion tenderness. You order a variety of laboratory tests on this patient. Question What is the most likely diagnosis? Answer Choices 1 Acute appendicitis 2 Acute salpingitis 3 Tubo-ovarian abscess 4 Ruptured ectopic pregnancy 5 Ruptured ovarian cyst

ruptured ectopic pregnancy Explanation Ruptured ectopic pregnancy is probably the most life-threatening condition in gynecology. Any practitioner that cares for reproductive-aged women must keep a high index of suspicion for this dangerous event in order to avoid a delay in diagnosis that may be fatal. Ectopic pregnancy is defined as extra-uterine implantation of a pregnancy; it occurs most commonly in the fallopian tube (96%). Most tubal pregnancies are found in the distal 2/3 of the tube. 2% of ectopic pregnancies are uterine ectopic pregnancies (interstitial); other sites include the cervix, ovary, or pelvic and abdominal cavities, although these implantations are rare. The major risk factors for ectopic pregnancy are a prior tubal infection or tubal surgery, a prior ectopic pregnancy (giving a risk of 10-25+% for recurrence), a history of diethylstilbestrol (DES) exposure, or presence of an IUD (intrauterine device). Ectopic pregnancy occurs in 0.5 - 1.0% of pregnancies; however, the incidence of ectopic pregnancy is increasing in recent years. This increase is probably a reflection of the increasing incidence of pelvic inflammatory disease, increased use of IUDs, and increasing incidence of tubal ligation for sterilization and other tubal surgeries. A tubal pregnancy will usually present with a history of vaginal spotting and crampy pain as early as 4 to 5 weeks after the last menstrual period (LMP). The tubal pregnancy will usually bleed slowly at first, but when the pregnancy expands to the point of tubal rupture, patients can present with rapid hemorrhage, experiencing shock and then death. On physical exam, the patient will exhibit tenderness and often peritoneal irritation, especially if blood has leaked out of the fallopian tube. The uterus will be enlarged, but it will be smaller than expected for the age of the pregnancy. There is often cervical motion tenderness, and an adnexal mass may be felt. Abdominal pain with vaginal bleeding accompanied by hypotension, marked abdominal guarding, and rebound tenderness is suggestive of ruptured ectopic pregnancy. A serum pregnancy test may reveal a smaller quantitative value than expected by LMP. If the diagnosis is in question and the patient stable, serial beta-hCG values may be estimated. In ectopic pregnancy, the serial beta-hCG does not rise appropriately with respect to the gestational age (doubling every 48 - 72 hours). However, transvaginal ultrasound is usually utilized to look for the absence of an intrauterine gestational sac and the presence of an adnexal mass, which confirms the diagnosis of ectopic pregnancy. If a patient has a beta-hCG level of 1,500 mIU per mL or greater, but the transvaginal ultrasonography does not show the presence of an intrauterine gestational sac, then ectopic pregnancy should be suspected. Culdocentesis, with the finding of old, non-clotting blood, used to be performed commonly in the assessment of tubal pregnancy, but it has largely been replaced by ultrasound because this imaging technique has increased sensitivity. An untreated ectopic pregnancy is usually fatal. Early diagnosis is the key to appropriate treatment, and the mortality rate for ectopic pregnancy in the US is currently 10-15%. The treatment for ectopic pregnancy is usually surgical, especially if there has been tubal rupture. However, some early ectopic pregnancies may be treated with methotrexate. These patients must be followed carefully with serial serum pregnancy tests and ultrasounds, and up to 1/3 of these patients will still require eventual surgical intervention. While appendicitis, salpingitis, ovarian cyst, and tubo-ovarian abscess may also present with abdominal pain, the absence of specific symptoms for each of these helps in ruling them out; symptoms of abdominal pain, vaginal bleeding, and amenorrhea are characteristic for ectopic pregnancy.

A 22-year-old couple presents for the evaluation of the primary infertility. The woman has no symptoms; her family and gynecological history are not relevant. Her spouse regularly takes pancreatic enzymes, albuterol, and vitamins; he also frequently takes antibiotics because of the presence of the recurrent respiratory tract infections, bronchiectasis, and bronchiolectasis; there is also exocrine pancreatic insufficiency and intestinal dysfunction. Question What is the next step in the evaluation of their inability to conceive? Answer Choices 1 Sweat test 2 Serial ovarian sonography 3 Semen analysis 4 Progesterone test 5 Hysterosalpingogram

semen analysis Explanation The correct response is semen analysis. Infertility is due to the female partner 1/3 of the time, the male partner 1/3 of the time, and both partners or the unknown problems in the remaining 1/3 of cases. In this case, the husband has signs and symptoms that could cause problems conceiving. The most common causes of the male infertility are structural abnormalities, sperm production disorders, ejaculatory disturbances, and immunologic disorders. Symptoms, signs and the therapy this patient is is receiving should raise the possibility that he suffers the cystic fibrosis (CF). It is estimated that more than 95% of CF male patients are infertile. The most common findings is azoospermia due to the epididymal obstruction and/or the failure of the vas deferens to develop properly. A small minority of male CF patients are still fertile, and for that reason sperm analysis should be routinely offered in all CF patients. Sample semen is evaluated for volume, sperm count, motility and morphology. Patients with poor semen quality or numbers may be prepared for intrauterine insemination by washing and concentrating the ejaculate. Symptoms, signs, and the therapy that the husband is receiving should raise the possibility that he suffers the cystic fibrosis (CF). The diagnosis of CF rests on the combination of clinical criteria and abnormal CFTR (cystic fibrosis transmembrane conductance regulator), as documented by elevated sweat chlorde values, nasal potential difference responses, and the administration of the various solutions and by CFTR mutation analysis. Even if it is estimated that more than 95% of male CF patients are infertile, a minority with certain mutations is still fertile. For that reason, in all CF patients, sperm analysis should be routinely offered along with reproductive health education. Serial ovarian sonographic evaluations may help in demonstrating the existence of the ovulation: the development of a mature antral follicle and its subsequent collapse during ovulation. It is time consuming and is not always accurate; however, it may be helpful in supporting the diagnosis of polycystic ovary syndrome (PCOS). There is no reason to perform this test in a female patient with a non-contributiong personal and family history when the most probable cause is a husband's disease. The progesterone test is a test used to confirm ovulation; blood is drawn about 7 - 10 days after ovulation at a time when progesterone levels peak, and the level of progesterone helps in the confirmation of ovulation. There is no reason to perform this test in a female patient with a non-contributing personal and family history. Hysterosalpingogram (HSG) is a test of the patency of the Fallopian tubes and exploration of the atrial cavity. Dye is injected through the cervix and up into the uterus and Fallopian tube; its movement is observed on a video screen. There is no reason to perform this test in a female patient with a non-contributing personal and family history.

A 25-year-old female presents for an ultrasound after having a positive home pregnancy test. She has an unremarkable past medical history and physical exam. She states she has been feeling fine without any abdominal discomfort or vaginal bleeding noted. On ultrasound you determine she is 10 weeks pregnant. You note a noncomplex unilateral mass on her left ovary measuring 2 cm in diameter. Which additional history would support your suspected diagnosis? A History of herpes simplex virus B She used clomiphene to conceive C Two previous Caesarian sections D 20-pound unintentional weight loss

she used clomiphene to conceive B Patients who used assisted reproduction, such as clomiphene, present a special subgroup as their ovaries frequently have ovarian cysts. This is common during the first trimester due to ovarian hyperstimulation. Herpes (A), Cesarean sections (C), and miscarriages (E) are not proven to increase the risk of functional ovarian cysts. A 20-pound unintentional weight loss (D) would have you consider a malignant cause, which is much rarer than a functional ovarian cyst.

A 32-year-old pregnant woman presents for a screening ultrasound at 16 weeks gestation. The ultrasound notes that the fetus is male, has shortened long bones (all below the 5%), and a large head circumference. Question You refer the patient to a tertiary care center for further evaluation and explain that you suspect that the fetus may have what condition? Answer Choices 1 Tiisomy 13 2 Trisomy 18 3 Trisomy 21 4 Skeletal dysplasia 5 45, X

skeletal dysplasia Explanation In this case, there are signs of a skeletal dysplasia where bony growth of the long bones of the skeleton is not normal. Increased head circumference size can also be observed in the skeletal dysplasias. While achondroplasia is the most common recognized skeletal dysplasia, over 100 disorders have been reported and additional ultrasound imaging and possibly genetic testing will be needed to make a specific diagnosis. Trisomy 13 is often fatal in utero or in the first few days or weeks of life. Global growth retardation (not just long bone shortening), renal anomalies, cleft lip/palate, and severe central nervous system malformations may be noted on ultrasound. Trisomy 18 is often fatal in utero or in the first few days or weeks of life. Growth retardation may be present, along with renal anomalies, severe cardiac malformations, and omphaloceles. Skeletally, there may be shortening of the sternum and prominent calcanei on the feet. Trisomy 21 sometimes leads to shortening of the long bones in utero. although head circumference is often decreased. Ultrasound markers would include the presence of congenital heart disease (1/3 of all live born cases), duodenal atresia, and increased nuchal translucency. 45 X, or Turner syndrome, has few major structural problems in fetal development in spite of the fact that many 45 X fetuses will spontaneously miscarry. Ultrasound imaging may identify a short neck, increased nuchal translucency (or even a cystic hygroma), and/or webbing of the neck. Additionally, the phenotype of the fetus in this problem is male, and the phenotype of 45 X is female.

A 62-year-old obese woman presents due to urine leakage. She has had some symptoms for about 2 years, but they are getting worse. She leaks urine when she coughs or sneezes. She is not very active, but if she jumps, she also leaks urine. She wears a pad daily, as she has leakage daily. The amount varies from a few drops to a gush. The patient denies hematuria, dysuria, and pelvic pain. She sometimes feels vaginal pressure and fullness. She is considering quitting her job because she is embarrassed to be in public when she has urine leakage. Her past medical history reveals she is menopausal; she has had 4 vaginal deliveries. She has no other known medical conditions; she has not had any surgeries; she takes no medications and has no allergies. She is married and works part-time at a call center; she denies the use of tobacco, alcohol, and drugs. On physical exam, she is obese, with an atrophic vulva/vagina. The pelvic examination reveals downward and forward rotation of the vaginal wall, with an anterior bulging when the patient is asked to strain. The remainder of her exam is normal. A dipstick urinalysis is normal. What is the most appropriate intervention for this patient's current condition? Answer Choices 1 Antibiotic therapy 2 Daily cranberry juice or supplement 3 Start prescription oxybutynin 4 Surgical repair 5 Use over-the-counter phenazopyridine (Azo) as needed

surgical repair Explanation This patient is presenting with a cystocele, a herniation of the bladder wall into the vagina. Common symptoms may include stress urinary incontinence (SUI) and a feeling of vaginal fullness; other symptoms include incomplete voiding and dyspareunia. Of the choices listed, surgical repair is the most appropriate intervention. Other possible approaches to cystocele treatment include pelvic floor physical therapy and use of a pessary. The type of surgical approach to cystocele varies, from a less-invasive mesh sling to a more invasive colpopexy. Antibiotic therapy would be appropriate if this patient's urinary symptoms were due to a cystitis, or urinary tract infection (UTI). Common signs and symptoms of a UTI include urinary frequency and/or hesitancy, dysuria, and (possibly) gross hematuria; urinalysis shows white blood cells, red blood, cells and nitrites. Some women have both a cystocele and a UTI, so it is important to rule out infection when evaluating urinary tract problems. Daily cranberry juice and/or supplements are often used for urinary tract health. However, limited evidence supports cranberry's benefits related to UTI prevention. There is no evidence that cranberry juice or supplements is helpful for cystoceles and stress urinary incontinence. If this patient were suffering overactive bladder (detrusor instability) with resulting urge incontinence, it would be helpful to start prescription oxybutynin, an anticholinergic medication. However, this patient's problem is primarily anatomic, and most medications are ineffective in treating cystocele. If this patient was experiencing both stress and urge incontinence (mixed incontinence), the medication (along with surgery) would be indicated. Patients with frequent UTI may be instructed to use over-the-counter phenazopyridine (Azo) as needed. The phenazopyridine helps with bladder pain and has mild bacteriostatic properties. However, it does not have a role in cystocele and SUI. This medication also turns the urine bright orange, and it can interfere with urinalysis dipstick interpretation.

A 29-year-old woman comes to the emergency department with abrupt onset of left lower quadrant pain approximately 1.5 hours ago after she and her roommate moved multiple heavy pieces of furniture into their new apartment. The pain is now excruciating and she rates it "100 out of 10." She vomits on admission to the department. She is not currently sexually active and has no other significant past medical history. On exam she is found to have fullness and extreme tenderness in the left adnexa. Of the following, what is the most likely diagnosis? A Diverticulitis B Pelvic inflammatory disease C Renal calculus D Ruptured ovarian cyst E Torsion of the left ovary

torsion of the left ovary E Ovarian torsion is often extremely painful and is often accompanied by vomiting, with sudden onset and may be associated with episodes of heavy exertion. Diverticulitis (A) typically causes LLQ pain, but would be unusual in a 26-year-old woman because it occurs more often in older adults. The symptoms of PID (B) do not begin abruptly and vomiting is not typical. The pain of renal calculi (C) is colicky and calculi are not associated with a palpable adnexal mass. The pain of a ruptured ovarian cyst (D) would be sudden in onset; however, rupture would not be associated with a palpable mass.

A 30-year-old G1P0 woman who is 15 weeks pregnant undergoes "triple screening." The maternal alphafetoprotein (AFP), human chorionic gonadotropin (hCG), and unconjugated estriol (uE3) levels are all lower than normal. This suggests which of the following possible problems with the fetus? A cystic fibrosis B Down syndrome C homocystinuria D G6PD deficiency E trisomy 18

trisomy 18 The triple screen detects possible Down syndrome and trisomy 18. When the fetus has Down syndrome (B), the AFP and uE3 are low and the hCG is high, while trisomy 18 is suggested by low values in all three. A positive screen must be followed up by fetal karyotyping. Cystic fibrosis (A), homocystinuria (C), and G6PD deficiency (D) are not detected by the triple screen.

A 25-year-old woman presents with severe abdominal pain for the last few hours. The patient admits to pelvic pain and vagina discharge for over 1 month, but she refused to make an appointment with her provider to be evaluated. She last had unprotected intercourse with an ex-boyfriend 3 months ago. Her examination reveals a fever of 103oF and abdominally rigidity, with light palpation that is worst in the left lower quadrant. She does not tolerate a speculum exam. A pelvic ultrasound reveals a 4 x 5 cm left adnexal mass. Urine hCG is negative, WBC is elevated, with a left shift. What is the most likely diagnosis? Answer Choices 1 Appendicitis 2 Tubo-ovarian abscess 3 Ovarian cancer 4 Ectopic pregnancy 5 Diverticulitis

tubo-ovarian abscess Explanation This patient present with an acute abdomen that was preceded by pelvic pain and high-risk intercourse. Her history, physical examination, and diagnostic findings are all consistent with a tubo-ovarian abscess. In cases of appendicitis and diverticulitis, you would not expect to find an adnexal mass; the age and presentation of the patient are not consistent with ovarian cancer, and her negative hCG rules out pregnancy, including ectopic pregnancy.

Twins are at a higher risk for growth problems and prematurity, although uncommon defects unique to twins occur. Which of the following is the most common developmental defect in a monochorionic twin pregnancy? Answer Choices 1 Conjoined twins 2 Acardiac twins 3 Twin-twin transfusion 4 Twin embolization syndrome 5 Monoamniotic, monochorionic twins

twin-to-twin transfusion Explanation Twin-twin transfusion syndrome occurs in approximately 10-20% of monochorionic twins. Twin-twin transfusion syndrome (TTTS) is a condition that affects monochorionic twin pregnancies, in which blood is shunted from one twin to the co-twin through vascular anastomoses in the placenta. The donor twin becomes progressively anemic, hypotensive, and hypovolemic with resulting oligohydramnios and lagging growth. The recipient twin becomes polycythemic, hypertensive, and hypervolemic with resulting polyhydramnios and normal growth. The prognosis is fatal if left untreated, mortality rates approaching 70-90% in such cases. The sonographic findings suggestive of TTTS includes monochorionicity, discrepancy in amniotic fluid in the amniotic sacs with polyhydramnios of one twin (largest vertical pocket greater than 8 cm) and oligohydramnios of the other (largest vertical pocket less than 2 cm), discrepancy in size of the umbilical cords, cardiac dysfunction in the polyhydramniotic twin, and significant growth discordance (often > 20%). An acardiac parabolic twin (parasitic twin that fails to develop a head, arms and heart) occurs in approximately 1:35,000 pregnancies. Conjoined twins occur in 1:50,000 pregnancies. Monoamniotic, monochorionic twining is when the twins share one sac with both the amnion and chorion being a single sac. Twin embolization syndrome (TES) is a complication associated with monozygotic twins, following in utero demise of the co-twin.

A 22-year-old man presents with a right groin bulge. During physical assessment, a single sac is found protruding just lateral to the epigastric vessels. Although the clinical scenario is highly suggestive of a hernia, what initial imaging study would help support this diagnosis? Answer Choices 1 Radiograph 2 Ultrasound 3 CT scan 4 MRI 5 Doppler study

ultrasound Ultrasonography is a convenient, safe, noninvasive imaging study that can be used even at bedside in this patient scenario to help confirm this is a true hernia. It is an imaging technique that utilizes sonic energy, which is also termed a nonionizing type of energy; therefore, it is safe for use in all patients, including pregnant and pediatric patients. Images are created by echoes or reflections of the ultrasound beam as in interacts with tissues of different properties. Ultrasound waves are greatly reflected by superficial, air-soft tissue, and bone-soft tissue interfaces, making its usage somewhat limited. Because the inguinal structures are superficial, a linear transducer ultrasound is very effective at helping confirm diagnosis. Larger body habitus does have to be taken into consideration in terms of interference of the ultrasound results. Having the patient perform the Valsalva maneuver at various stages of the ultrasound can help identify if the hernia has some degree of being transient. The characteristic movement of the herniating tissues that will be visualized during an active ultrasound study usually helps confirm the diagnosis.

You are providing care for a 21-year-old G1P1 who delivered a 3990 gram infant by normal spontaneous vaginal delivery. After delivery, she continues to bleed vaginally, more than is expected for routine delivery. What is the most likely cause of her hemorrhage? A cervical laceration B lateral vaginal wall laceration C retained placenta D uterine atony E placenta acreata

uterine atony D Uterine atony continues to be the most common cause of postpartum hemorrhage, even though all of the answer choices may cause it.

A 32-year-old woman is 2-hours status post cesarean delivery of a twin gestation at 36 weeks. Her pregnancy was uncomplicated. She presented in early labor, which became prolonged despite oxytocin infusion. A cesarean section was performed when her labor became non-progressive. In the recovery area, she begins to complain of nausea and lightheadedness. On exam, her heart rate is 133 beats per minute and blood pressure is 76/42. Significant vaginal bleeding is noted, and abdominal palpation reveals a soft uterus. Question What is the most likely diagnosis? Answer Choices 1 Cervical laceration 2 Placenta accreta 3 Prolapsed uterus 4 Uterine atony 5 Uterine rupture

uterine atony Explanation The most likely cause of bleeding in this patient is uterine atony. Uterine atony is when the uterine myometrium fails to contract following delivery. Contractions of the uterine wall after delivery normally tamponade bleeding by tightening around uterine arterioles. Absence of this response causes continued bleeding, which usually becomes evident early after delivery. Bimanual uterine palpation revealing a soft, "boggy" uterus is what typically confirms the diagnosis. Risk factors include uterine distension caused by multiple gestations or polyhydramnios, as well as prolonged oxytocin use prior to delivery, placental previa, and chorioamnionitis. Treatment involves uterine massage and the use of medications that promote uterine contractions, including oxytocin, ergot alkaloids, and prostaglandins (e.g., Hemabate). Uterine atony can also occur as a result of retained products of conception; these products inhibit uterine contraction. In this case, manual or surgical extraction is necessary. Lower genitourinary tract or rectal tears may cause bleeding after delivery, the site of which is often visible on pelvic exam and amenable to suture ligation. Neither of these conditions impairs uterine contractions. Placenta accreta describes a condition in which the placenta implants within the uterine myometrium. Although it can be a cause of postpartum hemorrhage, it is relatively less common than uterine atony and would have been recognized at the time of cesarean delivery. Uterine prolapse (also called uterine inversion) describes the collapse of the uterine fundus through the cervical os. The uterus may be seen protruding from the vagina and abdominal exam reveals absence of the uterine body. It is a rare but serious event requiring rapid correction to restore hemodynamic stability. Uterine ruptureis also a rare complication involving a complete tear of the uterine layers with extrusion of uterine content into the abdominal cavity. It most commonly occurs at the site of a uterine scar from prior cesarean section; and it often happens before delivery of the fetus, requiring emergent laparotomy.

A 20-year-old college football player presents with a chief complaint of a dull ache in his scrotum after prolonged standing on the sideline. It seems to get worse with vigorous activity and is relieved by lying down. Dilated veins in the left scrotum are observed on inspection, and both testicles are palpable and without masses. What is the most likely diagnosis? A varicocele B spermatocele C hydrocele D testicular mass

varicocele A A varicocele can be recognized by the presence of scrotal enlargement caused by dilation of the pampiniform venous plexus. Varicoceles present as a "bag of worms" in the spermatic cord and are more prominent when the patient stands. More than 80% of the time, varicoceles occur on the left side. Hydroceles and spermatoceles are caused by fluid collection and are usually asymptomatic. Testicular masses must always be included in the differential diagnosis of scrotal masses, as they generally present as painless.


Conjuntos de estudio relacionados

"The First Industrial Revolution: Politics, Economics, Rights and Socio-Economic Inequality" study terms

View Set

AAPC - Chapter 1: Practical Application

View Set

Chapter 11: The Cardiovascular System

View Set

SCM301: Supply Chain Process Integration

View Set

Chapter 1: Nutrition (Wiley Questions)

View Set

Nevada Statutes & Regulations Common to Life and Health Only

View Set